Sei sulla pagina 1di 51

St. Louis Review Center 7.

A friend is shopping for a toy to give to her


2ND FLOOR CAP BLDG, J. ROSALES AVENUE, CORNER J.C. nephew. The friend knows nothing about children
AQUINO AVENUE, BUTUAN CITY
and asks what would be the most appropriate toy
TELEPHONE NUMBER (085) 342-2339
to give an 18-month-old child. Based on growth
PEDIATRIC NURSING and development skills, the nurse recommends a:
a. Tricycle c. Pull toy
1. The nurse discusses dental care with the parents b. Large ball d. Stuffed animal
of a 3-year old. The nurse explains that by the age 8. The nurse is preparing an 8-year old child for a
of 3, their child should have: procedure. What is the most appropriate nursing
a. 5 "temporary" teeth c. 15 "temporary" intervention?
teeth a. Provide visual aids, such as dolls, puppets,
b. 10 "temporary" teeth d. 20 "temporary" and diagrams in the explanation
teeth b. Provide a written pamphlet for the child to
2. The mother of a 6-month old infant is concerned review prior to the procedure
that the infant's anterior fontanel is still open. c. Discourage any display of emotional
The nurse would inform the mother that further outbursts
evaluation is needed if the anterior fontanel is d. Request that parents wait outside while the
still open after: nurse provides instructions to the child
a. 6 months c. 18 months 9. The nurse explains that the Pediatrics formula are
b. 10 months d. 24 months continued in a child's dietary intake up until what
3. The nurse has discussed appropriate support of age?
the young infant to prevent injuries from falls. a. 6 months c. 18 months
The mother who needs further education is the b. 12 months d. 24 months
mother who states: 10. Piaget identifies that the 2 to 7 year old chi8ld is
a. "My child is not allowed to have his walker in a preoperational stage. The nurse observes a
near the stairs." toddler taking a toy from another. The nurse
b. "I never leave my baby unattended on my recognizes the child unable to put him or herself
bed." in the place of another is displaying:
c. "By the time my infant is 6 months old, hw a. Centration c. Egocentrism
will be able to sit without support." b. Negativism d. Selfishness
d. "Before my child is standing, I need to place 11. A 7-month old infant has all of the following
the crib mattress at its lowest level. abilities. Which skill was most recently acquired?
4. The 9-year-old child is at the 98th percentile for a. Smiling at self in a mirror
weight and at the 40th percentile for height. The b. Transferring a rattle from one hand to the
school nurse will interpret that this child is: other
a. Underweight or small in stature c. Rolling from back to abdomen
b. Overweight or large in stature d. Imitating sounds
c. Experiencing a prepubescent growth spurt 12. The school health nurse is doing vision testing.
d. Normal for size visual acuity is assessed using:
5. In discussing sexual maturation with a health a. The Snellen eye chart c. The cover-uncover
class, the nurse would include the information test
that secondary sex characteristics begin to appear b. An ophthalmoscope d. The Weber test
at: 13. Children are usually brought to the clinic for
a. 10 years in girl, 12 years in boys health care by a parent. At what age is it
b. 12 years in girls, 16 years in boys appropriate for the nurse to question the child
c. 8 years in boys, 10 years in girls about presenting symptoms?
d. 12 years in girls and boys a. 3 years b. 5 years c. 7 years d. 9 years
6. A recently hospitalized 2-year-old client screams 14. When recording the health history of a child,
and shouts that he wants a "bottle." His parents what information that is uniquely pertinent to
are puzzled, and state that he has drank from a children is important for the nurse to obtain?
cup for the past year. The nurse explains that: a. Past hospitalizations c. immunization status
a. Irritability is exhibited in all age group b. Coping strategies d. Past accidents
b. Temper tantrums often represent the child's 15. A mother overhears a nurse state that the nurse
need for parental attention is going to complete a genogram and asks the
c. Various forms of punishment are necessary nurse what that means. The nurse's reply would
when such behaviors occur be based on knowledge that a genogram is useful
d. Regression to an earlier behavior often helps for visually showing what information?
the child cope with stress and anxiety a. Treatment protocols

St. Louis Review Center Inc. – BUTUAN CITY; Telephone No. (085) 342 – 2339 1
b. Family history Past history frequency is incorrect
c. Immunization status 22. The nurse administers cefprozil (Cefzil) as ordered
16. When plotting a child's height and weight on a to a 22-month-old client with bacterial
growth grid, the nurse understands that which pharyngitis. The nurse notes patches of white on
range represents the normal percentile range for the child's oral mucosa that cannot be removed.
children? Which condition does the nurse suspect?
a. 10th to 90th percentile a. Allergic reaction to the medication
b. 25th to 75th percentile manifested by the development of stomatitis
c. 50th to 100th percentile b. A herpes simplex virus infection
d. 5th to 95th percentile c. Oral thrush caused by Candida albicans
17. When assessing a child who complains of d. Mumps
abdominal pain, what is the most appropriate 23. A pediatric client has been diagnosed with otitis
nursing action? media. The nurse should place highest priority on
a. Palpate the most painful area first teaching the parent:
b. Palpate for rebound tenderness a. How to administer eardrop
c. Avoid painful areas until the end of the b. The importance of completing the full course
assessment of antibiotic therapy
d. Use deep palpation for abdominal tenderness c. About myringotomy and tympanostomy tube
18. When sharing the purpose of the Denver insertion
Development Screening Test (Denver II) with d. About eliminating environmental allergens
parents of an 18-month old, the nurse should 24. Nursing care of the child who is postoperative for
explain that: a tonsillectomy should include:
a. The Denver II is a test that will predict future a. Applying warm, moist compresses to the neck
intellectual ability area
b. The Denver II is a screening test used to b. Observing for excessive swallowing
detect children who may be slow in c. Maintaining the child in a supine position
development d. Offering warm liquids with a straw for the
c. The Denver II is used for early detection of child to sip
speech disorders 25. The nurse is caring for a child with a common cold
d. The Denver II measures psychological, (nasapharyngitis). The primary goal of nursing
cognitive, and social development care is directed toward:
19. What should the nurse do first when preparing to a. Preventing injury
do a physical assessment on a sleeping 8-month- b. Promoting nutrition
old baby? c. Relieving symptoms
a. Measure the occipital-frontal head d. Administering antibiotics
circumference 26. The nurse obtains a health history on a pediatric
b. Auscultate the heart and lungs client. A sign alerting the nurse to possible
c. Check the eyes for the red reflex hearing impairment in the child is:
d. Wake the baby a. Distractibility and short attention span
20. When preparing to examine a preschool child, the b. Disinterest in reading storybooks
nurse should: c. Turning up the volume on the family
a. Give detailed explanations to alleviate the television set
child's anxiety d. Temper tantrums
b. Give reassurance and feedback to the child 27. The nurse is caring for a 1-month old client who is
during the examination blind, secondary to retinopathy or prematurity.
c. Suggest that the child act like "the big kids" The nurse is teaching the parents about activities
when he or she is examined to promote their infant's development. Which of
d. Say that the shirt is the only clothing that the following statements by the nurse is correct?
must be removed a. "Infants with visual impairment respond to
21. The physician orders amoxicillin (Amoxil) 500mg tactile stimuli rather than auditory stimuli."
IVPB q 8 hours for a pediatric client with b. "Talking, holding, and singing to your baby
tonsillitis. What is the appropriate nursing action? are appropriate activities at this age."
a. Question the order because the route of c. "You should expect your baby to smile in
administration is incorrect response to your voice by 4 months of age."
b. Give the medication as ordered d. "Position the baby side-lying in the crib at all
c. Question the order because the dosage is too times, and avoid loud noises that could startle
high the infant."
d. Question the order because the dosing 28. The nurse is assessing a child with conjunctivitis

St. Louis Review Center Inc. – BUTUAN CITY; Telephone No. (085) 342 – 2339 2
(pink eye). Which of the following would the a. Throat culture c. Past medical history
nurse most likely assess? b. Vital signs d. Auscultation of chest
a. Serous drainage from the affected eye 34. The nurse is providing home care instructions to
b. Severe eye pain the parents of a child with cystic fibrosis. Which
c. Periorbital edema statement by the parents indicated that they do
d. Crusting of eyelids and eyelashes not understand the treatment regimen?
29. The nurse teaches a child with conjunctivitis a. "We will perform chest physiotherapy and
measures to prevent the spread of infection. The postural drainage four times a day."
nurse recognizes that further teaching is needed b. "We will keep her away from the church
when the child states the following: nursery if any of the children are coughing
a. "I will wash my hands frequently." and have fever or runny noses."
b. "I will use a tissue to clean my eye and then c. "If her bowel movements are normal and her
throw the tissue away." appetite is good, she does not need her
c. "I will use my own washcloth and towel, and pancreatic enzymes."
not use my brother's." d. "The relay races and swimming at our Sunday
d. "I will carry a handkerchief with me so that I school picnic next week will be good exercise
can wipe my eyes during the day." for her."
30. A 4-month old infant has severe nasal congestion, 35. A 2-year-old child is being discharged after
nasal mucous drainage and crusting in and bronchoscopy for removal of a coin from his
around the nares. What is the best way for the esophagus. The most important topic of
nurse to clear the infant's nasal passage? discharge teaching would be the importance of:
a. Administer vasoconstrictive nose drops every a. Reassuring the child that he is fine
3 hours b. Proper nutrition for the next few days
b. Place the infant in a mist tent c. Restricting his access to small toys or objects
c. Administer saline drops in the nose and d. Administering acetaminophen for his sore
suction with bulb syringe throat
d. Instruct the client to blow the nose and keep 36. A 15-year-old child with a history of cystic fibrosis
disposable tissues handy is admitted to the pediatric unit with assessment
31. The mother of an infant who has had recurrent findings of crackles, increased cough, and
respiratory infections asks the nurse why infants greenish sputum. A 2-week hospitalization is
are at increased risk for complications from anticipated. Which nursing intervention holds the
respiratory infections. The best response by the highest priority?
nurse explains that in infants, the: a. Referral to Child Life Services for school
a. Airway structures are larger, allowing for lesson plans
larger numbers of organisms b. Arranging for liberal visitation from peers
b. Respiratory rate is slower than in adults c. Taking a diet history
c. Parents are unable to accurately assess d. Gaining intravenous access
respiratory problems 37. A 7-year-old child is brought to the Emergency
d. Airways are narrower and more easily Department for an acute asthma attack. He is
obstructed wheezing, tachypneic, diaphoretic, and looks
32. The mother of a neonate hospitalized with upper frightened. The nurse should prepare to
respiratory tract infections asks why her baby administer:
won't take her bottle. The nurse's best answer a. IV methylprednisolone
would be: b. Racemic epinephrine
a. "She's probably not hungry." c. Oral prednisone
b. "It's okay because we're giving her d. Cromolyn sodium
intravenous fluids, therefore she is not 38. An appropriate nursing diagnosis for the family of
hungry." a toddler being treated for acute
c. "Newborns breathe through their noses. laryngotracheobronchitis to:
Congestion may be interfering with her a. Anticipatory grieving
breathing and eating at the same time." b. Altered growth and development related to
d. "She might need a different type of formula. acute onset of illness
We'll call the physician to get a new order." c. Impaired social interaction related to
33. A 4-year old female child presents to the confinement in hospital
emergency department with a sore throat, d. Fear/anxiety related to dyspnea and noisy
difficulty swallowing and a suspected diagnosis of breathing
acute epiglottitis. Which of the following should 39. A child with bacterial pneumonia is crying and
not be included in her initial assessment? says it hurts when he coughs. The nurse would

St. Louis Review Center Inc. – BUTUAN CITY; Telephone No. (085) 342 – 2339 3
teach the child to: discharge teaching, the nurse instructs the
a. Hug his teddy bear when he coughs parents:
b. Ask for pain medicine before he coughs a. To prevent the child from crying at all
c. Take a sip of water before coughing b. To observe the child for signs of increased
d. Try very hard not to cough intracranial pressure
40. An infant with chronic bronchopulmonary c. In cardio-pulmonary resuscitation
dysplasia (BPD) and a tracheostomy is being d. To identify growth and development
discharged on home oxygen therapy. Which milestones
statement by the mother indicates that further 47. A client with rheumatic fever is admitted to the
teaching is needed before discharge? nursing unit. The nurse's most important
a. "I will call my pediatrician if she gets a fever intervention is to:
or has more secretions than usual from her a. Prevent spread of rheumatic fever
tracheostomy." b. Provide comfort from arthralgia
b. "I have a cute bib to loosely cover her c. Evaluate for nervous system complications
tracheostomy when she eats and when we go d. Teach parents about cardiopulmonary
outside in the wind." resuscitation (CPR)
c. "We are so glad the baby will get to go with 48. A child with Kawasaki's disease is admitted to the
us on our camping trip to Yellowstone pediatric unit. Since promotion of comfort is an
National Park. We have been waiting for her appropriate nursing goal, the nurse:
to get well so we can go." a. Administers aspirin and immunoglobulins as
d. "We have already notified Alabama Power ordered
Company that our baby is coming home today b. Administers Tylenol and immunoglobulins as
41. An infant is admitted with an acyanotic heart ordered
defect. Which assessment finding should be c. Keeps child NPO for the first 24 hours
discussed with the physician? d. Encourages a vigorous exercise program
a. Heart murmur c. Weight gain 49. A pediatric client is discharged after an acute
b. Dyspnea d. Eupnea phase of rheumatic fever. The priority discharge
42. For an infant client with a cyanotic heart defect, instruction given by the nurse is that the child:
which symptoms would indicate risk for a. Is to resume regular activities
congestive heart failure? b. Needs to take antibiotics as ordered
a. Respiratory crackles and frothy secretion c. Needs to maintain complete bed rest
b. Decreased cyanosis d. Will experience central nervous system (CNS)
c. Increased blood pressure complications
d. Oxygen saturation increase 50. A pediatric client with a cyanotic heart defect
43. A child is admitted with a diagnosis of "rule out experiences a cyanotic episode. Symptoms
rheumatic fever". Which assessment finding consistent with this cyanotic episode would
supports this diagnosis? include:
a. Elevated antistreptolysin-O (ASO) a. Skin is ruddy or mottled prior to cyanosis
b. Elevated hematocrit b. Decreased rate of respirations
c. Decreased hemoglobin c. Decreased heart rate
d. Decreased salicylate level d. Lethargy
44. A child is admitted with possible coarctation of 51. A child has been admitted with a history of a
the aorta. Which of the following orders should seizure 2 hours ago. The history reports fever,
be questioned? chills, and vomiting for the past 24 hours. In
a. Regular diet report, the nurse is told that the child has a
b. BP of upper and lower extremities q 4 hours positive Brudzinski's sign. The nurse knows that
c. Intake and output this is most likely caused by:
d. Vital signs on admission, then Q.D. a. Increased intracranial pressure
45. A child with tetralogy of Fallot becomes acutely ill b. Meningeal irritation
with an increase in cyanosis, tachycardia, and c. Encephalitis
tachypnea. Which nursing action would be most d. Intraventricular hemorrhage
effective to relieve cardiac load? 52. A nurse is assessing a new admission. The 6-
a. Place child in Trendelenburg position month-old infant displays irritability, bulging
b. Place child in knee-chest position fontanels, and setting-sun eyes. The nurse would
c. Have oxygen equipment available suspect:
d. Maintain suction equipment available a. Increased intracranial pressure
46. A child with a cyanotic heart defect is being b. Hypertension
discharged home to await surgical repair. In the c. Skull fracture

St. Louis Review Center Inc. – BUTUAN CITY; Telephone No. (085) 342 – 2339 4
d. Myelomeningocele department following a head injury from football.
53. An 8-year-old client with a ventriculoperitoneal During the first few hours after admission, he
shunt was admitted for shunt malfunction. He sleeps unless awakened, but he can be aroused
presents with symptoms of increased intracranial easily and is oriented. In charting assessment
pressure. The mechanism of the development of findings, the nurse would describe this level of
his symptoms is most probably related to: consciousness as:
a. Increased flow of cerebrospinal fluid a. Semicomatose c. Obtunded
b. Increased reabsorption of cerebrospinal fluid b. Lethargy d. Stuporous
c. Obstructed flow of cerebrospinal fluid 60. A young child has just been diagnosed with
d. Decreased production of cerebrospinal fluid cerebral palsy. The nurse is teaching the parents
54. A child with a myelomeningocele is started on a how to meet the dietary needs of their child. The
bowel management plan. the child's mother nurse would explain that children with cerebral
questions why this is being done. The nurse's palsy frequently have special dietary needs or
response will be based on the understanding feeding challenges because:
that: a. The paralysis of their muscles decreases their
a. Lack of innervations to the colon predisposes caloric need
the child to diarrhea b. The spasticity of muscles increases their
b. Lack of innervations to the anal sphincter caloric need
predisposes the child to being incontinent c. All children with cerebral palsy require
c. Lack of mobility increases the gastric-colic assistance with feedings
reflex d. The child's inactivity increases the risk of
d. Lack of mobility decreases the need for obesity
regular bowel movements 61. A newborn is found to have exstrophy of the
55. A child has just been diagnosed with bacterial bladder. The nurse should evaluate the infant for:
meningitis. The parent asks the nurse how long a. Hypospadias c. Cryptorchidism
the child will be in isolation. The nurse's reply will b. Epispadias d. Acute tubular necrosis
be based on a protocol that isolation continues 62. A child has been admitted to the hospital with a
until: diagnosis or “rule out nephrosis.” The nurse
a. The organism is located would assess the child for:
b. The antibiotics are initiated a. Hematuria c. Petechial rash
c. The antibiotics have been administered for 24 b. Edema d. Dehydration
hours 63. The nurse is caring for a toddler who is not toilet-
d. Ten days of antibiotic therapy have been trained. The doctor has ordered intake and
completed output measurement. The nurse will most
56. The nurse observes a client with the neck and accurately measure the urine by:
back arched and extremities severely extended. a. Estimating output as small, moderate, or
The mother asks why the child is doing that. The large and recording on the child’s chart.
nurse explains that this posturing is called: b. Weighing each wet diaper and recording the
a. Decerebrate c. Jacksonian seizure amount of urine output as the weight of the
b. Decorticate d. Opisthotonos diaper.
57. A child is being treated for increased intracranial c. Subtract the weight of a dry diaper from a
pressure (ICP). Which intervention would be wet diaper and record this amount.
contraindicated to order to decrease ICP? d. Determine urine output by the number of
a. Keeping head of bed at a 30-degree angle diaper changes in each 24-hour period.
b. Providing supplemental oxygen 64. The nurse is teaching the parents of preschooler
c. Turning head to one side information about urinary tract infection and
d. Administering IV osmotic diuretics as ordered means of reducing their recurrence. Which
58. A 10-year-old boy receives a blow to his head statement by the parents indicated the need for
with a hard baseball and is admitted to the additional teaching?
hospital for observation. If the child were to a. “I should try to get her to drink a lot of water
develop an epidural hematoma, the child would and juices.”
most like display symptoms b. “I will buy her underwear a little large.”
a. In the emergency room or soon after arriving c. “Soaking in a bubble bath will reduce meatal
on the unit irritation.”
b. On the unit over the next few days d. “If I notice her starting to wet the bed again, I
c. After discharge home need to have her checked for another urinary
d. Over the next two months tract infection.”
59. A 15-year-old client is seen in the emergency 65. The nurse would include which of the following in

St. Louis Review Center Inc. – BUTUAN CITY; Telephone No. (085) 342 – 2339 5
the care of a child with acute glomerulonephritis? time to collect this specimen because:
a. Careful handling of edematous extremities a. At 24 hours, the T4 level will be extremely low
b. Observing the child for evidence of b. There is an immediate rise in the TSH after
hypotension birth
c. Providing fun activities for the child on bed c. The baby needs to digest formula before a
rest blood sample can be taken
d. Feeding the child a protein-restricted diet d. A thyroid scan should be done first
66. A urinalysis is ordered for a child with a throat 73. The nurse is administering propylthiouracil to a
culture positive for group-A beta-hemolytic 12-year old female recently diagnosed with
streptococcus (strep throat). The mother asks Graves’ disease. The child has been receiving the
why this test is being ordered. The nurse Explains: drug 3 times a day for 3 weeks. She suddenly
a. The urinalysis will indicate whether an HIV complains of severe soar throat. What would be
infection is also present. the appropriate nursing action?
b. Urinary tract infections are common with a. Continue to give the medication or she will
streptococcal infections and need to receive continue to exhibit signs of Grave’s disease
prompt treatment. b. Offer lozenges for the relief of the sore throat
c. Pyelonephritis is a potential complication of c. Hold that dose and report the complaint to
antibiotic therapy. the physician since a sore throat is a common
d. Group-A beta-hemolytic streptococcus side effect
infections can be followed by the d. Assume that she is complaining in order to
complication of acute glomerulonephritis. avoid going to the school room in the hospital
67. An appropriate nursing diagnosis for a toddler 74. A 10-year old diabetic client told the nurse that
with unrepaired exstrophy of the bladder would he had some early signs of hypoglycemia while
be: attending school. What would be the best action
a. Disorganized infant behavior c. Urinary for the child to take?
retention a. Take an extra shot of regular insulin
b. Sexual dysfunction d. Risk for infection b. Drink a glass of orange juice
68. A child has been admitted to the unit with acute c. Skip the next dose of insulin
glomerulonephritis. The test that would confirm d. Start exercising
this diagnosis is: 75. The nurse is teaching an adolescent client about
a. Antistreptolysis-O (ASO) titer the different types of insulin. The client takes
b. Urinalysis NPH insulin at 8:00 A.M. The nurse would instruct
c. Blood cultures d. White blood cell (WBC) the client that he could possibly expect an insulin
count reaction at what time of the day?
69. The doctor orders a clean-catch urine specimen a. While working out at 9:00 A.M.
on an infant who is not toilet-trained. The best b. While taking a test at 10:00 A.M.
means of collecting this urine would be to: c. While eating lunch at noon
a. Perform a straight catheterization d. While golfing after school at 3:00 P.M.
b. Apply a urine collection bag 76. A teenage mother arrives at the clinic with her
c. Use diaper analysis new baby who has recently been diagnosed with
d. Perform Foley catheterization congenital hypothyroidism. When instructing the
70. A teenage child is being treated for renal failure. mother about administering levothyroxine
The nurse would ensure that the child follows a: medication, the nurse would include the
a. High-sodium diet c. Low-sodium diet information that she should:
b. High-protein diet d. Low-fiber diet a. Crush the medication and place in a full bottle
71. A mother tells the nurse that her child has been of formula or juice to disguise the taste
diagnosed with hypertrophy of the thyroid gland. b. Administer the medication every third day
The mother states the doctor called it something c. Give the crushed medication in a syringe or in
else and asks the nurse what the other name for the nipple mixed with a small amount of
this is. The nurse’s reply should be: formula
a. Glandular enlargement c. Lymphadenopathy d. Understand that the medication will not be
b. Goiter d. Thyrotoxicosis needed after age 5
72. An infant was born 24 hours ago. The nurse has 77. A new mother of an infant diagnosed with
been instructed to collect blood by heel stick for phenylketonuria (PKU) meets with the nurse who
neonatal screening for congenital hypothyroidism informs her that PKU follows autosomal recessive
before the baby is discharged. The nurse inheritance. The mother states that is a relief
discusses options with the pediatrician knowing since she now knows her next baby will not have
that 24 to 48 hours after birth is not the optimal the disease. What additional information does

St. Louis Review Center Inc. – BUTUAN CITY; Telephone No. (085) 342 – 2339 6
the mother need? being asleep
a. With autosomal recessive inheritance, each d. Skin surrounding the cast is warm
baby has a 25 percent chance of having the 82. Which of the following nursing care measures
disease takes highest priority in caring for a child in
b. Only female babies will have PKU skeletal traction?
c. The mother passes the gene only to male a. Assessing bowel sounds every shift
offspring b. Assessing temperature every 4 hours
d. Since she already has one baby with the c. Providing adequate nutrition
disease, the next one will probably be a d. Providing age-appropriate activities
carrier for the disease 83. A nurse performs a triage in a pediatric
78. A 4-month old infant has been diagnosed with orthopedic clinic. Which of the following should
PKU. The child has eczema and sensitivity to the the nurse recognize as a symptom of slipped
sunlight. The mother asks the nurse why her capitol femoral epiphysis?
child’s skin is so sensitive. An appropriate a. Pain in the hip of a preadolescent child
explanation by the nurse would be: b. Acute onset of knee pain
a. “Some children just have sensitive skin.” c. Presence of a limp in a school aged child
b. “Your child will outgrow his sensitivity when d. Painful external rotation of the affected leg
he is 5 years old. Just use sunscreen for now.” 84. Which of the following statements made by the
c. “Your child has a deficiency in melanin parent of a child being discharged with
because of decreased tyrosine. You will osteomyelitis requires further teaching by the
always have to take special care of his skin.” nurse?
d. “The phenylketones in your baby’s blood a. “I can stop the antibiotics when I see that my
concentrate the sun’s rays, making burning child is feeling better.”
more likely. Children with PKU can never be in b. “We will make sure that our child has plenty
the sun. of calcium and protein.”
79. The nurse was working with a group of parents of c. “I will look at the intravenous site for signs of
children with phenylketonuria. The nurse has infection a couple of times a day.”
completed family teaching on the dietary d. “My child won’t take physical education at
restrictions. The parents are given sample menus school until allowed by the doctor.”
to choose a meal for their child. Which menu 85. A 5-month-old infant is being assessed for
choice indicates understanding of the dietary develop0mental dysplasia of the hip. The nurse
instructions? will look for a positive:
a. A hamburger and a diet soda sweetened with a. Ortolani sign c. Allis sign
aspartame b. Barlow sign d. Trendelenburg sign
b. Steak and mashed potatoes with orange juice 86. A newborn is being admitted to the newborn
c. A large bowl of cereal with strawberries and nursery. The nurse would assess the infant for
apple juice congenital defects. In addition to the abnormal
d. Milkshakes and grilled cheese sandwich position of the foot, the nurse would note which
80. Mothers in the waiting room of the endocrine of the following if clubfoot were present?
clinic are discussing their children’s illnesses. The a. Affected foot is larger and longer
mothers of children with phenylketonuria and b. Affected limb is longer
congenital hypothyroidism recognize there is a c. There is calf muscle atrophy of the affected
common goal in the early treatment of their limb
children. That goal is the avoidance of: d. Affected foot is cooler
a. Mental retardation c. Obesity 87. A child is admitted with osteogenesis imperfecta
b. Fever d. Protein foods (OI). In reviewing laboratory findings, the nurse
would expect to find abnormal levels of:
81. A 6-year-old child has a cast applied for a a. Calcium c. Precollagen type I
fractured radius. The nurse completes an b. Phosphorus d. Vitamin D
orthopedic assessment on this child. Which of the 88. Which of the following statements made by a
following symptoms requires immediate parent of a child with OI needs clarification by the
attention and should be reported to the nurse?
physician? a. “My child may be able to participate in
a. Capillary refill of 4 seconds in the affected sports.”
hand b. “There are no medications available to help
b. Edema in the affected fingers that improves this disease process.”
with elevation c. “Surgery may be needed to place rods in the
c. Child describing feeling of the affected hand bone for stability.”

St. Louis Review Center Inc. – BUTUAN CITY; Telephone No. (085) 342 – 2339 7
d. “My child will need to be home schooled to a. Continuing oral antibiotics until the
protect him from injury.” prescription is completed
89. Which of the following interventions is b. Strict bed rest with the left leg elevated
inappropriate to incorporate into the plan of care c. Increased fluid intake
for a child with Duchenne muscular dystrophy d. Limiting visitors to prevent spreading
hospitalized for a respiratory infection? infection
a. Physical therapy 99. When teaching a mother how to use an anti-lice
b. Vigorous antibiotic therapy shampoo, the nurse should include the
c. Passive range of motion exercises information that she should:
d. Strict bed rest a. Use ample shampoo to cover the hair
90. A 14-year-old adolescent has just been fitted for a b. Apply about 2 ounces of shampoo to wet hair
Milwaukee brace. Which of the following should c. Leaver the shampoo on hair for 20 to 30
the nurse include in teaching about this brace? minutes before rinsing
a. The brace should be worn only when the d. Use hot water for both shampoo and rinse
adolescent is sleeping or in the recumbent 100. The nurse would share which of the following
position pieces of information to increase treatment
b. The brace should be worn next to the skin compliance in a teenager with eczema?
c. Exercises to increase pelvic tilt should be a. The appearance of the skin will improve in a
done several times per day while in the brace few days
d. The adolescent should experience no pain as b. Avoiding foods with eggs and milk will speed
a result of wearing this brace healing
91. When bathing a 3-year-old with eczema, the c. Scarring is not likely if the treatment plan is
nurse tells the mother to have the bathwater: followed
a. As hot as the child can tolerate d. This problem will not likely recur past
b. Hot to the touch on the inner wrist adolescence
c. Tepid d. Cool
92. The nurse explains to the mother that a child who
has begun treatment for impetigo with a topical
“There is no limit to the good a person can do if
antibiotic can return to daycare:
he is willing to let someone else take the credit.”
a. Immediately c. As soon as crust are evident
b. After 48 hours d. When crusts fall off
93. When assessing a child’s hair and scalp, the nurse
notices what looks like dandruff but it does not
flake off easily. The nurse suspects the child has:
a. Scabies c. Pediculosis capitus
b. Eczema d. Impetigo
94. The nurse plans to position a child with a
circumferential burn of the right leg”
a. Flat in bed
b. With the right leg dependent
c. On the left side
d. With the right leg elevated
95. A 3-year-old child has been diagnosed with
eczema. The nurse will assess for:
a. Pruritus c. Vesicles
b. Pustules d. Lichenification
96. A child has been diagnosed with eczema. While
taking the nursing history, the nurse will assess
for a family history of:
a. Scabies b. Cellulitis c. Asthma d. Impetigo
97. When assessing a child with Periorbital cellulites,
the nurse will want to ask the parent about a
recent history of:
a. Otitis media c. Dog bite
b. Sinusitis d. Sun exposure
98. A child will be treated at home for cellulites of the
left leg. The nurse will include in client education
the need for:

St. Louis Review Center Inc. – BUTUAN CITY; Telephone No. (085) 342 – 2339 8
antibiotics for an infection. The physician has ordered
gentamycin (Garamycin). Because of the side effects of
this drug, the nurse would monitor:
a. Temperature
b. Blood pressure
c. Intake and output
d. Breath sounds
(Answer: C) Rationale: One of the most common side
effects of gentamycin is nephrotoxicity. The nurse can
monitor kidney function by monitoring intake and output.

102. A 3-year-old child is admitted to the hospital to rule


out an infection. Which diagnostic test is likely to
differentiate an infection from an allergic response?
a. Hemoglobin and hematocrit
b. Red blood cell count
c. White blood cell count differential
d. Platelet agglutinization
(Answer: C) Rationale: White blood cells are one
component of the general nonspecific immune response.
They among the first responders stimulated by a
pathogenic organism. A white blood cell differential can
often determine if the illness is of bacterial, viral, or
allergic origin.

103. A 2-year-old child has eczema that causes extreme


itching. Treatment has not been able to control the rash.
It has been determined that the primary allergen is
wheat. An appropriate nursing diagnosis for this child
would be:
a. Risk for infection
b. Altered nutrition, more than body requirements
c. Ineffective infant feeding behavior
d. Noncompliance
(Answer: A) Rationale: Because of the itching, the child
will be scratching. Intense scratching can break the skin,
and the child might develop a bacterial infection
secondary to the skin trauma. Altered nutrition, more
than body requirements, does not clearly state the
problem with the food allergies, nor does ineffective
infant feeding behavior. There is no evidence of
noncompliance.

104. A child’s mother tells the nurse that her child has
been on steroids for several months. Which of the
following vaccines is contraindicated?
a. Tetanus toxoid
b. Measles, mumps, and rubella (MMR) made from
egg embryo
c. Poliovirus vaccine inactivated
d. Poliovirus vaccine live oral trivalent
(Answer: D) Rationale: Oral poliovirus vaccine contains
a live virus, which could cause an infection in a child who
is immune-depressed as a result of taking steroids.

105. A child with severe combined immunodeficiency


disorder (SCID) is being discharged from the hospital to
home. The nursing care goal for the client before and
after discharge would be that the child:
a. Remains well oxygenated
b. Remains free of infection
c. Maintains hydration
d. Avoids contact with other people
(Answer: C) A teenager can and should be part of the (Answer: B) Rationale: Care of the immunocompromised
treatment plan. If itching is avoided to prevent child focuses on preventing infection. The nursing
excoriation and secondary infection, scarring is unlikely. interventions related to reaching this goal might include
Improvement is often slow, and the problem may persist limiting contact with a large number of people, but that
into adulthood. Food avoidance will not change the would not be the goal of the nursing care plan.
course of the disease.
106. A child is being worked up for allergies. The mother
101. A 14-year-old child is receiving intravenous asks how the diagnosis will be made. The nurse’s

St. Louis Review Center Inc. – BUTUAN CITY; Telephone No. (085) 342 – 2339 9
response is based on the knowledge that allergies can “belly button” during bathing. Initial assessment should
be diagnosed based on: not include:
a. Medical history of urticaria alone a. Measuring weight and height
b. IgG levels b. Further palpation often area
c. Decreased eosinophil count c. Performing urine testing
d. RAST test d. Taking vital signs
(Answer: D) Rationale: Allergies are confirmed by RAST (Answer: B) Rationale: This is the usual presentation of
test. RAST is a radioallergosorbent test that detects IgE Wilms’ tumor (nephroblastoma), and palpating the area
antibodies that are part of the allergic response. Urticaria may cause the tumor to spread. Since Wilm’s tumor is a
is itching and is symptomatic of allergies and other cancer of the kidney, it important to assess growth and
diseases, and an increase in eosinophils is diagnostic of development, kidney function, and blood pressure, which
allergies. may be elevated because of increased rennin
production.
107. A child is born with microcephaly. Part of his
assessment includes a TORCH test. In providing client 112. The parent of a child with neuroblastoma verbalizes
education, the nurse explains to the mother that TORCH regret at not coming in earlier for the client’s complaints.
test will assess for: An appropriate response is:
a. Presence of the TORCH virus a. This tumor may be diagnosed early because of
b. Complications of pregnancy obvious symptoms
c. Presence of one or more specific viruses b. This is a silent tumor, which is difficult to
d. Evidence of thalidomide poisoning diagnose early
(Answer: C) Rationale: The acronym TORCH stands for c. This is a very common brain tumor in children
toxoplasmosis, other (syphilis, hepatitis), rubella, d. This is the most common childhood cancer
cytomegalovirus, and herpes simplex virus. It is a study (Answer: B) Rationale: This tumor occurs in 1 in 10,000
of common viruses that cause significant fetal damage. live births. It arises out of embryonic neural crest cells
and, therefore, is usually found in the adrenals or
108. A 4-month-old infant has been admitted with a retroperitoneal sympathetic chain. Symptoms are vague
diagnosis of sepsis. The nurse would monitor the child and depend on location.
for evidence of:
a. Hypothermia 113. A 4-year-old is diagnosed with acute lymphocytic
b. Rash leukemia. Following teaching about the testing and
c. Sunken fontanels therapy, the nurse evaluates the family’s understanding
d. Glucosuria of the problem. The statement by the family that
(Answer: A) Rationale: Neonates with sepsis may indicates appropriate knowledge would be:
display either hypothermia or hyperthermia, but a. “Tests will determine the extent of the tumor
hypothermia is more common. The other symptoms are process and support the need for palliation.”
not associated with sepsis. b. “Tests will help to determine if radiation or
chemotherapy should be used in the treatment
109. A 12-year-old child with HIV + antibodies is going plan.”
home from the hospital. Which of the following are the c. “Tests will determine if surgery is needed.”
most important home-going instructions? d. “Tests will determine the extent of eth malignant
a. Growth and developmental milestones process and stage the leukemia.”
b. Immunization schedules (Answer: D) Rationale: Acute lymphocytic leukemia is
c. Lab studies and results stages at diagnosis to determine treatment. The goal is
d. Prevention of the spread of HIV remission, which is usually accomplished using
(Answer: D) Rationale: Families need to know that chemotherapy. Radiation therapy to the central nervous
casual contact cannot spread HIV. However, basic system is rarely used because of untoward side effects.
infection control practices must be maintained t prevent
exposure through body fluids. 114. A school health nurse would suspect a brain tumor
after noting the presence of which of the following
110. A 4-year-old has been exposed to chickenpox. After symptoms that is compatible with this health problem?
the nurse has provided information about chickenpox, a. Ataxia and irritability
the nurse asks the mother to repeat the information. The b. Papilledema and positive red reflex
statement by the mother that indicates a need for c. Headache and vomiting
additional information is: d. Fever and seizures
a. “During the prodomal period, my child will have (Answer: C) Rationale: The most common reported
pox all over his body.” symptoms of brain tumors in children are headache,
b. “Chickenpox is a viral infection that can be especially upon awakening, and vomiting that is
spread to other children.” unrelated to eating. Both are related to increased
c. “I should monitor my child for Reye syndrome, intracranial pressure. Irritability and ataxia may also be
which is a complication of chickenpox.” present; however, presenting symptoms are often
d. “My child should not visit my pregnant sister at vague. Fever is not a symptom of a brain tumor.
this time.” Papilledema may be noted, but red reflex is not
(Answer: A) Rationale: The prodomal period refers to the indicative of brain tumors.
period of time between the initial symptoms and the
presence of the full-blown disease. The rash would not 115. An adolescent is being admitted for an amputation
be apparent during this time. All the other statements are related to a bone tumor. Which age-related nursing
correct. diagnosis is most appropriate?
a. Risk for disuse syndrome
111. An 18-month-old client is brought in for a well child b. Body image disturbance
visit. The parent reports feeling a lump to the right of the c. Self-care deficit

St. Louis Review Center Inc. – BUTUAN CITY; Telephone No. (085) 342 – 2339 10
d. Activity intolerance b. Lung sounds
(Answer: B) Rationale: Bone tumors usually occur in c. Temperature
other wise healthy children. Given the interruption of d. Kidney function
normalcy and the developmental tasks of the (Answer: D) Rationale: All of these assessments look at
adolescent, body image disturbance can occur when a possible postoperative complications. Since the child is
limb is lost. left with only one kidney, failure of that kidney caused by
inadequate blood flow, infection, or any other cause
116. A child has been treated with chemotherapy for could be fatal.
cancer. Neutropenia is an expected consequence. The
nurse would teach the parents to do which of the 120. A child will be undergoing chemotherapy. The
following, anticipating that neutropenia may occur? nurse discusses the issue of hair loss with the child and
a. Avoid contact sports family before chemotherapy begins. Later the family
b. Avoid crowded spaces questions the nurse on why this information was given to
c. Avoid spicy foods the child at this time. The nurse’s response will include
d. Avoid all immunizations the information that:
(Answer: B) Rationale: Neutropenia is a reduced white a. Hair loss is a symptom of toxic blood levels of
blood cell count, which increases the risk for infection. chemotherapy, so the child should be watching
Only live vaccines are contraindicated in children who for this phenomenon
are immunocompromised. Contact sports would be a b. The presence or absence of hair is related to
problem with thrombocytopenia, and spicy foods would body image. Strategies for handling hair loss
increase discomfort if an alteration in mucous should precede the event.
membranes occurred. c. It is the nurse’s legal responsibility to discuss
this issue with the child
117. A child diagnosed with Ewing’s sarcoma is being d. Hair loss can be prevented with appropriate
treated with chemotherapy. The results of a complete nursing interventions
blood count (CBC) indicate severe thrombocytopenia. (Answer: B) Rationale: Preparation helps individuals
Nursing interventions related to this finding would handle stressful situations. If the child had not been
include: prepared for hair loss, it could be more anxiety provoking
a. Encouraging foods high in iron for the child. Hair loss cannot be prevented.
b. Limiting physical contact with the child
c. Removing fresh flowers from the child’s room 121. An 18-month-old child with a history of cleft lip and
d. Clearing the floor of the child’s room to prevent palate has been admitted for palate surgery. The nurse
falls and bruises teaches the parents not to use a toothbrush immediately
(Answer: D) Rationale: Thrombocytopenia refers to a after surgery because:
decrease in platelets. Preventing falls and bruises would a. The Toothbrush would be frightening to the child
be appropriate for an individual with platelet deficiencies. b. The child no longer has deciduous teeth
Fresh flowers may contain molds and fungus that can c. The suture line could be interrupted
lead to infection and would be a concern for a child wit d. The child will be NPO
neutropenia. Providing foods high in iron would be (Answer: C) Rationale: During the immediate
appropriate to restore red blood cells. Limiting contact postoperative period, protecting the operative site is a
with the child could affect his or her body image and self- priority in the nursing care of this child. A toothbrush
esteem. Contact is acceptable as long as the individual should be a familiar object to an 18-month-old child.
is not infectious. Deciduous (primary) teeth are still present at this age
and are replaced by permanent (secondary) teeth
118. The parents of a child with neutropenia secondary around 6 years of age. Oral care will be performed
to chemotherapy have been taught protective isolation according to the physician’s orders but usually consists
behaviors. Nursing observations that indicate a need for of cleansing the area with sterile water.
further education is when the parents:
a. Bring the child toys from home 122. The nurse instructs the parents about the
b. Encourage friends to visit by phone rather than postoperative feeding schedule following their infant’s
visit pyloromyotomy. The nurse evaluates that the parents
c. Pull the child in a wagon around the nursing unit understand the instructions when they state they may
for entertainment begin feedings:
d. Wash their hands before entering the child’s a. 6 hours postoperatively
room but not upon exiting the room b. 8 hours postoperatively
(Answer: C) Rationale: Healthy children are often a c. 10 hours postoperatively
source of infectious organisms. Children in hospitals d. 12 hours postoperatively
may carry a number of infectious organisms. (Answer: A) Rationale: Clear liquids are usually
Hospitalized neutropenic children should be protected prescribed 4 to 6 hours after surgery.
from exposure to other children whenever possible. Toys
form home would not carry a higher risk. Hand washing 123. Shortly after the delivery of an infant with an
before contact with the child is the important omphalocele, the nurse would initially:
intervention. Limiting physical contact with peers would a. Weigh the infant
decrease exposure to infectious organisms. Telephone b. Insert an orogastric tube
contacts allow for the peer support the child needs. c. Call the blood bank for 2 units of blood
d. Cover the sac with moistened sterile gauze
119. Following diagnosis of Wilms’ tumor, the child (Answer: D) Rationale: Omphalocels are congenital
undergoes removal of the affected kidney. In the post- malformations in which abdominal contents protrude
operative period, priority-nursing assessments should through the umbilical cord. A translucent sac covers the
focus on: protrusion; immediately after birth, the sac requires
a. The incision priority attention. The sac is covered with sterile gauze

St. Louis Review Center Inc. – BUTUAN CITY; Telephone No. (085) 342 – 2339 11
soaked in normal saline solution to prevent drying and b. Until the child has achieved all major
injury. developmental milestones
c. Only until all symptoms are resolved
124. While gathering admission data on a 16-month-old d. Until the child has reached adolescence
child, the nurse notes all the following abnormal findings. (Answer: A) Rationale: Discharge planning focuses on
Which finding is related to a diagnosis of Hirschsprung’s educating the parents in maintaining a gluten-free diet
disease? for the child. Dietary modifications are life long and
a. Projectile vomiting lacking bile should not be discontinued when the child is symptom-
b. Decreased urine output free. Symptoms will return if dietary restrictions are not
c. Weights less than expected for height and maintained.
weight
d. Intermittent sharp pain 128. An appropriate nursing assessment of an infants
(Answer: C) Rationale: Infants with Hirschsprung’s suspected of having necrotizing enterocolitis would be:
disease usually display failure to thrive, with poor weight a. pH evaluation of the stomach
gain and delayed growth. Projectile vomiting is usually b. Neurological status every 2 hours
associated with pyloric stenosis. Decreased urine output c. Rectal temperature every 2 hours
and intermittent sharp pain are nonspecific symptoms d. Abdominal girth every 4 hours
that can be associated with many different diseases and (Answer: D) Rationale: Measuring the abdominal girth
disorders. frequently aids in early detection of necrotizing
enterocolitis, which, in turn, minimizes loss of bowel.
125. A 6-week-old boy is brought into the pediatrician’s Assessment of stomach pH is not done. Frequent
office with a history of frequent vomiting after feedings assessment of the neurologic status is not specific to this
ands failure to gain weight. The diagnosis of disease. Rectal temperatures are contraindicated
gastroesophageal reflux is made and discharge because of the increased risk of perforation.
instructions are begun. While planning discharge
teaching on feeding techniques with the parents, the 129. The nurse is developing a teaching plan for the
nurse should include instructions to: parents of an infant diagnosed with hepatitis A. Which of
a. Dilute the formula the following instructions would be included to reduce
b. Delay burping to prevent vomiting the risk for transmission of this disease?
c. Change from milk-based formula to soy-based a. Disinfect all clothing and eating utensils on a
formula daily basis
d. Position the infant at a 30 to 45 degree angle b. Tell family members to wash their hands
after feedings frequently
(Answer: D) Rationale: Small, frequent feedings followed c. Spray the yard to eliminate infected insects
by placing the infant at a 30 to 45 degree angle has d. Vacuum the carpets and upholstery to rid the
been shown to be beneficial in treating house of the infectious host
gastroesophageal reflux. Diluting the formula would not (Answer: B) Rationale: Hepatitis A is highly contagious
be recommended because the infant needs the calories and is transmitted primarily through the fecal-oral route.
from the full-strength formula. It may be recommended The virus is transmitted by direct person-to-person
to thicken the formula with rice cereal. It is contact or through ingestion of contaminated food or
recommended to burp frequently; to delay burping would water, especially shellfish growing in contaminated
only increase the occurrences of reflux. water. The remaining answers are related to other
Gastroesophageal reflux is not related to milk infectious diseases.
intolerance so changing the formula would not help the
child. 130. Which of the following signs would the nurse
recognize as an indication of moderate dehydration in a
126. A 14-year-old boy is brought into the Emergency preschooler?
Department with a diagnosis of rule out appendicitis. He a. Sunken fontanel
is complaining of right lower quadrant pain. The nurse’s b. Diaphoresis
most appropriate action to assist in managing his pain c. Dry mucous membranes
would be to: d. Decreased urine specific gravity
a. Insert a rectal tube (Answer: C) Rationale: Mucous membranes typically
b. Apply an ice bag appear dry when moderate dehydration is observed.
c. Apply a heating pad Other typical findings associated with moderate
d. Administer an intravenous antispasmodic agent dehydration include restlessness with periods of
(Answer: B) Rationale: An ice bag may help relieve his irritability (especially infants and young children), rapid
pain. A rectal tube is contraindicated because it pulse, poor skin turgor, delayed capillary refill, and
stimulates bowel motility, which would increase the pain. decreased urine output. Both anterior and posterior
A heating pad is contraindicated because it increases fontanels are closed in a preschool-aged child. The skin
the flow of blood to the appendix and may lead to is usually dry with decreased elasticity, not diaphoretic.
rupture. An antispasmodic agent would not be beneficial Urine specific gravity increases with decreased urine
for the pain associated with appendicitis. Antispasmodic output associated with dehydration.
agents are typically used to inhibit smooth muscle
contractions. 131. The nurse has completed some child and family
education for a child diagnosed with thalassemia. The
127. The nurse has completed discharge teaching on medical plan of treatment includes blood transfusions
the dietary regimen of a child wit celiac disease. The when the anemia reaches a severe point. Which
nurse recognizes that client education has been statement by the parents indicates a need for further
successful when the mother states that the child must education?
comply with the gluten-free diet: a. “Because of the anemia, my child will need extra
a. Throughout life rest periods.”

St. Louis Review Center Inc. – BUTUAN CITY; Telephone No. (085) 342 – 2339 12
b. “My child inherited this disorder from both of us.” appropriate as iron is best absorbed on an empty
c. “We should be alert to periods when our child stomach.
seems paler than usual.”
d. “My child needs an iron supplement.” 136. The nurse is admitting a child newly diagnosed with
(Answer: D) Rationale: A child diagnosed with disseminated intravascular coagulopathy (DIC).
thalassemia who will receive multiple transfusions Although the physician has explained the plan of care to
throughout life will need chelation therapy for excessive the family, they continue to ask about each nursing
iron stores. An iron supplement would be inappropriate activity. The nurse notes that the family seems unable to
in this child. comprehend the answers. The nurse would:
a. Notify the doctor because the family seems to
132. The nursing assistant is setting up a hospital room have a comprehension problem
preparing to admit a child with disseminated b. Ask the doctor to write down the information for
intravascular coagulopathy. Which item would the nurse the family
remove from the set-up? c. Recognize that the family is under stress and
a. Rectal thermometer continue to answer their questions
b. Bedpan d. Determine if they are an English-as-a-second-
c. Intravenous therapy start kit language (ESL) family
d. Sphygmometer (Answer: C) Rationale: In an acute care setting such as
(Answer: A) Rationale: Rectal temperatures can a hospital and with a potentially life-threatening disease
traumatize the fragile rectal mucosa leading to bleeding such as DIC, the family may need help with coping with
and should be avoided. The vital signs will need to be the stress they are feeling. This stress often interferes
measured on a regular basis. An intravenous start kit is with communication. A patient response by the nurse
appropriate as the child will need plasma ands blood with repetition of information will allow the family to
products. A bedpan will be needed if the child is on bed absorb the information.
rest.
137. You are administering factor VIII to a child with
133. At a hemophilia camp, several children with injuries hemophilia. You should observe for which potential
arrive at the clinic at the same time. When prioritizing complication during the infusion?
care for the children, the child who requires the most a. Fluid overload
immediate care from the nurse is the child with: b. Transfusion reaction
a. A swollen knee c. Emboli formation
b. Abrasions on both arms d. Contracting AIDS
c. A slight head injury (Answer: B) Rationale: Factor VIII concentrate is a blood
d. A puncture wound in the foot product. Fluid volume overload is an unlikely concern, as
(Answer: C) Rationale: All of the injuries require nursing the factor will be given in a comparatively small volume
care; however, the child with the head injury has a of fluid. There is no greater a chance of emboli formation
potentially life-threatening injury and needs attention with administration of factor VIII than with any other IV
before the other three. preparation. Concern as to contracting AIDS from
administration of a blood product is a long-term concern
134. A 14-year-old boy with sickle cell anemia is related to multiple administrations. It is not a concern
admitted with severe pain in his abdomen and legs. He during the actual administration of the factor.
asks why the doctor ordered oxygen when he is not
having any problems breathing. The nurse would reply 138. As you make a plan of care for a child experiencing
that the main therapeutic benefit of oxygen is to: a sickle cell crisis, you should base your actions on the
a. Prevent further sickling knowledge that the pain of a vaso-occlusive crisis is
b. Prevent respiratory complications caused primarily by:
c. Increase the oxygen-carrying capacity of red a. Obstruction of small blood vessels
blood cells (RBCs) b. Sequestration of blood
(Answer: A) Rationale: RBCs sickle under conditions c. Hepatosplenomegaly
where low oxygen concentrations exist; therefore, d. Increased RBC destruction
administering oxygen will prevent additional sickling. The (Answer: A) Rationale: As RBCs clump together, they
oxygen has no effect on the oxygen-carrying capacity of block small blood vessels, which can lead to tissue
RBCs. It will not have an effect on development of ischemia, necrosis, and death. This is the chief cause of
respiratory complications. It will not decrease the pain during a vaso-occlusive crisis. Blood sequestration,
potential for infection. Hepatosplenomegaly, and increased RBC destruction
are not the primary causes of pain during a crisis.
135. You are administering a liquid iron preparation to a
3-year-old with iron deficiency anemia. It will be most 139. Which of the following statements should be
appropriate to: included when teaching the parents of a 7-month-old
a. Mix the medication in his milk and give it to him infant about preventing anemia?
at lunch a. Anemia for the duration of infancy is unusual as
b. Give the medication after lunch with a sweet infants use fetal iron stores until 18 months of
dessert to disguise the taste age
c. Give the medication to him in a small cup and b. Cow’s milk is an excellent source of iron, and
allow him to sip it through a straw infants should be changed from formula to milk
d. Allow him to decide whether to take the as soon as possible after 6 months of age
medicine with breakfast or dinner c. Milk is a poor source of iron, and infants should
(Answer: C) Rationale: Iron preparations should be be given solid foods high in iron such as cereals,
taken through a straw in order to prevent staining the vegetables, and meats
teeth. While it is generally best to give toddlers choices d. Anemia can easily occur during infancy and all
in the hospital setting, the other options are not infants should receive iron supplements

St. Louis Review Center Inc. – BUTUAN CITY; Telephone No. (085) 342 – 2339 13
(Answer: C) Rationale: Anemia does occur easily infancy d. Assign the child to attend a session of health
and infants have limited stores of iron. The first solid class that deals with smoking
food offered to infants is often cereal, which is an (Answer: A) Rationale: Options 2, 3, and 4 are
excellent source if iron. All infants do not require iron appropriate ongoing activities to promote substance
supplements; it is preferable that the iron comes form abuse prevention; however, peer-led programs have
dietary intake. proven to be the most successful when teaching children
about the hazards of substance and tobacco use and
140. What precautions should the nurse take when abuse.
discontinuing an IV for a child with alterations in platelet
function? 144. You are providing care to a toddler who has
a. Restrict movement of the arm for 12 hours ingested an unknown amount of his grandfather’s
b. Obtain a culture of the tip of the IV catheter medication, which is described as “a white pill.” The
c. Place steri-strips over site and have child hold physician has ordered the administration of syrup of
the arm above heart level for 15 minutes ipecac and activated charcoal. What action would you
d. Apply direct pressure to the site for at least 5 take?
minutes a. Question the order because syrup of ipecac and
(Answer: D) Rationale: Alterations in platelet function activated charcoal are not to be used together
necessitate treating a break in the skin’s integrity as you b. Administer the ipecac, and after the child vomits,
would an arterial stick – apply pressure for 5 minutes or administer the activated charcoal
more. The goal of treatment is to apply pressure long c. Administer the activated charcoal, then
enough that the defective clotting mechanism will have administer the syrup of ipecac
time to form a clot. Steri-strips would close the wound d. Insert a nasogastric tube for the administration
adequately, and restricting arm movement will not assist of the syrup of ipecac and activated charcoal
in the initial formation of a clot. (Answer: B) Rationale: Option 1 is incorrect as ipecac is
often given preceding the administration of activated
141. After writing a suicide note, a 16-year-old swallows charcoal. To reduce the risk of aspiration, activated
numerous anti-anxiety pills belonging to a friend. Which charcoal should never be administered before syrup of
of the following factors would indicate to you that the ipecac (option 3), and there is no indication to insert a
teenager is at risk for a repeated attempt at suicide? nasogastric tube. Administering the syrup of ipecac,
a. She stated that she wishes she hadn’t made waiting for the child to vomit, then administering the
such a “stupid mistake.” activated charcoal is the correct method.
b. Her grades have dropped over the past few
weeks 145. A child is brought to the Emergency Department
c. Her father died recently with excessive drooling, edema of lips and tongue,
d. She lives with her mother and stepfather swollen mucous membranes, and is hypotensive and
(Answer: C) Rationale: Parental loss is a risk factor tachycardic. Based on this initial assessment, you
associated with suicide. A decline in grades is a suspect that the child has ingested which of the following
symptom exhibited before her first suicide attempt, and agents?
without further investigation, there is no indication that a. A corrosive agent
there is a dysfunctional relationship between her and the b. Aspirin
mother or stepfather. Indicating remorse for the action is c. Hydrocarbons
a positive step towards recovering. d. Acetaminophen
(Answer: A) Rationale: Corrosive agents cause the signs
142. When performing a health screening on an and symptoms listed. Indications of aspirin overdose are
adolescent in the health clinic, you would determine he nausea, vomiting, diaphoresis, and seizures.
is at a higher risk of suicide than other adolescents of his Hydrocarbons cause nausea, vomiting, cyanosis, and
age based on which of the following facts that he altered sensorium, ands acetaminophen causes nausea,
discloses? vomiting, diaphoresis, and later, jaundice.
a. He states that hw sleeps late on the weekends
b. He states that he only has a small group of close 146. You are the telephone triage nurse and have
friends received a call from a mother who states that her 4-year-
c. He states that he is a homosexual old son has ingested an unknown amount of aspirin. She
d. He states that he often skips meals and does administered 15mL of syrup of ipecac 20 minutes ago,
not worry about nutrition but the child has not vomited yet. You instruct the
(Answer: C) Rationale: Homosexual adolescents are at mother to take which of the following actions?
an extremely higher risk of suicide than other a. Wait 15 more minutes, then if the child still has
adolescents their age, especially if the family does not not vomited, take him to the Emergency
offer support. Sleeping late on weekends and skipping Department immediately
meals without concern for nutrition is normal for b. Take child to the Emergency Department
adolescents, as is having a small group of close friends. immediately
c. Repeat the 15mL of syrup of ipecac now, and
143. An 11-year-old female was discovered smoking immediately take the child to the Emergency
cigarettes in the school bathroom. The school nurse Department
should implement which of the following plans for this d. Repeat the syrup of ipecac, but increase the
school-aged children? dose to 30mL, and immediately take the child to
a. Assign the child to a peer-led program to teach the Emergency Department
the consequences of smoking (Answer: C) Rationale: 15mL of syrup of ipecac can be
b. Recommend that the child attend a community- repeated once for a 4-year-old child who has not
based smoking prevention program vomited after 20 minutes. All children with accidental
c. Assign the child videos to view that demonstrate poisonings should be taken to the Emergency
the effects of smoking Department for evaluation and treatment even if vomiting

St. Louis Review Center Inc. – BUTUAN CITY; Telephone No. (085) 342 – 2339 14
has occurred. Radiologic examination reveals fractured ribs and a
healed fractured humerus. Based on these findings, your
147. A 10-year-old child with mild mental retardation next course of action would be which of the following?
wants to join his younger brother’s Cub Scout group. His a. Report the child as a victim of child abuse
parents are apprehensive about allowing him to join and immediately
ask you for advice. Your response will be based on the b. Ask the father to provide further details of the
fact that children with mental retardation: incident, obtain a medical history of the child,
a. Have the same need for socialization as children and then interview the child separately
without mental retardation c. Ask the father if he has been physically abusive
b. Should not be encouraged to participate in clubs to the child
because of their developmental delay d. Ask the father if he believes the child’s mother
c. Should participate in clubs especially created for has been physically abusive to the child
children that are cognitively impaired (Answer: B) Rationale: It is important to establish a
d. Do not have a need for socialization thorough history and a detail of the incident before
(Answer: A) Rationale: Children with mental retardation making assumptions of abuse. The child is safe from
have the same need for socialization as others and harm in the Emergency Department, allowing time to
should be encouraged to participate in clubs and adequately assess the situation. You should not make
activities with children of the same developmental age. premature assumptions (option 1), and if abuse is
There is no need to encourage participation only in clubs suspected, you should not inform the parent (option 3
exclusive to children with cognitive impairment; this and 4).
would limit the child’s social interaction.

148. An 11-year-old child with attention deficit


hyperactivity disorder (ADHD) being treated with
methylphenidate (Ritalin) twice a day reports that he is
having difficulty falling to sleep at night. Upon
questioning him, you discover he is taking the
medication in the morning before leaving for school and
in the evening after supper. Based on the information
provided, you would instruct him in which of the
following?
a. Continue taking the morning dose as previously,
but take the evening dose earlier in the
afternoon
b. Stop taking the medication until he can be
evaluated by his physician
c. Take both doses of the medication in the
morning before leaving for school
d. Reduce the evening does of medication t half
the prescribed dose
(Answer: A) Rationale: Ritalin is a central nervous
system (CNS) stimulant, and if taken in the later
evening, may cause insomnia. The medication should
not be discontinued unless ordered by a physician, nor
should the dosages be adjusted without the physician’s
instructions.

149. The parents who have just experienced the death


of an infant from sudden infant death syndrome (SIDS)
request time alone with the infant. You should take
which of the following actions?
a. Discourage the parents from seeing the infant
because it will be too painful PRETEST:
b. Allow the parents as much time alone with the 01. Play during infancy is:
infant as they need a. Initiated by the child
c. Allow the parents to view the infant, but remain b. A way of teaching how to share
in the room with them c. More important than in later years
d. Deny the parents’ request because they are d. Mostly used for physical development
emotionally distraught
(Answer: B) Rationale: Parents need the opportunity to 02. The primary task to be accomplished between 12
hold their infant and to say goodbye in private for as long and 15 months of age is to learn to:
as they need. A peaceful, quiet, supportive environment a. Use a spoon
should be provided. Options 1, 3, and 4 are incorrect, as b. Climb stairs
they do not demonstrate compassionate care for parents c. Walk
who have just experienced the death of a child. d. Say simple words

150. A 3-year-old child is brought to the Emergency 03. Preschool children role-play. This is an important
Department for treatment of injuries the father stated part of socialization, since it:
were obtained when the child fell off of his tricycle. Upon a. Encourages expression
assessment, numerous bruised areas, old and fresh, are b. Helps children think about careers
noted on the child’s back, buttocks, and shoulders. c. Teaches children about stereotypes

St. Louis Review Center Inc. – BUTUAN CITY; Telephone No. (085) 342 – 2339 15
d. Provides guidelines for adult behavior b. Be unlikely to need this protection during their
shortened life span
04. Learning processes associated with a particular c. Be allergic to rabbit serum, which is used as a
stage of development often are referred to as basis for these vaccines
“developmental tasks.” A characteristic of developmental d. Be susceptible to infection because of their
tasks is that: depressed immune response
a. There is no uniform time of learning a task
b. Tasks are learned at the same age in children 12. An injection consisting of bacterial cells that have
c. Tasks occur with predictable rhythm been modified is:
d. Most development tasks are learned by school a. A vaccine
age b. An antitoxin
c. A toxoid
05. Parents can predispose their children to problems d. A toxin
with nutrition by using food in early childhood as a
means of: 13. A child comes to the hospital after exposure to
a. Socializing diphtheria and is given antitoxin. This type of immunity is
b. Reward and punishment known as:
c. Acculturation a. Active natural immunity
d. Teaching discipline b. Active artificial immunity
c. Passive natural immunity
06. The major influence on eating habits of the early d. Passive artificial immunity
Schoolage child is the:
a. Example of parents at mealtime 14. Immunity by antibody formation during the course of
b. Food preferences of the peer group a disease is:
c. Availability of food selections a. Active natural immunity
d. Smell and appearance of food b. Active artificial immunity
c. Passive natural immunity
07. Selection of drugs of choice for the treatment of d. Passive artificial immunity
pneumonia depends primarily on:
a. Selectivity of the organism 15. A viral disease that begins with respiratory
b. Tolerance of the client inflammation and skin rash and may result in grave
c. Preference of the physician complications is:
d. Sensitivity of the organism a. Rubeola
b. Rubella
08. Mrs. Legere and her son Johnny are seen at the c. Yellow fever
clinic. They both have severe upper respiratory tract d. Chickenpox
infections, and the physician plans to prescribe
tetracycline (Achromycin). The nurse reminds him that 16. Mary has received her primary immunizations, so her
Johnny is 6 years old and that Mrs. Legere is in her mother asks the nurse which ones should receive prior
eighteenth week of pregnancy. The data are important to starting kindergarten. The nurse suggests the
because the drug may cause: following booster doses:
a. Persistent vomiting when given to small children a. DTP, OPV
and pregnant women b. Measles, DTP
b. Tooth enamel defects in children under 8 years c. OPV, rubella
of age and in the maturing fetus d. DTP, tuberculin test
c. Lower red blood cell production at times in their
development when anemia is a common 17. Occasionally infants are born without an immune
problem system. They can live normally with no apparent
d. Changes in the bone structure of young children problems during their first months after birth because:
and pregnant women a. Limited antibodies are produced by the fetal
thymus during the eighth and ninth months of
09. A viral infection characterized by red blotchy rash gestation
and Koplik’s spots in the mouth is: b. Antibodies are passively received from the
a. Rubeola mother through the placenta and milk
b. Rubella c. Limited antibodies are produced by the infant’s
c. Chickpox colonic bacteria
d. Mumps d. Exposure to pathogens during this time can be
limited
10. Under certain circumstances the virus that causes
chickenpox can also cause: 18. James, a 2-year-old child, is admitted to the hospital
a. Athlete’s foot with a diagnosis of pneumonia. He is given antibiotics,
b. Infectious hepatitis forced fluids, and oxygen. James’s temperature
c. Herpes zoster continues to rise until it reaches 103F (39.4C). The
d. German measles nurse calls the physician at the mother’s request, but the
physician sees no cause for alarm or change in
11. Using live virus vaccines against measles is treatment, even though James has a history of
contraindicated in children receiving corticosteroids or convulsions during previous periods of high fever.
antineoplastic or irradiation therapy because these Although the nurse is concerned, she takes no further
children may: action. Later James has a convulsion that results in
a. Have had the disease or have been immunized neurologic impairment of the left arm and leg. Legally:
previously

St. Louis Review Center Inc. – BUTUAN CITY; Telephone No. (085) 342 – 2339 16
a. The nurse’s actions did not derive from b. Grunting and rapid respirations
observations, client’s history, or scientific fat c. Profuse perspiration
b. The physician’s decision takes precedence over d. Desire for increased fluids during the feedings
the nurse’s concern
c. High temperatures are common in children, and 27. Among the last signs of heart failure in the infant and
this situation presented little cause for undue child is:
concern a. Rapid respiratory rate in the supine position
d. The physician is totally responsible for the b. Orthopnea
client’s health history and treatment regimen c. Tachypnea
d. Peripheral edema
19. Four-year-old Bobby has a seizure disorder and has
been taking phenytoin (Dilantin) for 3 years. An 28. A newborn of a few hours appears to be les cyanotic
important nursing measure for Bobby would be to: when he cries. The nurse should observe for:
a. Offer the urinal frequently a. Twitching of the body resulting form neural
b. Administer scrupulous oral hygiene damage
c. Check for pupillary reaction b. Unequality of chest expansion associated with
d. Observe for flushing of the face atelectasis
c. Alterations in heart rate associated with an
20. When teaching parents at the school about atrioventricular septal defect
communicable disease, the nurse reminds them that
these diseases are serious, and that encephalitis can be 29. A mother brings her week-old newborn to the clinic
a complication of: because he continually regurgitates. Chalasia is
a. Chickenpox suspected. The nurse instructs the mother to:
b. Pertussis a. Keep the infant prone following feedings
c. Poliomyelitis b. Not permit the infant to cry for prolonged periods
d. Scarlet fever c. Keep the infant in a semisitting position,
particularly after feedings
21. A viral disease caused by one of the smallest human d. Administer a minimum of 8 oz of formula at each
viruses that infect the motor cells of the anterior horn of feeding
the spinal cord is:
a. Rubeola 30. The best legal definition of assault is:
b. Rubella a. The application of force to another person
c. Poliomyelitis without lawful justification
d. Chickenpox b. Threats to do bodily harm to the person of
another person
22. A Streptococcus infection characterized by swollen c. A legal wrong committed by one person against
joints, fever, and the possibility of endocarditis and death the properly of another
is: d. A legal wrong committed against the public and
a. Whooping cough punishable by law through the state and courts
b. Measles
c. Tetanus 31. In legal terminologies, the term battery means:
d. Rheumatic fever a. Doing something that a reasonable person with
the same education or preparation would not do
23. A skin infection that can be a sequela of a b. A legal wrong committed by one person against
staphylococcal infection or glomerulonephritis is: the property of another
a. Herpes simplex c. The application of force to the person of another
b. Scabies person without lawful justification
c. Intertrigo d. Maligning the character of an individual while
d. Impetigo threatening to do bodily harm

24. A small toddler is admitted to the hospital because of 32. Dietary treatment of PKU includes a:
sudden hoarseness and an insistence on continuous a. Low-phenylalanine diet
and somewhat unintelligible speech. In talking with the b. Phenylalanine-free diet
mother, the nurse will be particularly concerned about: c. Dietary supplement for phenylalanine
a. Acute respiratory tract infection d. Protein-free diet
b. Undetected laryngeal abnormality
c. Respiratory tract obstruction due to a foreign 33. Alan, age 3 months, has been diagnosed as having
body cretinism. If care is not instituted until after early infancy,
d. Retropharyngeal abscess he will probably:
a. Have myxedema
25. An infection caused by the yeast Candida albicans b. Be somewhat mentally retarded
often occurring in infants and debilitated individuals is: c. Have abnormal deep tendon reflexes
a. Typhoid fever d. Have thyrotoxicosis
b. Thrush 34. Three-month-old Lisa is diagnosed as having
c. Malta fever cretinism. She is to receive thyroxine sodium, 0.35 mg,
d. Dysentery QD-PO. The medication is available in elixir form, 0.25
mg/mL. The nurse should administer:
26. A mother talks to the nurse about her sick infant and a. 0.8 mL
she is disturbed because she did not realize the baby b. 1.4 mL
was ill. A major indication of illness in an infant is: c. 0.6 mL
a. Longer periods of sleep d. 1.0 mL

St. Louis Review Center Inc. – BUTUAN CITY; Telephone No. (085) 342 – 2339 17
d. Generalized physical symptoms and behavior
35. Three-year-old Karen Allen may have celiac disease. problems may precede drug-induced lvier
One symptom common in children with celiac disease is damage
stool that is:
a. Large, frothy, dark green 42. Fourteen-year-old Evelyn is severely hurt while on a
b. Small, pale, mucoid skateboard and develops muscle contractures in all her
c. Large, pale, foul smelling limbs. She refuses to move, so the nurse should
d. Moderate, green, foul smelling encourage her by:
a. Explaining that some pain is inevitable
36. Mrs. Joyce asks the nurse how to tell the difference b. Allowing friends to visit every day
between measles (rubeola) and German measles c. Permitting her to make decisions regarding her
(rubella). The nurse tells Mrs. Joyce with rubeola the care
child has: d. Setting strict limits to increase her security
a. A high fever and Koplik’s spots
b. Symptoms similar to a cold, followed by a rash 43. Nine-year-old Harold has a fractured femur and a full
c. Nausea, vomiting, and abdominal cramps leg cast has just been applied. Which of the following
d. A rash on the truck with pruritus observations made by the nurse should be reported to
the physician immediately?
37. A physician orders a tap water enema for 6-month- a. Pedal pulse of 90
old Bart. The nurse considers that a tap water enema b. Cast still damp and warm after 4 hours
could: c. Inability to move the toes
a. Cause a fluid and electrolyte imbalance d. Increased urinary output
b. Increase his fear of intrusive procedures
c. Result in shock form a sudden drop in 44. Two-year-old Jimmy swallowed kerosene from a
temperature soda bottle stored in the garage. Immediate treatment
d. Results in loss of necessary nutrients for ingestion of petroleum distillates is to have the child
swallow:
38. Chickenpox can sometimes be fatal to children who a. Milk of magnesia
are receiving: b. Strong tea
a. Antibiotics c. Weak salt solution
b. Steroids d. Mineral oil
c. Anticonvulsants
d. Insulin 45. A toddler has swallowed liquid drain cleaner
containing lye. The immediate intervention is to
39. The nurse explains to the parent group that the most administer:
important complication of mumps in postpubertal males a. Syrup of ipecac
is: b. Two ounces of milk
a. Decrease in libido c. Dilute vinegar solution
b. Hypopituitarism d. Sodium bicarbonate and water
c. Sterility
d. A decrease in androgens 46. Susan is found by her mother playing with an open
bottle of diuretic tablets. The physician tells Susan’s
40. Paula, a 3-year-old with eczema of the face and mother to give syrup of ipecac to Susan. The effect of
arms, has not heeded the nurse’s warnings to “stop the drug will be enhanced by:
scratching – or else!” The nurse finds Paula scratching a. Resting until vomiting occurs
so intensely that her arms are bleeding. With great flurry, b. Drinking 2 to 3 glasses of water
the nurse ties Paula’s arms to the crib sides saying, “I’m c. Actively playing until vomiting occurs
going to teach you one way or another.” In this situation, d. Stimulating the gag reflex
the nurse:
a. Had to protect Paula’s skin and acted as any 47. The primary reason for using prednisone in the
reasonably prudent nurse would do treatment of acute leukemia in children is that it is able
b. Tried to explain to Paula and rightly expected to:
her to understand and cooperate a. Suppress mitosis in lymphocytes
c. Has used actions that can be interpreted as b. Reduce irradiation edema
assault and battery c. Decrease inflammation
d. Has merely done her job with considerable d. Increase appetite and sense of well-being
accountability
48. A combination of drugs, which includes vincristine
41. Nancy, age 8, is receiving tetracycline (Achromycin). (Oncovin) and prednisone, is prescribed for a child with
Her fever is down and secretions have lessened; but she leukemia. Because of their toxicity the nurse should
is eating poorly, s withdrawn, lethargic, and irritable, and expect:
sobs readily. The nurse should promptly discuss the a. Neurologic symptoms
problem with the physician because: b. Irreversible alopecia
a. She needs a higher food intake to fight the c. Anemia and fever
infection d. Gastrointestinal symptoms
b. Anemia is a frequent occurrence after infection
and treatment with antibiotics 49. Which of the following responses is unusual in
c. Concurrent bladder infection may be present as infants subjected to prolonged hospitalization?
an extension of her gram-negative infection a. Lack or slowness of weight gain
b. Looking at ceiling lights rather than at persons
caring for them

St. Louis Review Center Inc. – BUTUAN CITY; Telephone No. (085) 342 – 2339 18
c. Limited emotional response to stimuli d. Increasing use of ventilating systems
d. Excessive crying and clinging when approached
58. Elouise, 8 months old, has a gastrostomy tube and is
50. A characteristic of infants and young children who given 240mL of tube feeding q2h. One of the primary
have experienced maternal deprivation is: nursing responsibilities at the time of the feeding is to:
a. Extreme activity a. Elevate the tube 30cm (12 inches) above the
b. Proneness to illness mattress
c. Responsiveness to stimuli b. Give 10mL of normal saline before and after
d. Tendency toward overeating feeding
c. Position on the right side after feeding
51. Naomi, 9 years old, is about to have surgery. The d. Open the tube 1 hour before feeding
physician orders meperidine (Demerol), 20mg, IM
preoperatively. The container reads “50mg/ml.” The 59. Prior to administering a gastrostomy tube feeding to
nurse should administer: an infant, the nurse should:
a. 0.6 ml a. Slowly instill 5mL of water
b. 0.4 ml b. Aspirate the tube
c. 0.8 ml c. Provide the baby with a pacifier
d. 1.0 ml d. Place in a semi-Fowler’s position

52. An infant scheduled for surgery is diagnosed as 60. Sal has been admitted to the hospital for surgery to
having a diaphragmatic hernia. Measures that the nurse correct his congenital megacolon. Enemas are ordered
would expect to be employed at this time include: preoperatively to cleanse the bowel. The nurse should
a. Positive pressure oxygen by mask use:
b. Positive pressure oxygen by endotracheal tube a. Soap suds
c. Increased oxygen concentration by any method b. Hypertonic phosphate
d. Humidity of 40% c. Isotonic saline
d. Tap water
53. It is expected that, after some surgical intervention
for atelectasis, lung expansion will recur within: 61. If monocular strabismus is not corrected early
a. 1 hour enough:
b. 48 to 72 hours a. Vision in both eyes will be diminished
c. 4 hours b. Peripheral vision will disappear
d. 12 to 48 hours c. Dyslexia will develop
d. Amblyopia develops in the weak eye
54. Dina, 18 months old, is to receive 5% dextrose and
Ringer’s lactate, 1000mL IV, in 24 hours. The drop factor Situation: Mr. Crew, a nursing student, is doing
of the minidropper is 60 drops/ml. The nurse should therapeutic play wit the children in the playroom of the
regulate the IV to run at: well-baby clinic. To understand how to plan for children
a. 34 drops per minute of various ages, he needs to have knowledge of their
b. 38 drops per minute developmental norms. Questions 62 through 65 refer to
c. 42 drops per minute this situation.
d. 21 drops per minute 62. Mr. Crew observes that 2-year-old Mark:
a. Builds houses with blocks
55. When picked up by either the mother or the nurse, b. Is very possessive of toys
an 8-month-old infant screams. The scream seems to be c. Attempts to stay within the lines when coloring
that of pain. At his clinic visit the nurse will note and talk d. Amuses himself with a picture book for 15
particularly to the mother about: minutes
a. The infant’s food and specific vitamins given to
him, including vitamins given to him, including 63. Mr. Crew observes 4-year-old Colin having difficulty
vitamins C and D playing with the other children. He understands that it is
b. Accidents and injuries and the importance of normal for Colin to:
their prevention a. Exaggerate and boast to impress others
c. Any other behavior of the infant that may have b. Have pierce temper tantrums and negativism
been noticed by the mother c. Engage in parallel or solitary play
d. Limiting the play time and activities that this d. Be almost totally dependent on parents
infant has with other children in the family
64. Fifteen-month-old Nadia is playing in the playpen.
56. Eleven-year-old Harry has gained weight. His mother Mr. Crew observes her activities and realizes that her
is concerned that Harry, who loves sports, may become physical tasks are within the norms when she is able to:
obese. The nurse: a. Build a tower of six blocks
a. Urges a decreased caloric intake b. Stand in the playpen holding onto the sides
b. Explains this is normal for a preadolescent c. Throw all the toys out of the playpen
c. Advises an increase in activity d. Walk across the playpen with ease
d. Discusses the relationship of genetics and
weight gain 65. Mr. Crew would encourage two 6-year-old boys in
the playroom to play with:
57. Exposure to hepatitis B may be expected to occur in a. A board game
hospitals because of: b. An erector set
a. Needle sticks and mucous membrane exposure c. Checkers
b. Careless handling of excreta by staff d. Clay
c. Newer treatments for hemophilia A

St. Louis Review Center Inc. – BUTUAN CITY; Telephone No. (085) 342 – 2339 19
Situation: Mr. And Mrs. Bee were emotionally upset c. Blood disorders
when their baby girl Sue was born with a cleft palate and d. Introduction of solid foods too early for proper
double cleft lip. Questions 66 through 70 refer to this absorption
situation.
66. To give the most support to the parents, the nurse 74. Anemia, a nutritional problem encountered in
should: children and adults, involves several different nutrients.
a. Discourage them form talking about the baby The nutrients include proteins, iron, vitamin B12, and:
b. Encourage them to express their worries and a. Carbohydrates
fears b. Thiamine
c. Tell them not to worry because the defect can c. Calcium
be repaired d. Folic acid
d. Show them postoperative photographs of babies
who had similar defects 75. The food that the nurse would emphasize to David’s
mother as a source of iron to be included in his diet daily
67. The most critical factor in the immediate care of Sue is:
after repair of the lip is: a. Orange juice
a. Maintenance of a patent airway b. Lamb
b. Administration of drugs to reduce oral secretions c. Mineral-fortified cereal
c. Administration of parenteral fluids d. Milk
d. Prevention of vomiting
76. David’s mother also states that he has eight teeth
68. Additional nursing care for Sue after the surgical lip and asks when she should take him to the dentist. For
repair would include: dental prophylaxis, the nurse encourages her to take
a. Placing the infant in a semisitting position him:
b. Keeping the infant from crying a. The next time another family member goes to
c. Spoon feeding for the first 2 days after surgery the dentist
d. Keeping the baby NPO b. Before starting school
c. Between 2 and3 years of age
69. At 2 years of age Sue returned for palate surgery. d. When he begins to lose his deciduous teeth
The most important factor in preparing her for this
experience is: 77. The nurse’s background knowledge of the basic
a. Her previous hospital visits nutrients that act as partners in building red blood cells
b. Gratification of all her wishes will form the basis for a teaching plan. These nutrient
c. Never leaving her with strangers partners of iron are:
d. Assurance of affection and security a. Calcium and vitamins
b. Carbohydrates and thiamine
70. A toothbrush would not be used on Sue immediately c. Proteins and ascorbic acid
after palate surgery because: d. Vitamin D and riboflavin
a. The suture line might be injured
b. She was not accustomed to a brush at home Situation: Four-year-old Ann weighs 18 kg (40 pounds)
c. She probably has no teeth and is in a private pediatric room on “gown, glove, and
d. It might be frightening to her linen precautions.” She was admitted for weight loss,
anorexia, viginitis, and insomnia. A diagnosis of pinworm
Situation: David, age 1 year, weighs 12.6 kg (28 infestation was made. Questions 78 through 82 refer to
pounds) but is pale and lethargic. His hemoglobin level this situation:
is 5 g and he has an enlarged heart. When taking a 78. The most effective time for the nurse to collect a
nursing history form his mother, the nurse learns that he cellophane tape test for p8inworms is:
refuses food so she gives him a quart of milk per day a. At bedtime before bathing
from a bottle. Questions 71 through 77 refer to this b. Just following a BM
situation. c. Immediately after meals
71. The nurse suggests that his mother: d. Early morning before arising
a. Put a large hole in the nipple and put baby food
in with his milk 79. Pinworms cause a number of symptoms besides
b. Take him to the metabolic clinic for a checkup anal-itching. A rare sequela of pinworm infestation that
c. Immediately begin the weaning process the nurse should observe for is:
d. Give him finger foods such as raisins and a. Pneumonitis
chopped meat b. Stomatitis
c. Hepatitis
72. David should have been started on solid foods by at d. Appendicitis
least 5 or 6 months of age because:
a. His fetal reserve of iron was depleted 80. Pyrvinium pamoate (Povan) is an effective single-
b. It would have taught him how to chew dose drug to eliminate pinworms. How many milligrams
c. His bone marrow activity has slacked off at this will Ann receive if 5 mg per kilogram of body weight are
time ordered?
d. It would have helped control his weight a. 90 mg
b. 18 mg
73. The most prevalent nutritional disorder among c. 40 mg
children in the United States is iron deficiency anemia. A d. 200 mg
major reason for this in young children is:
a. Overfeeding of milk
b. Lack of adequate iron reserves from the mother

St. Louis Review Center Inc. – BUTUAN CITY; Telephone No. (085) 342 – 2339 20
81. After administering Pyrvinium pamoate (Povan) to 89. The nurse explains to Mrs. Kee that a very important
Ann, it is important to alert the staff that a normal side aspect of care for John at home should include:
effect of this drug is that it colors the stool or vomitus: a. At least 14 hours sleep per day
a. Dark brown b. Rigorous exercise and play
b. Light green c. Ingestion of large quantities of liquids
c. Bright red d. High-calorie diet
d. Gentian blue
90. Infants with sickle cell anemia may not be diagnosed
82. The nurse’s decision to alert the staff is based on the as having this disorder because of:
knowledge that: a. The presence of fetal hemoglobin during the first
a. Irritation by pinworms in the rectum may cause year of life
ulceration and bleeding b. Compensation of increased hematocrit and
b. The cyanine dye origin of the drug colors the hemoglobin if well fed
stool c. Absence of respiratory disorders
c. The stool contains hemoglobinlike metabolic d. General good health and an excellent growth
products of disintegrating pinworms curve
d. The drug is irritating to the intestinal mucosa and
may cause transient bleeding 91. The sickling process of the red blood cell occurs in
conditions of:
Situation: Seven-year-old Johnny has been admitted for a. Hemodilution
a tonsillectomy. Questions 83 through 85 refer to this b. Hypoxia
situation: c. Thrombocytopenia
83. An essential nursing action preoperatively is to: d. Hypocalcemia
a. Encourage parent to stay until Johnny goes to
the operating room 92. To prevent thrombus formation in capillaries, as well
b. Provide him with his favorite toy as other problems form stasis and clotting of blood in the
c. Observe his ASO titer sickling process, the main nursing intervention is:
a. Administration of oxygen
84. The nurse suspects hemorrhage postoperatively b. Increasing fluids by mouth and a humidifier
when Johnny: c. Complete bed rest
a. Snores noisily d. Use of heparin or other anticoagulants
b. Becomes pale
c. Complains of thirst 93. Common nursing care that helps prevent both sickle
d. Swallows frequently cell crisis and celiac crisis is:
a. Limitation of activity
85. Johnny is complaining of pain in his throat. Which of b. Protection from infection
the following medications for pain would be best for him c. High-iron, low-fat, high-protein diet
at this time? d. Careful observation of all vital signs
a. Aspirin, 300mg
b. Tylenol, 300mg Situation: Johnny Smith, 12 months of age, is brought
c. Phenobarbital, 15mg to the pediatric health clinic for a regular physical
d. Demerol, 50mg assessment. Questions 94 through 98 refer to this
situation.
Situation: Two-day-old Edward has a 94. In reviewing his immunizations for the past 10
meningomyelocele. He is scheduled for surgery. months the nurse would expect him to have been
Questions 86 through 88 refer to this situation: immunized against:
86. Prior to the surgical correction, a primary nursing a. Measles, rubella, polio, tuberculosis, and
goal in caring for a child with a meningomyelocele is to: pertusis
a. Observe for bowel and bladder dysfunction b. Polio, Pertussis, tetanus, and diphtheria
b. Prevent infection c. Measles, mumps, rubella, and tuberculosis
c. Prevent skin breakdown d. Pertussis, tetanus, polio, and measles
d. Observe for increasing paralysis
95. Mrs. Smith asks the nurse how the DTP injection
87. After closure of Edward’s meningomyelocele, it is works. The nurse, in formulating a response, recalls that
essential that his nursing care include: in active immunity:
a. Decrease of environmental stimuli a. Blood antigens are aided by phagocytes in
b. Strict limitation of leg movement defending the body against pathogens
c. Measurement of head circumference daily b. Protein antigens are formed in the blood to fight
d. Observation of serous drainage from the nares invading antibodies
c. Protein substances are formed by the body to
88. To meet a major developmental need of Edward’s destroy or neutralize antigens
the nurse should: d. Lipid agents are formed by the body against
a. Provide a soft cuddly toy antigens
b. Give him a pacifier
c. Warm his formula before feeding 96. The measles immunization is usually routinely given
d. Put a mobile over his crib after 15 months of age because of the:
a. Increased hazard of side effects in infants
Situation: Eight-year-old John Kee is being discharged b. Presence of maternal antibodies during the first
following treatment for sickle cell crisis. He is allowed to year
return to school and resume normal activities. Questions c. Interference it causes with effectiveness of
89 through 93 refer to this situation. pertussis, diphtheria, and tetanus immunizations

St. Louis Review Center Inc. – BUTUAN CITY; Telephone No. (085) 342 – 2339 21
d. Rare incidence of measles infection prior to 12 delivery
months of age c. If PKU is untreated, mental retardation occurs
d. Treatment for PKU includes life-long diet therapy
97. Infants receive immunizations made up of attenuated
viruses. This means that these immunizations: 106. A test that was done on Baby Bronson in the
a. Contain passive antibodies nursery to detect PKU is:
b. Contain active antibodies a. Guthrie blood test
c. Cause the development of passive antibodies b. Ferric chloride urine test
d. Cause the development of active antibodies c. Phenistix test
d. Clinitest serum phosphopyruvic acid
98. In terms of preventive teaching for a 1-year-old, the
nurse would speak to Mrs. Smith about: 107. When teaching Mr. and Mrs. Bronson about their
a. Adequate nutrition daughter’s disorder, the nurse should state that:
b. Accidents a. Phenylalanine is not necessary for growth
c. Sexual development b. Other amino acids can be increased to
d. Toilet training substitute for phenylalanine
c. A low-phenylalanine diet is required
Situation: Nancy Hand, a 5-year-old only child, is d. Phenylalanine can be administered to correct
admitted to the hospital with pneumonia. She requires the deficiency
bed rest, a soft diet, liberal fluid intake, and ampicillin,
250 mg, po qid. She is restless and fretful and tells the 108. In terms of dietary counseling, the parents need
nurse she feels sick. Questions 99 through 104 refer to much help and support in adhering to specific regimens.
this situation. A frequent question asked by parents is, “How long will
99. The immediate priority in Nancy’s nursing care is: my child have to be on this diet?” An appropriate
a. Nutrition response by the nurse is:
b. Rest a. “Unfortunately, this is a life-long problem and
c. Exercise dietary management must always be
d. Elimination maintained.”
b. “Usually, if the child does well for 1 year, she
100. Nursing care likely to be most effective in alleviating then can gradually begin eating regular foods.”
Nancy’s fretfulness is: c. “As of now, research shows that a child needs to
a. Giving her a jigsaw puzzle be on this diet until she is about 6 to 8 years of
b. Putting her in a room by herself age. Then she can gradually begin to eat other
c. Letting her play with a doll foods.”
d. Reading a story to her d. “No one knows, but why don’t you discuss it with
your doctor.”
101. The best choice of between-meal nourishment for
Nancy is: Situation: Mrs. Simmons brings 3-year-old Sam to the
a. Fresh fruit emergency room, indicating he has had a fever for
b. Hard candy several days, has held his neck rigid, and is now
c. Skim milk vomiting. While being examined he has a convulsion and
d. Creamed soup is admitted to the pediatric unit. Questions 109 through
114 refer to this situation.
102. When the nurse brings her dinner tray Nancy says, 109. While instituting nursing measures to reduce Sam’s
“I’m too sick to feed myself.” The nurse should respond: fever, the nurse recognizes that an important
a. “Try to eat as much as you can.” consideration is to:
b. “Be a big girl and don’t act like a baby.” a. Monitor vital signs every 10 minutes
c. “Let it go until you feel better.” b. Force oral fluids
d. “Wait 5 minutes and I will help you.” c. Measure output every hour
d. Limit exposure to prevent shivering
103. Nancy’s statement is most likely indicative of:
a. Immaturity 110. One morning, while Sam is in crib, the nurse notes
b. Loneliness his jaws are clamped and he is having a seizure. The
c. Regression most important nursing responsibility at this time is to:
d. Temper tantrum a. Insert a padded tongue blade
b. Start oxygen at 10L by mask
104. Nancy is apathetic about eating. Nursing care c. Protect Sam from harm from the environment
directed toward supporting her nutrition should include: d. Restrain Sam to prevent injury to soft tissue
a. Providing Diversional activity at mealtime
b. Eliminating all between-meal nourishment 111. Febrile convulsions are common in children and:
c. Asking her parents to visit at mealtime a. Usually occur after the first year of life
d. Giving her only the foods she likes best b. The cause is usually readily identified
c. May occur in minor illnesses
Situation: Mrs. Bronson is informed that her infant d. Occur more frequently in females than males
daughter has phenylketonuria (PKU). Questions 105
through 108 refer to this situation. 112. The physician orders acetaminophen 150mg, po,
105. Which of the following statements is true q4h, and prn for fever above 101F (38C). The nurse
concerning PKU? has on hand a bottle labeled “1 tablet equals 80mg.” The
a. PKU is transmitted by an autosomal dominant nurse should administer:
gene a. ½ Tablet
b. The infant is tested for PKU immediately after b. 1 Tablet

St. Louis Review Center Inc. – BUTUAN CITY; Telephone No. (085) 342 – 2339 22
c. 1 ½ Tablet a. Making decisions
d. 2 Tablet b. Tying his shoelaces
c. Abstract thought
113. Sam is diagnosed as having meningococcal d. Hand-eye coordination
meningitis. The nurse observes Sam for the:
a. Identifying purpuric skin rash 122. Sam loves to ride his bike, and his parents are very
b. Continual tremors of the extremities concerned about his activity when he returns home. The
c. Low-grade nature of the fever nurse bases the answer to them the fact that after the
d. Palatal paralysis and glossitis urinary findings are nearly normal:
a. He must remain in bed for 2 weeks
114. The most serious complication of meningitis in b. Activity does not affect the course of the disease
young children is: c. He must not play active games
a. Hydrocephalus d. Activity must be limited for 1 month
b. Blindness
c. Peripheral circulatory collapse Situation: Sue Green, a 2-year-old girl, is admitted to
d. Epilepsy the pediatric unit with respiratory wheezing, dyspnea,
and cyanosis. One of the tentative diagnoses is cystic
Situation: Three-month-old Matt Quincy is admitted to fibrosis. Questions 123 through 129 refer to this
the hospital with bile-stained vomitus and abdominal situation.
distention. Questions 115 through 118 refer to this 123. Cystic fibrosis can predispose Sue to bronchitis
situation. mainly because:
115. The nurse should also observe for: a. Tenacious secretions obstruct the bronchioles
a. Constant severe pain and absence of stools and respiratory tract and provide a favorable
b. Bounding pulse and hypotonicity medium for growth of bacteria
c. Paroxysmal pain and grunting respirations b. Increased salt content in saliva can irritate and
d. High-pitched cry and weak thready pulse necrose mucous membranes in nasopharynx
c. Neuromuscular irritability causes spasm and
116. Prior to surgery for the intestinal obstruction, Matt is constriction of the bronchi
kept NPO and has a Levin tube in place. To calm Matt d. The associated heart defects of cystic fibrosis
and also to meet his developmental needs best, the cause congestive heart failure and respiratory
nurse should: depression
a. Allow him to suck on a pacifier
b. Hang a brightly colored mobile in his crib 124. The problem of cystic fibrosis is sometimes first
c. Place him on his abdomen and permit him to noted by the nurse in the newborn nursery because of
crawl the infant’s:
d. Allow him to hold his favorite toy a. Increased heart rate
b. Abdominal distention
117. Matt develops diarrhea postoperatively and is given c. Excessive crying
IV fluids. The nurse to must observe the rate of flow d. Sternal retractions
often:
a. Avoid IV infiltration 125. Sue is small and underdeveloped for her age
b. Prevent increased output primarily because she:
c. Prevent cardiac embarrassment a. Ingested little food for several months because
d. Replace all fluids lost of poor appetite
b. Was unable to absorb nutrients because of a
118. When Mrs. Quincy returns to the surgical clinic for lack of pancreatic enzymes
follow-up care, the nurse includes the following c. Secreted less than normal amounts of pituitary
preventive suggestion in her teaching: growth hormone
a. Remove all tiny objects from the floor d. Developed muscular and bony atrophy from lack
b. Keep crib rails up to the highest position of motor activity
c. Cover electric outlets with safety plugs
d. Remove poisonous substances from low areas 126. When caring for the child with cystic fibrosis and the
nurse should:
Situation: Five-year-old Sam has been hospitalized with a. Perform postural drainage
acute glomerulonephritis. Questions 119 through 122 b. Encourage active exercise
refer to this situation: c. Prevent coughing
119. The nurse observes Sam primarily for: d. Provide small frequent feedings
a. Polyuria, high fever
b. Dehydration, hematuria 127 The foul-smelling, frothy characteristic of the stool in
c. Hypertension, circumocular edema cystic fibrosis results from the presence of large
d. Oliguria, hypotension amounts of:
a. Sodium and chloride
120. When planning nursing care for Sam, the nurse b. Semidigested carbohydrates
realizes that he needs help in understanding his c. Undigested fat
restrictions, one of which is: d. Lipase, trypsin, and amylase
a. Bed rest for at least 4 weeks
b. A bland diet high in protein 128. Medications that will probably be used for Sue in
c. Daily doses of IM penicillin her therapeutic regimen include:
d. Isolation from other children with infections a. A steroid and an antimetabolite
b. Antibiotics, a multivitamin preparation, and
121. The average 5-year-old is incapable of: cough drops

St. Louis Review Center Inc. – BUTUAN CITY; Telephone No. (085) 342 – 2339 23
c. Pancreatic enzymes and antibiotics b. Swimming
d. Aerosol mists, decongestants, and fat-soluble c. Badminton
vitamins d. Golf

129. In cystic fibrosis, frequent stools and tenacious 135. To assist her curvature correction, Sara is fitted
mucus often produce: with a Milwaukee brace. The nurse explains to Sara and
a. Intussusception her parents that the length of time the brace must be
b. Anal fissures worn varies but it is usually worn until:
c. Meconium ileus a. Cessation of bone growth at the time of physical
d. Rectal prolapse maturity
b. The curvature of the spine is completely
Situation: Two-year-old Mike Coz is admitted to the straightened
hospital for the second surgical repair of is clubfoot. Mrs. c. Pain on prolonged standing diminishes
Coz cannot stay overnight with her son, since visiting d. The iliac crests are at equal levels
hours are restricted. On the morning after admission,
Mike is standing in his crib crying. He refuses to be 136. One of the earliest signs of sexual maturity in a
comforted and calls for his mother. Questions 130 young girl such as Sara is:
through 133 refer to this situation: a. Interest in the opposite sex
130. The nurse approaches Mike to bathe him and he b. Attention to grooming
screams louder. This behavior is recognized as the c. An increase in the size of the breasts
stage of protest, and the nurse: d. The appearance of axillary and pubic hair
a. Picks him up and walks with him around the
room Situation: At 2 weeks of age Baby Williams begins to
b. Sits by his crib and bathes him later when his vomit after his feedings and is admitted to the hospital
anxiety decreases for observation with a tentative diagnosis of pyloric
c. Decides he really does not need a bath when he stenosis. Questions 137 through 141 refer to this
is this upset situation.
d. Fills the basin with water ands proceeds to 137. The nurse should carefully observe him for:
bathe him a. Signs of dehydration
b. Coughing and gagging after feeding
131. On the third postoperative day Mike begins to c. Quality of cry
regress and lies quietly in his crib with his blanket. The d. Quality of stool
nurse recognizes that Mike is in the stage of:
a. Denial 138. When vomiting is uncontrolled in an infant, the
b. Mistrust nurse should observe for signs of:
c. Rejection a. Tetany
d. Despair b. Alkalosis
c. Acidosis
132. During his second week of hospitalization, Mike d. Hyperactivity
smiles easily, goes to all the nurses happily, and does
not express a great deal of interest in his mother when 139. The maintenance of fluid and electrolyte balance is
she visits. After leaving Mike’s room, Mrs. Cox tells the more critical in children than in adults because:
nurse she is pleased that Mike is adjusting well but a. Renal function is immature in children below the
expresses some concern about his reactions to her. age of 4
Before responding to Mrs. Cox, the nurse should b. Cellular metabolism is less stable than in adults
understand Mike-s behavior and realize that he: c. The proportion of water in the body is less than
a. Is repressing his feelings for his mother in adults
b. Has established a routine and feels safe d. The daily fluid requirement per unit of body
c. Feels better physically so his behavior has weight is greater than in adults
improved
d. Has given up fighting and accepts the 140. What is the most critical actor confronting the nurse
separation in the administration of IV fluids to a small, dehydrated
infant?
133. The nurse explains the meaning of Mike’s behavior a. Maintenance of the prescribed rate of flow
to Mrs. Cox and tells her that after he goes home she b. Maintenance of the fluid at body temperature
should expect that: c. Calculation of the total necessary intake
a. Mike will miss the nurses and hospital routine d. Assurance of sterility
b. It will be easier for Mike to adjust to his home
situation 141. Surgery is performed and Baby Williams’ condition
c. Mike will continue his happy, normal behavior is stable. The nurse caring for him notices that is post-
d. It will take some time before the mother-child operative orders are similar to those for other infants
relationship is reestablished having undergone such surgery and include:
a. Withholding all feedings for the first 24 hours
Situation: Sara, 12 years old, was diagnosed at the b. Additional glucose feedings after the first 24
orthopedic clinic as having idiopathic scoliosis. Proper hours
exercising and avoidance of fatigue are essential c. Thickened formula 24 hours after surgery
components of Sara’s care. Questions 134 through 136 d. Diluted formula feeding 24 hours after surgery
refer to this situation:
134. Early in Sara’s treatment the nurse can suggest Situation: Karen Vale, a 5-year-old girl, is admitted to
which of the following sports as therapeutic? the hospital 1 week before surgery for tetralogy of Fallot.
a. Bowling Questions 142 through 146 refer to this situation:

St. Louis Review Center Inc. – BUTUAN CITY; Telephone No. (085) 342 – 2339 24
142. The defects associated with this heart anomaly c. Carefully monitoring urinary output
include: d. Decreasing external stimuli
a. Right ventricular hypertrophy, atrial and
ventricular defects, and mitral valve stenosis Situation: Three-day-old Patty is diagnosed as having
b. Right ventricular hypertrophy, ventricular septal congenital hip dysplasia. Questions 149 through 151
defect, stenosis of pulmonary artery, and refer to this situation.
overriding aorta 149. An early sign able to be observed by the nurse in
c. Origin of the aorta from the right ventricle and of the newborn nursery is:
the pulmonary artery from the left ventricle a. Limitation in adduction of the leg
d. Abnormal connection between the pulmonary b. Shortening of the leg on the unaffected side
artery and the aorta, right ventricular c. Depressed dance reflex
hypertrophy, and atrial septal defects d. Asymmetry of the gluteal folds

143. A common finding in most children with cardiac 150. At 3 months of age Patty has a spica cast applied
anomalies is: from below the axilla to below the knee. To prevent a
a. Mental retardation serious complication that often occurs in infants in a
b. Cyanosis and clubbing of fingertips spica cast, the nurse teaches Patty’s parents to:
c. A family history of cardiac anomalies a. Seek immediate medical care if Patty develops a
d. Delayed physical growth cough
b. Limit movement to prevent cast damage
144. Karen is to receive digoxin (Lanoxin) elixir, c. Change Patty’s diapers frequently
0.010mg, po. Based on developmental norms for a 5- d. Feed Patty a low-calorie diet
year-old, the nurse would withhold the medication and
notify the physician if the apical rate is below: 151. When elevating Patty’s head, the nurse is aware
a. 60 beats per minute that it is important to:
b. 80 beats per minute a. Limit this position to 1 hour at a maximum
c. 90 beats per minute b. Raise the entire mattress and spring at the head
d. 100 beats per minute of the bed
c. Use at least two pillows under her shoulders
145. Karen’s laboratory analysis indicates a high red d. Place folded diapers at the edge of the cast
blood cell count. This polycythemia can best be
understood as a compensatory mechanism for: Situation: Working on a cardiac care unit, the nurse will
a. Cardiomegaly care for many children with a variety of congenital
b. Low iron level anomalies. Questions 152 through 155 refer to this
c. Low BP situation.
d. Tissue oxygen need 152. Meg, 2 years old, has a cyanotic congenital heart
disease. The nurse would expect to observe:
146. Karen undergoes heart surgery to repair the a. Edema in the extremities
anomaly. Postoperatively it is essential that the nurse b. An elevated hematocrit
prevent: c. Absence of pedal pulses
a. Constipation d. Orthopnea
b. Unnecessary movement
c. Crying 153. Baby boy Vics has been found to have a patent
d. Coughing ductus arteriosus, which is:
a. A narrowing of the pulmonary artery
Situation: Johnny, a 15-year-old, is taken to the b. An abnormal opening between the right and left
emergency room of the local hospital because he ventricles
stepped on a nail. Questions 147 and 148 refer to this c. A connection between the pulmonary artery and
situation: the aorta
147. The puncture wound is cleansed and a sterile d. An enlarged aorta and pulmonary artery
dressing applied. While doing these tasks, the nurse
asks the mother if Johnny has been immunized against 154. The nurse is caring for a child with an acyanotic
tetanus. The reply is affirmative. Penicillin is heart disease. A major common symptom of acyanotic
administered, and Johnny is sent home with instructions heart disease is:
to return if there is any change in the wound area. A few a. Polycythemia
days later, Johnny is admitted to the hospital with a b. Clubbing of fingers and toes
diagnosis of tetanus. Legally: c. Severe retarded growth
a. The possibility of tetanus could not have been d. The presence of an audible heart murmur
foreseen, since he had been immunized
b. The nurse’s judgment was adequate in view of 155. Alma has coarctation of the aorta. When taking her
the client’s symptoms vital signs, the nurse can expect to observe:
c. Assessment by the nurse was incomplete and a. Weak, thready radial pulses
the treatment was inadequate b. Higher BP in upper extremities
d. Hospital protocol should govern treatment in c. Bounding femoral pulses
emergency room care d. Notching of the clavicle

148. After Johnny’s admission, one of the most Situation: Ten-year-old Jim Smith is admitted to the
important aspects of nursing care should be directed emergency room after a car accident. However, normal
toward: measures to stop his bleeding are unsuccessful, and, on
a. Maintaining body alignment further study, Jim is found to have a mild case of classic
b. Encouraging high intake of fluid hemophilia. Mr. And Mrs. Smith is very concerned about

St. Louis Review Center Inc. – BUTUAN CITY; Telephone No. (085) 342 – 2339 25
this and wonders how it happened. Questions 156 a. Lacks a swallow reflex
through 158 refer to this situation. b. Chokes on her feeding
156. In discussing hemophilia with Mr. and Mrs. Smith c. Pharynx and larynx
the nurse should explain that: d. Anal area
a. Hemophilia is an X-linked disorder in which the
mother is usually the carrier of the illness but is 164. When reviewing the data recorded on Nellie’s chart,
not affected by it the information that might indicate to the nurse that the
b. Hemophilia is an autosomal dominant disorder baby requires special attention would be:
in which the woman carries the trait a. Birth weight of 3500g
c. Hemophilia follows regular laws of Mendelian b. 20mL of milky-colored fluid aspirated from
inherited disorders such as sickle cell anemia stomach
d. This disorder can be carried by either male or c. The infant has a positive Babinski reflex
female but occurs in the sex opposite that of the d. The Apgar score at birth was 3
carrier
Situation: Three-year-old Roger is admitted to the
157. Jim’s parents are very worried about their other pediatric unit with a diagnosis of nephrosis. Questions
children, two girls and another boy, and want to know 165 through 168 refer to this situation.
what the chances are concerning their having the 165. The most important nursing intervention for Roger
disorder or being a carrier. An appropriate answer to this is:
question would be that: a. Encouraging fluids
a. All the girls will be normal and the other son a b. Regulating his diet
carrier c. Maintaining bed rest
b. Each son has a 50% chance of being a victim d. Preventing infection
and each daughter a 50% of being a carrier
c. All the girls will be carriers and one half the boys 166. As Roger gets older and has repeated attacks of
will be victims nephrosis, it is most important for the nurse to help him
d. Each son has a 50% chance of being either a develop:
victim or carrier, and the girls will all be carriers a. Fine muscle coordination
b. Acceptance of possible sterility
158. The most common site of internal bleeding in c. A positive body image
hemophiliacs is the: d. The ability to test his own urine
a. Cerebrum
b. Ends of the long bones
c. Intestines
d. Joints

Situation: Eight-year-old Cara has juvenile rheumatoid


arthritis. Drug therapy includes the administration of
sodium salicylate, 10gr, 4 times daily. Questions 159
through 161 refer to this situation:
159. During the salicylate therapy the nurse should
observe Cara for:
a. Nausea, dizziness, edema, headache
b. Gastric distress, nausea, vomiting, tinnitus
c. Constipation, deafness, nausea, headache
d. Diarrhea, gastric distress, edema of the face

160. Sodium salicylate is classified as an:


a. Antibiotic and antipyretic
b. Analgesic and antipyretic
c. Analgesic and sedative
d. Antipyretic and hypnotic

161. While Cara is in bed convalescing, she becomes


very bored and irritable. The nurse plans activities that a
school-age child would like and suggests she:
a. Play chess
b. Start a collection
c. Watch game shows on TV
d. Do arithmetic puzzles

Situation: Nellie, a newborn, is admitted to the pediatric


unit with the diagnosis of choanal atresia. Questions 162
through 164 refer to this situation.
162. Choanal atresia is an anomaly located in the:
a. Nasopharynx
b. Intestinal tract
c. Pharynx and larynx
d. Anal area

163. While feeding Nellie, the nurse notices that she:

St. Louis Review Center Inc. – BUTUAN CITY; Telephone No. (085) 342 – 2339 26
sufficient muscle mass. The Denver Developmental
Screening is an assessment tool. Subtracting six weeks
PRETEST from the due date because of prematurity is done for
01. The nurse has assessed four children of varying developmental assessment, not for immunizations.
ages; which one requires further evaluation? Administration of either live or inactive vaccines is not
a. A 7-month-old who is afraid of strangers contraindicated in prematurity.
b. A 4-year-old who talks to an imaginary playmate
c. A 9-year-old with enuresis 05. Fifteen-month-old Anne is brought in for her regular
d. A 16-year-old male who had nocturnal checkup. All of her immunizations are given as
emissions recommended. Anne’s mother asks the nurse what
(Answer: C) Rationale: The child may have a physiologic immunizations her daughter will receive today. The
or psychologic problem; all other answers indicate age- nurse’s best response is that she is due for her:
appropriate situations. a. Third DTaP
b. MMR vaccine
02. The nurse is caring for a 5-year-old child who has c. First Hib immunization
leukemia and is now out of remission and not expected d. First OPV immunization
to survive. The child says to his mother, “Will you take (Answer: B) Rationale: Current recommendations call for
care of me when I am dead the way you do now?” The measles, mumps, and rubella-combined vaccine (MMR)
child’s mother asks the nurse how to answer her child. to be given at 15 months. DTaP is given at 2, 4, and 6
The nurse’s response should be based on which of the months, followed by a booster 6 to 12 months after the
following understandings of the child’s behavior? third dose. OPV is given at 2 to 4 months followed by
a. The child is denying that he has a terminal boosters between 12 and 24 months and between 4 and
illness 6 years of age. Hib is given at 2, 4, and 6 months.
b. The child may be hallucinating
c. Children of this age do not understand the 06. The presence of what condition would necessitate a
finality of death change in the standard immunization schedule for a
d. Most 5-year-old children have a great fear of child?
mutilation a. Allergy to eggs
(Answer: C) Rationale: Preschool children do not b. Immunosuppression
understand the finality of death. They often view it as a c. Congenital defects
long sleep. It is common for preschoolers to ask who will d. Mental retardation
take care of them when they die. Preschool children may (Answer: B) Rationale: Immunosuppressed clients may
know the word dead but do not really comprehend what need alteration in immunization protection as live viruses
it means. The child is not in denial or hallucinating; he is may overwhelm them. Mental retardation and congenital
behaving normally for his age. Fear of mutilation is more defects would not interfere with the immunization
common among school-age children. The statement as schedule unless neurologic defects were present. MMR
given in the question does not describe mutilation. vaccine does contain eggs but a child allergic to eggs
would receive the vaccine on schedule in very small
03. The nurse is talking with the mother of a one-year- doses at 20-minute intervals with adrenalin available
old child in well-baby clinic. Which statement the mother should anaphylaxis occur.
makes indicates a need for more instruction in keeping
the child safe? 07. Jimmy Wilkins, 2 years old, is brought to the
a. “I have some syrup of ipecac at home in case pediatric clinic with an upper respiratory infection. After
my child ever needs it.” assessing Jimmy, the nurse informs the physician that
b. “I put all the medicines on the highest shelf in she suspects this child may be a victim of child abuse.
the kitchen.” Physical signs that almost always indicate child abuse
c. “We have moved all the valuable vases and are:
figurines out of the family room.” a. Diaper rash
d. “My husband put the gates up the top and b. Bruises on the lower legs
bottom of the stairs.” c. Asymmetrical burns on the legs
(Answer: B) Rationale: At one year of age babies are or d. Welts or bruises in various stages of healing
soon will be climbing on everything. Putting medicines (Answer: D) Rationale: Injuries at various stages of
on the highest shelf is not sufficient. All medicines healing are symptomatic of child abuse. Bruises on the
should be put in a locked cabinet. Syrup of ipecac lower legs are common occurrences in a healthy active
should be in every home in case of accidental poisoning. child. Diaper rash may be seen in a well-cared-for child.
The poison control center may recommend a dose syrup Burns characteristic of child abuse usually have shapes
of ipecac. The family is wise to “baby proof” the house resembling the item used to burn the child, such as
rather than always saying “no” to the child. Moving cigarette butts.
breakable valuable removes temptation from the child.
Putting gates up at the stairs will help to prevent falls. 08. Which parent-child interaction does NOT warrant
further assessment when child abuse is suspected? The
04. Suzy Walker was born 6 weeks prematurely. She is parent who:
not 2 months old and her mother brings her to the clinic a. Appears tired and disheveled
for her checkup. Administration of DTaP will depend on: b. Is hypercritical of the child
a. The presence of sufficient muscle mass c. Pushes the frightened child away
b. Whether the vaccines are live or inactive d. Expresses far more concern than the situation
c. The Denver Developmental Screening results warrants
d. Calculating her age by subtracting six weeks (Answer: A) Rationale: Being tired and disheveled gives
from the due date no clue to parent-child interaction. The parent who
(Answer: A) Rationale: DTaP is given intramuscularly, pushes the frightened child away, who is hypercritical of
therefore administration is dependent on the presence of the child, and who expresses far more concern than the

St. Louis Review Center Inc. – BUTUAN CITY; Telephone No. (085) 342 – 2339 27
situation warrants exhibits extremes in behavior that are (Answer: B) Rationale: A weight gain would suggest
seen frequently in parents who physically abuse their greater circulating volume. Blood has weight. Depressed
children. fontanels are a sign of dehydration. AS the infant
improves, the fontanels should no longer be depressed.
09. When child abuse is suspected, the nurse knows The skin turgor should improve, not remain poor as
that abusive burns will: described in option 3. Hematocrit rises with dehydration.
a. Have a number of scars There is less fluid in the blood and consequently a
b. Have identifiable shapes greater portion of blood is red cells.
c. Display an erratic pattern
d. Be on one side of the body 13. Jeffrey, age 17, has Down’s syndrome. He is 57
(Answer: B) Rationale: Burns typical of child abuse have inches tall and weighs 155 pounds. In planning his care,
symmetrical shapes and resemble the shape of the item it is most important for the nurse to take into
used to burn the child. When a child is burned consideration:
accidentally, the burns form an erratic pattern and are a. His mental age
usually irregular or asymmetrical. A number of scars are b. His chronologic age
inconclusive evidence of child abuse. When burning is c. His bone age
used as a means of physically abusing the child, the d. Growth chart percentiles
burns are usually bilateral. (Answer: A) Rationale: All Down’s syndrome children are
retarded and care must be geared to their mental age.
10. The mother of an eight-year-old brings her child to The other measurements of age do not reflect the
the physician because the child has a “funny red circle” special needs of these children.
on his leg. The mother reports that the child went on a
camping trip last weekend. The physician draws blood to 14. Down’s syndrome is caused by:
rule out Lyme disease and prescribed for a tick bite. The a. An autosomal recessive defect
nurse’s response is based on which of the following b. An extra chromosome
understanding? c. A sex-linked defect
a. Lyme disease weakens the person so they are d. A dominant gene
susceptible to infections (Answer: B) Rationale: In Down’s syndrome there is an
b. Lyme disease is caused by a spirochete that is extra chromosome on the 21st pair, which is why the
sensitive to doxycycline disease is also called trisomy 21. The mutation is
c. Doxycycline will kill the tick, which may still be in spontaneous and not inherited.
the child
d. Antibiotics are given to cure the infection at the 15. Louise was born with a myelomenigocele with
site of the tick bite accompanying hydrocephalus. She has had a shunt
(Answer: B) Rationale: Lyme disease is caused by a procedure to alleviate the hydrocephalus. Louise should
spirochete that is sensitive to antibiotics. Lyme disease be placed in which of the following positions?
does not make the client more susceptible to infections. a. Trendelenburg’s
The tick will not be in the child. There is not likely to be b. On her back
an infection at the site of the bite. The red bull’s eye c. With her legs adducted
around the tick bite is not an infection. d. On her abdomen
(Answer: D) Rationale: Pressure must be kept off the
11. The mother of a three-year-old child calls her nurse spinal sac. As there is paralysis of the lower extremities,
neighbor in a panic state, saying that the child swallowed the legs should be abducted. Trendelenburg’s position is
most of a bottle of aspirin. The nurse determines that the contraindicated in hydrocephalus.
child is still alert. In addition to calling the poison control
center, the nurse should: 16. The nurse is caring for an infant who is born with
a. Induce vomiting in the child hydrocephalus. She has a shunt inserted. Which of the
b. Observe the child carefully until the ambulance following signs indicates that the shunt is functioning
arrives properly?
c. Immediately start CPR a. The sunset sign
d. Give the child lots of milk to drink b. A bulging anterior fontanel
(Answer: A) Rationale: Since the child is still alert, the c. Decreasing daily head circumference
nurse should plan to administer syrup of ipecac. If the d. Widened suture lines
child were not conscious, he would be lavaged upon (Answer: C) Rationale: With improved draining of the
arrival at the emergency room. There is no indication CSF, the head circumference should become smaller.
that the child is not breathing; CPR in not indicated. It All the other signs indicate increased ICP.
would be better to remove the poison than to dilute it
with milk. 17. Henry is a 13-year-old who has been diagnosed as
having epilepsy. A positive sign that Henry is taking his
12. An eight-month-old infant was admitted to the Dilantin properly is:
hospital with severe diarrhea and dehydration. Fluid a. Hair growth on his upper lip
replacement therapy was initiated. Which observation b. Absence of seizures
the nurse makes indicates an improvement in the c. Lowered hemoglobin and hematocrit
infant’s status? d. Drowsiness
a. Fontanels are depressed (Answer: B) Rationale: Dilantin is an antiepileptic drug
b. Infant has gained 3 oz since yesterday that controls seizures. All the other signs ands
c. Skin remains pulled together after being gently symptoms are side effects of the medication.
pinched and released
d. The infant’s hematocrit is greater today than 18. Jennifer Pine, 3-years-old, is admitted with a
yesterday diagnosis of viral meningitis. During an initial
assessment of Jennifer the nurse would expect to find:

St. Louis Review Center Inc. – BUTUAN CITY; Telephone No. (085) 342 – 2339 28
a. Headache, fever, and petechiae treatment of viral meningitis. The client should resume
b. Seizures, lethargy, and hypothermia activities as tolerated in time.
c. Pallor, anorexia, and bulging fontanels
d. Fever, irritability, and nuchal rigidity 22. Phillip, 6-years-old, is brought to the Emergency
(Answer: D) Rationale: The clinical symptoms of viral Department unconscious after being hit by a car. The
meningitis (aseptic meningitis) include fever, irritability, most helpful information for the nurse performing the
and stiffness of the neck (nuchal rigidity). Other neurological examination on Phillip is the nurse’s
symptoms include drowsiness, photophobia, weakness, knowledge of:
painful extremities, and sometimes seizures. Aseptic a. Normal growth and development
meningitis usually resolves within tow weeks. Headache b. Phillip’s usual behavior and status
and fever may be seen in meningitis, but petechiae are c. Phillip’s past medical history
not. Seizures and lethargy may be seen in meningitis, d. Phillip’s growth and developmental progress
but hypothermia is not. Bulging fontanels are a sign of during infancy
hydrocephalus. (Answer: B) Rationale: The child’s usual behavior and
level of development is what provides critical information
19. To meet the sensory need of a child with viral about his pretrauma neurological function. This would
meningitis, nursing strategies should include: serve as baseline data, present prior to the injury. It is
a. Minimizing bright lights and noise important to know normal growth and development, but it
b. Promoting active range of motion is not individualized to this child. Phillip’s past medical
c. Increasing environmental stimuli history is important but is not the most helpful
d. Avoiding physical contact with family members information when doing the neurological exam.
(Answer: A) Rationale: Photophobia and hypersensitivity
to environmental stimuli are common clinical 23. The nurse is assessing a child who has a head injury
manifestations of meningeal irritation and infection. for the occulocephalic reflex (doll’s eyes). The nurse
Comfort measures include providing an environment that understands that the doll’s eye reflex is present it the
is quiet and has minimal stressful stimuli. Promoting child’s eyes:
active range of motion at this time would not be a. Move in the same direction in which his head is
beneficial. The client needs rest. Increasing turned
environmental stimuli would only exacerbate the b. Move in the direction opposite to which his head
symptoms of photophobia and hypersensitivity. The is turned
client needs quiet and rest. Viral meningitis is not c. Remain midline when his head is turned
contagious so contact with family members is not d. Move to the medial aspect of the orbit when his
contraindicated. head is turned
(Answer: B) Rationale: The occulocephalic reflex occurs
20. When addressing the emotional needs of the parents if, when the head of an unconscious child is turned
of a young child with meningitis, the primary focus rapidly in one direction, the eyes move in the opposite
should be on: direction. In the occulocephalic reflex, the eyes do not
a. Assuming all responsibility for physical care of move in the same direction in which the head is turned.
the child The eyes do not remain in the midline when the head is
b. Providing reassurance that the symptoms will turned. The eyes do not move to the medial aspect of
resolve within the week the orbit when the head is turned.
c. Reinforcing information about the child’s
condition and plan of treatment 24. Phillip’s pupils are dilated and react sluggishly. This
d. Explaining the importance of an optimistic is indicative of:
outlook when interacting with their child a. Barbiturate overdosage
(Answer: C) Rationale: Successful coping in times of b. Damage to the diencephalon
anxiety and stress requires that the nurse be available to c. Damage to the sympathetic system
provide information that validates parental right to know d. Damage to the parasympathetic system
and participation in their child’s care. Honesty, patience, (Answer: D) Rationale: When dilated pupils react
and unhurried repetition of information are necessary to sluggishly or are non reactive, it is an indication that
support the family. The parents should not be expected there has been damage to the parasympathetic nervous
to assume all responsibility for the child’s physical care. system, which controls the pupillary construction
The symptoms will not necessarily resolve within a response. Damage to the diencephalon would show
week. An optimistic outlook is also important when pupils that are small and reactive to light. Barbiturates
interacting with the child but it is not the primary focus. cause pupillary dilation. Damage to the sympathetic
nervous system would inhibit the normal dilation
21. Discharge teaching for the parents of a child who response and would cause pupils to be constricted.
had meningitis should include:
a. Engaging a tutor to assist with learning problems 25. A 4-year-old with tetralogy of Fallot is seen in a
b. Administering the prescribed antibiotic squatting position near his bed. The nurse should:
c. Notifying the physician if her fever or headache a. Administer oxygen
persists more that a few days after discharge b. Take no action if he looks comfortable but
d. Encouraging Jennifer to resume normal continue to observe him
activities immediately c. Pick him up and place him in Trendelenburg’s
(Answer: C) Rationale: Parents should be instructed to position in bed
contact the physician if the child’s symptoms worsen or d. Have him stand up and walk around the room
persist. Te child recovering from viral meningitis should (Answer: B) Rationale: Squatting is a normal response to
show signs of feeling better a week after discharge. A a cyanotic heart defect. This position increases
tutor may be necessary if cognitive function priority in pulmonary blood flow because it changes the
discharge teaching. Antibiotics are not indicated in the relationship between systemic and pulmonary vascular
resistance. The squatting position alone should increase

St. Louis Review Center Inc. – BUTUAN CITY; Telephone No. (085) 342 – 2339 29
the child’s oxygen level, so administration of oxygen will a. The fact that you brought Alice to the hospital
most likely not be necessary. Picking the child up and early enough will decrease the chance of her
placing him in a Trendelenburg position (head down) siblings getting it
decreases his pulmonary blood flow and makes it harder b. It is caused by an autoimmune reaction an is not
for him to breathe. His cardiopulmonary status would not contagious
allow him to walk around the room at this time. c. You appear concerned that your daughter’s
disease is contagious
26. Christopher, 2-month-old, is suspected of having d. Your other children should be taking antibiotics
coarctation of the aorta. The cardinal sign of this defect to prevent them from catching rheumatic fever
is: (Answer: B) Rationale: Rheumatic fever is an
a. Clubbing of the digits and circumoral cyanosis autoimmune reaction to a streptococcal infection and is
b. Pedal edema and portal congestion limited to the person having the reaction. Rheumatic
c. Systolic ejection murmur fever is not a contagious disease.
d. Upper extremity hypertension
(Answer: D) Rationale: Coarctation of the aorta is 31. A 10-year-old child is admitted with rheumatic fever.
characterized by upper extremity hypertension and In addition to carditis, the nurse should assess the child
diminished pulses in the extremities. These signs would for the presence of:
be seen in a child with a cyanotic heart defect. Pedal a. Arthritis
edema and portal hypertension are seen in clients with b. Bronchitis
congestive heart failure secondary to many congenital c. Malabsorption
cardiac disorders. Systolic ejection murmurs are present d. Oliguria
in a child with atrial septal defect. (Answer: A) A major symptom of rheumatic fever is
arthritis. Bronchitis is not seen with rheumatic fever.
27. When assessing the apical heart rate in infants and Malabsorption is not seen with rheumatic fever. Oliguria
toddlers, the point of maximal impulse (PMI) is: is not seen with rheumatic fever.
a. Between the third and fourth left intercostal
space 32. The Tates have one child with hemophilia. Mrs. Tate
b. Between the fourth and fifth left intercostal wants to have another child and she asks the nurse what
space her chances are of having another child with hemophilia.
c. At the fifth intercostal space to the right of the The nurse’s best response is:
midclavicular line a. All of your daughters will be carriers of the
d. In the aortic area disease
(Answer: A) Rationale: The heartbeat is most easily b. If you have another son, there is almost a 100%
counted at the point of maximum impulse. From birth chance he will have hemophilia
through toddlerhood it is located between the third and c. If you have a son, there is a 50% chance he will
fourth left intercostal space. The PMI in an adult is have hemophilia but none of your daughters will
located at the fifth intercostal space inside the left have it
midclavicular line. d. There is a 25% chance of having another child
with hemophilia
28. Two-week-old Jonathon has a patent ductus (Answer: C) Rationale: Hemophilia is inherited as an X-
arteriosus. Prior to administering digoxin the nurse linked recessive trait. If this family has another son, there
should: is a 50% chance that he will have the disease. If they
a. Take the apical pulse for 30 seconds and have a daughter, she will not have the disease but there
multiply by 2 is a 50% chance that she will be a carrier of the disease.
b. Give the medication if his pulse is 92, but notify
the physician 33. John is 4-years-old and has been diagnosed as
c. Take the radial pulse for 1 full minute having iron deficiency anemia. A liquid iron preparation
d. Give the medication after finding that the pulse has been prescribed. When administering John’s
is 135 beats/minute medication the nurse should:
(Answer: D) Rationale: The apical pulse is taken for 1 full a. Ask him if he wants to take his medicine
minute and the medication is withheld if the pulse is less b. Mix the medication in his milk bottle and give it
than 100 beats/minute. to him at nap time
c. Allow him to sip the medication through a straw
29. The nurse is planning care for a two-week-old infant d. Give the medication after lunch with a sweet
who has a congenital heart defect. Which of the dessert t disguise the taste
following actions is not appropriate? (Answer: C) Rationale: Iron is given with a straw to
a. Using a soft “preemie” nipple for feedings prevent staining the teeth. It is best absorbed on an
b. Providing passive stimulation empty stomach in an acidic environment. Do not ask
c. Allowing him to cry to promote increased toddlers if they want to take their medication. They are
oxygenation likely to say no.
d. Placing him in orthopneic position
(Answer: C) Rationale: Crying expends already 34. Todd, age 10, has hemophilia A and is admitted to
decreased energy. All the other measures help to the hospital for hemarthrosis of the right knee. He is in
compensate for the decreased cardiac output. great deal of pain. Which of the following interventions
would aggravate his condition?
30. Alice White, 10-years-old, has been hospitalized for a. Applying an ice bag to the affected knee
two weeks with rheumatic fever. Alice’s mother b. Administering children’s aspirin for pain relief
questions whether her other children can catch the c. Elevating the right leg above the level of his
rheumatic fever. The nurse’s best response is: heart
d. Keeping the right leg immobilized

St. Louis Review Center Inc. – BUTUAN CITY; Telephone No. (085) 342 – 2339 30
(Answer: B) Rationale: Aspirin interferes with the clotting and cries with pain with his wrists and elbows are
mechanism. All the other measures listed will help moved. He is admitted to the hospital with a diagnosis of
decrease bleeding or alleviate the associated pain. sickle cell crisis. Sammy’s mother asks the nurse why
Sammy has not been symptomatic before now. The best
35. Melissa Heller, 17-years-old, has had sickle cell response by the nurse would be:
anemia since she was a toddler. Her brother has sickle a. High fetal hemoglobin protected Sammy against
cell trait. The client with sickle cell trait: sickling
a. Has a chronic form of sickle cell anemia b. His red blood cell levels remained normal
b. Has the most lethal form of the disease c. Maternal antibodies protected Sammy against
c. Will transmit the disease to all children sickling
d. Has some normal and some abnormal d. Sickle cell hemoglobin was not present until
hemoglobin cells about 1 year of life
(Answer: D) Rationale: Clients with sickle cell trait inherit (Answer: A) Rationale: High levels of fetal hemoglobin
only one defective gene. They can synthesize both inhibit sickling of red cells prior to the age of 6 months.
normal and abnormal hemoglobin chains. If two people His red blood cell level may have been normal, but his
with sickle cell trait marry, some of their children may abnormal blood cells were protected by the fetal
inherit abnormal genes. It is not a lethal form of the hemoglobin. Maternal antibodies do not protect the
disease, nor is it sickle cell anemia. baby’s blood cells from sickling. The baby is still not a
36. The mother of a child with sickle cell anemia tells the year old.
nurse that, when she was reading about sickle cell
anemia, she learned that sickled blood cells do not have 41. In planning care for a child with newly diagnosed
as long a life expectancy as normal red cells. The life sickle cell anemia, his mother should be taught that
expectancy of a sickled blood cell is approximately: vaso-occlusive crises might be prevented by:
a. 5 days a. Prophylactic administration of acetaminophen
b. 15 days b. Eating food with a high iron content
c. 30 days c. Exercising regularly
d. 60 days d. Promoting hydration
(Answer: B) Rationale: The life span of a sickled cell is 6 (Answer: D) Rationale: Promoting good hydration is a
to 20 days major factor in maintaining the blood viscosity needed to
maximize the circulation of red blood cells. Good
37. The child with sickle cell anemia may exhibit: hydration can help to minimize the severity of symptoms
a. Vitiligo should the child develop sickle cell crisis.
b. Hyperactivity Acetaminophen would not be helpful in decreasing the
c. Mild mental retardation sickling process. Iron administration would be of no
d. Delayed physical development value because in sickle cell anemia there is abnormally
(Answer: D) Rationale: Children with sickle cell disease shaped hemoglobin. The problem is not due to iron
manifest an impairment of growth. They do not deficiency. Regular exercise is important for children
experience mental retardation, hyperactivity, or Vitiligo with sickle cell disease, but it will not prevent a vaso-
(abnormal pigmentation). occlusive crisis.

38. A child who has sickle cell anemia has developed 42. Jimmy had a tonsillectomy performed earlier in the
stasis ulcers on her lower extremities. This is due to: day. He is now 4 hours post-op. Which of the following is
a. Poor range of motion an abnormal finding and a cause for concern?
b. Ruptured blood vessels a. An emesis of dried blood
c. Impaired venous circulation b. Increased swallowing
d. Hypertrophy of muscular tissue c. Pink-tinged mucus
(Answer: C) Rationale: The tissues of a client with sickle d. Jimmy’s complaints of a very sore throat
cell disease are constantly vulnerable to microcirculatory (Answer: Increased swallowing could be a sign of
interruptions. The vessels do not actually rupture; they hemorrhage from the surgical site. Vomiting of dried
are blocked with the defective cells. The stasis ulcers blood would be expected following a tonsillectomy. Pink-
are not related to range of motion. Hypertrophy of tinged mucus would be an expected finding. Complains
muscular tissue does not occur with sickle cell anemia. of a very sore throat would be expected following
surgery:
39. Which complication is associated with sickle cell
anemia? 43. Kevin is 7-years-old and has been diagnosed as
a. Constipation having cystic fibrosis. Chest physiotherapy has been
b. Hypothyroidism ordered. Kevin’s chest percussion should be performed:
c. Addison’s disease a. Before postural drainage
d. Cerebrovascular accidents b. ½ hour before meals
(Answer: D) Rationale: The sudden appearance of a c. Before an aerosol treatment
stroke in sickle cell anemia is related to the d. After suctioning
microcirculatory interruptions that are caused by the (Answer: B) Rationale: Chest percussion is done prior to
sickled cell. The blood supply to the bowel may be meals to prevent vomiting. It follows aerosol therapy and
damaged; however, constipation is not a common positioning. Suctioning may be needed afterwards to
problem associated with sickle cell disease. Addition’s remove thick secretions after percussion has loosened
disease and hypothyroidism are endocrine disorders that them.
are not related to sickle cell disease.
44. The nurse is performing chest physiotherapy on a 6-
40. Both of Sammy’s parents carry the sickle cell anemia year-old child who has congestion in his left lower lobe.
trait. Sammy, 8-month-old, contracted chickenpox from The nurse should position the child on his:
his brother and now is very weak, febrile, and anorexic, a. Left side in semi-Fowler’s position

St. Louis Review Center Inc. – BUTUAN CITY; Telephone No. (085) 342 – 2339 31
b. Right side in semi-Fowler’s position (Answer: A) Rationale: Leaving something of his
c. Left side in Trendelenburg’s position mother’s with the child and telling him that she will be
d. Right side in Trendelenburg’s position back in the morning is the best approach in developing
(Answer: D) Rationale: The affected lobe must be trust between the mother and her child. Sneaking out
uppermost to be drained by gravity. and not being truthful with the child does not allow the
child to develop trust. Lying to the child and giving him
45. An infant is being evaluated for possible cystic false hope that the mother will return after supper is not
fibrosis. The sweat test will show an elevation of which appropriate.
electrolyte?
a. Chloride 49. Albert is a 2-year-old who has cystic fibrosis. His
b. Fluoride mother tells the nurse that the family is planning their
c. Potassium first summer vacation with Albert. She wants to know if
d. Calcium there are any special precautions needed because he
(Answer: A) Rationale: There is increased excretion of has cystic fibrosis. The nurse should tell her that children
chloride in the sweat of children wit cystic fibrosis. A with cystic fibrosis are particularly susceptible to:
chloride level of over 60 mEq/liter is diagnostic for the a. Severe sunburn
disease. b. Infectious diarrhea
c. Heat prostration
46. Michael, 2-years-old, is admitted to the hospital with d. Respiratory allergies
cystic fibrosis. As a result of malabsorption, he is small (Answer: C) Rationale: Clients with cystic fibrosis are
for his age. What dietary suggestions can the nurse pone to electrolyte imbalances due to increased loss of
recommend to Michael’s mother to enhance his growth? sodium and potassium in their sweat. The mother should
a. Low-fat, low-residue, and high-potassium diet avoid having her child become overheated and should
b. Low-carbohydrate, soft diet with no sugar frequently replenish body fluids with water or fruit juices.
products A child with cystic fibrosis is no more susceptible to
c. High-carbohydrate, high-fat diet with extra water sunburn than any other child. Any young child should be
between meals protected with sunscreen when in the sun for long
d. High-protein, high-calorie meals with skim- periods of time. A child with cystic fibrosis is no more
milkshakes between meals likely to get infectious diarrhea than any other child,
(Answer: D) Rationale: A client with cystic fibrosis lacks although they do have noninfectious diarrhea. A child
pancreatic enzymes necessary for fat absorption. A diet with cystic fibrosis is no more likely to have respiratory
high in protein and calories is necessary to meet the allergies than any other child. A child with cystic fibrosis
child’s growth needs. Between-meal snacks of skim-milk will have respiratory problems but they will not
milkshakes may be given to provide additional protein, necessarily be allergy related.
vitamins, and calories. The child needs carbohydrates. A
soft diet with sugar restriction is not soft diet with sugar 50. Amy, 4-years-old, is admitted to the hospital for the
restrictions is not necessary. Fat must be restricted in a treatment of an acute asthma attack. Her health history
client with cystic fibrosis. reveals that she has been blind since birth and has had
four asthma attacks in the past six months. She received
47. The nurse is caring for a 2-year-old who has cystic epinephrine (Adrenalin) in the emergency department
fibrosis. His mother asks why the child developed cystic and was transferred to the pediatric unit with
fibrosis. The nurse explains that cystic fibrosis: aminophylline infusion. When evaluating Amy for
a. Develops due to meconium ileus at birth positive effects of the aminophylline treatment, the most
b. Is an autosomal recessive genetic defect significant finding is:
c. Occurs during embryologic development a. A decrease in mucous production
d. Results from chromosomal nondysjunction that b. A decrease in wheezing
occurred at conception c. An increase in blood pressure
(Answer: B) Rationale: Cystic fibrosis is an autosomal d. A sleeping child
recessive genetic disease. If both parents have the (Answer: B) Rationale: Aminophylline is a
cystic fibrosis trait, each child has a 25% chance of bronchodilator. As it exerts its effects, wheezing will
developing the disease, a 50% chance of not having the decrease. Aminophylline has no effect on mucous
disease. Meconium ileus (obstruction of the small production. A toxic effect of aminophylline is
intestine in the newborn caused by impaction of thick, hypotension. A sleeping child does show evidence that
dry, tenacious meconium) often occurs but is a result of the drug worked, but this is not the most significant
cystic fibrosis and is one of the earliest signs. Autosomal finding.
disorders are hereditary and depend on the autosomes.
It is unrelated to embryological development. Autosomal 51. Bryan, 12-months-old, is hospitalized for a severe
disorders are hereditary. Te abnormal recessive gene case of croup and has been placed in an oxygen tent.
present at conception. Today the oxygen order has been reduced from 35% to
25%. His blood gases are normal. Bryan refuses to stay
48. A 2-year-old is admitted to the hospital and will need in the oxygen tent. Attempts to placate him only cause
t stay for several days. The child’s mother is unable to him to become more upset. The most appropriate action
stay overnight because there is no one to care for her for the nurse is to:
other children. The nurse recommends that she: a. Restrain him in the tent and notify the physician
a. Leave something of hers with Michael and tell b. Take him out of the tent and notify the physician
him she’ll be back in the morning c. Take him out of the tent and let him sit in the
b. Leave while he is in the playroom playroom
c. Leave after he has faller asleep d. Tell him it will please his mother if he stays in
d. Tell him she’ll be back in a few minutes after she the tent
has dinner (Answer: B) Rationale: The energy exerted by the child
in resisting the oxygen tent is causing increased

St. Louis Review Center Inc. – BUTUAN CITY; Telephone No. (085) 342 – 2339 32
respiratory effort. The child should be removed from the (Answer: A) Rationale: Cleft palate repair should be
tent and closely monitored to be sure that he handles done prior to the development of speech. This allows for
being in room air. The physician should be notified the formation of a more normal speech pattern. Being
because the oxygen content of room air is only 20%, toilet trained has no relation to cleft palate surgery. The
which is less than that ordered. Restraining him in the child does not have to be completely weaned form the
oxygen tent would further increase respiratory effort. It bottle and the pacifier prior to the surgery. However, the
would be best to leave him in his room where he can be child must be able to drink form a cup because a bottle
closely monitored and where the oxygen tent is available may not be used until surgical repair is healed. A large
if needed. A proper nursing response would help the holed nipple is generally used for a premature infant with
child deal with the treatment by himself and not just do it a weak sucking reflex. The child with cleft palate repair is
to please someone else. The twelve-month-old is not allowed to suck until the repair is healed.
moving into the stage of toddlerhood and the
development of a sense of autonomy. 56. Bobby, 2-years-old, has had a cleft palate repair. A
priority in the post-op plan of care for Bobby includes
52. Janie, age 9, has celiac disease, which has been in teaching his mother:
good control since it was diagnosed six years ago. She a. To resume toilet training after he is up and
has now been admitted to the hospital for an emergency around
appendectomy. Which preoperative procedure should b. To use a cup or wide bowl spoon for feeding
the nurse withhold? c. That he will be more prone to respiratory
a. A cleansing enema infections now that his airway is smaller
b. Starting an IV d. That no further treatment will be needed until his
c. Keeping her NPO adult teeth come in at age 6
d. Obtaining a blood sample for a CBC (Answer: B) Rationale: Care must be taken to not put
(Answer: A) Rationale: Enemas, cathartics, and heat to anything in the mouth that could damage the suture line.
the abdomen should all be avoided in appendicitis Toilet training has no relationship to the surgery done.
because they may cause perforation of the appendix. All The child may have regressed in toilet training since the
the other measures are appropriate pre-op interventions. surgery but may begin again whenever he seems
interested. Now that the repair has been done he should
53. Alice, age 8, has celiac disease. She had an experience fewer respiratory infections. Additional
emergency appendectomy. She is progressing well and stages of treatment may need to be done prior to age 6.
is having her first real meal. Which food should the nurse
remove from her tray? 57. Tyrone, age 4 years, has just been diagnosed as
a. Chicken rice soup having nephrotic syndrome. His potential for impairment
b. Crackers of skin integrity is related to:
c. Hamburger patty a. Joint inflammation
d. Fresh fruit cup b. Drug therapy
(Answer: B) Rationale: The prescribed diet for children c. Edema
with celiac disease is gluten free; crackers contain d. Generalized body rash
gluten, but the other foods do not. (Answer: C) Rationale: A child with nephrotic syndrome
will have massive edema. A child with edema is prone to
54. Barry K., 12-months-old, had a cleft lip repaired skin breakdown. Joint inflammation and generalized
successfully as an infant. His mother brings him to the body rash are not usually problems in the child with
clinic for a checkup and his MMR immunization. While nephrotic syndrome. Drug therapy is usually steroids
talking to the nurse, Ms. K. reports that her teenage and diuretics, neither of which is related to impairment of
babysitter has just come down with rubeola. The most skin integrity.
appropriate plan of treatment for Barry is to:
a. Administer immune serum globulin 58. Walter is 20 months old and admitted to the hospital
b. Administer prophylactic penicillin with a diagnosis of cryptorchidism. Surgical correction is
c. Vaccinate him now with MMR performed at this time to prevent:
d. Allow him to catch measles from the babysitter a. Difficulty in urinating
in order to develop active immunity b. Sterility
(Answer: A) Rationale: Administration of immune serum c. Herniation
globulin will provide the child wit passive immunity to d. Peritonitis
prevent a full-blown case of measles or reduce the (Answer: B) Rationale: if the testes remain in the
severity of symptoms. Penicillin does not prevent or treat abdomen beyond the age of 5, damage resulting from
a viral infection. Active immunization with MMR should exposure to internal body temperature can result in
be given at a later time. It takes time for active immunity sterility.
to develop. It is not appropriate to let the child get a
disease when the disease can be prevented. 59. Charlie Urich, 3 days old, is diagnosed with
55. The nurse is caring for a 12-month-old child who has Hypospadias. His parents are very upset and have been
a cleft palate. A cleft lip was repaired when he was2 willing listeners as the nurse has explained this problem
months old. His mother asks the nurse when he will be to them. The nurse explained that in Hypospadias, the
ready for a cleft palate repair. The most appropriate physical problem is primarily:
response is that cleft palate repair is usually done: a. Ambiguous genitalia
a. Prior to development of speech b. Urinary incontinence
b. When the child is toilet trained c. Ventral curvature of the penis
c. When the child is completely weaned from the d. Altered location of the urethral meatus
bottle and pacifier (Answer: D) Rationale: In Hypospadias, the urethral
d. When a large-holed nipple is ineffective for his opening may be anywhere along the underside of the
feedings penis. This dislocation may be accompanied by genital
edema and fibrous penile dislocation, which may make

St. Louis Review Center Inc. – BUTUAN CITY; Telephone No. (085) 342 – 2339 33
sexual identification difficult. If uncorrected, the boy will (Answer: D) Rationale: Displacement of the hip on one
have problems controlling his urinary stream when he side causes asymmetry of skin folds. The baby will not
has outgrown diapers. have pain when her leg is abducted, and skin folds near
he buttocks and thighs will not be symmetrical.
60. The parents of a newborn who has Hypospadias ask
about surgical repair. They are told that the preferred 65. An infant is being treated for congenital hip dysplasia
time to schedule surgical repair of Hypospadias is when with a Pavlik harness. The baby’s mother asks if she can
the boy is: remove the harness if it becomes soiled. The best
a. 9 months old response for the nurse to make is:
b. 5 years old a. No, the harness may not be removed
c. 12 years old b. No, she will only be wearing it a few days
d. 17 years old c. Yes, just long enough to clean the area
(Answer: A) Rationale: Most surgical repairs are d. Yes, just overnight while she is sleeping
scheduled for the child between 6 and 18 months of age. (Answer: A) Rationale: The harness is not to be removed
If left until later, body image problems and until the hip is stable with 90 degrees of flexion and x-ray
mutilation/castration anxieties may occur. confirmation. This usually occurs after about three
weeks in a Pavlik harness. No, she cannot remove it, but
61. The parents of a baby boy who was born with it will be in place about three weeks until the hip is
Hypospadias want to know about the surgical repair. The stable.
nurse tells them that they will be able to evaluate the
success of Hypospadias surgery by: 66. A 10-year-old Jamie takes aspirin QID for Still’s
a. The cosmetic appearance of the penis disease (juvenile rheumatoid arthritis). What symptoms
b. Maintaining stable blood pressure in the child would her mother observe that would be indicative of
c. Observing a straight stream when he voids aspirin toxicity?
d. His ability to void without discomfort a. Hypothermia
(Answer: C) Rationale: Observing the child void in a b. Hypoventilation
straight stream while standing is the expected successful c. Decreased hearing acuity
outcome of Hypospadias repair. The penis may be d. Increased urinary output
swollen and discolored after surgery and voiding may be (Answer: C) Rationale: Tinnitus or ringing in the ears is a
uncomfortable initially. Having a stable blood pressure is side effect of aspirin therapy. Hypothermia,
always important, but its presence does not indicate hypoventilation, and increased urinary output are not
successful surgical repair of Hypospadias. signs of aspirin toxicity. In salicylate poisoning, the child
will have hyperthermia, hyperventilation to compensate
62. Doug, age 3, has a fractured femur and is in Bryant’s for metabolic acidosis and may develop renal failure.
traction. To evaluate correct application of the traction
the nurse should note that: 67. Jenny, age 8, is newly diagnosed with diabetes
a. Doug is being continuously and gradually pulled mellitus. Which of the following symptoms is different
toward bottom of bed from what you would expect to find in maturity onset
b. Doug’s buttocks are raised slightly (Type II) diabetes?
c. Doug’s leg is at a 45 angle to the bed a. Increased appetite
d. Doug can move the unaffected leg freely b. Increased thirst
(Answer: B) Rationale: In Bryant’s traction both legs are c. Increased urination
in traction at a 90 angle and the buttocks are raised off d. Weight loss
the mattress. The child’s buttocks are raised off the (Answer: D) Rationale: Weight loss is associated with
mattress. The child’s weight provides the juvenile diabetes, whereas weight gain develops in
countertraction; he should not be pulled down toward the maturity-onset diabetes. The other three signs appear in
bottom of the bed. both forms of the disease.

63. Ethel, age 14, is in a hip spica cast. To turn her 68. Jane is 7 years old and newly diagnosed with
correctly, the nurse should: diabetes mellitus. She had an injection of regular and
a. Use the cross bar NPH insulin at 7:30 A.M. At 3:10 P.M. she complains
b. Turn her upper body first, then turn the lower that she does not feel well. She is pale, perspiring, and
body trembling. The nurse should:
c. Log roll her a. Tell her to lie down and wait for the dinner trays
d. Tell her to pull on the trapeze and sit p to help in to arrive
turning b. Ask her to give a urine specimen and test it for
(Answer: C) Rationale: The client in a hip spica cast sugar and acetone
should be turned as a unit. The stabilizing bar should not c. Give her a carbohydrate snack
be handled. d. Administer the afternoon dose of regular insulin
(Answer: C) Rationale: She probably is having a
64. A routine physical examination on 2-day-old Melissa hypoglycemic reaction form the NPH having a
Carter uncovered evidence of congenital dislocation, or hypoglycemic reaction from the NPH insulin and needs
dysplasia, of the right hip. When assessing Melissa, a an afternoon snack.
sign of one-sided hip displacement is:
a. An unusually narrow perineum 69. A 10-year-old with diabetes mellitus is learning how
b. Pain where her leg is abducted to administer her insulin. She asks the nurse why she
c. Symmetrical skin folds near her buttocks and cannot take pills like her grandmother who also has
thigh diabetes. The best response for the nurse to make is:
d. Asymmetrical skin folds over the buttocks and a. How long has your grandmother been taking
thigh oral medication?

St. Louis Review Center Inc. – BUTUAN CITY; Telephone No. (085) 342 – 2339 34
b. You’ll be able to stop taking insulin once you a. Leave her alone as much as possible and
stop growing whisper when in her room in order not to disturb
c. You have a different kind of diabetes and you her:
will need to take insulin throughout your life b. Assist her in giving away her possessions to
d. You’ll be able to switch to pills when you reach friends and family
your grandmother’s age c. Encourage Louise’s parents to explain to her 5-
(Answer: C) Rationale: Juvenile diabetics need lifetime year-old sister that Louise will asleep for a long
insulin. She will never be able to switch to oral time
hypoglycemics. d. Reduce emotional stress by not having Louise’s
parents/family participate in her care
70. A 10-year-old is being prepared for a bone marrow (Answer: B) Rationale: Adolescents may want to give
transplant. The nurse can assess how well he away their belongings. The other interventions are all
understands this treatment when he says: inappropriate and may increase Louise’s fears and
a. I’ll be much better after this blood goes to my anxiety.
bones
b. I won’t feel too good until my body makes 74. Jack is 10-years-old and is receiving cranial
healthy cells irradiation for a brain tumor. He has developed alopecia.
c. This will help all of the medicine they give me to Which of the following is an appropriate nursing
work better intervention?
d. You won’t have to wear a mask and gown after a. Have Jack identify famous movie stars and
my transplant sports heroes who are bald
(Answer: B) Rationale: The goal of a bone marrow b. Assure Jack that his hair will grow in before he
transplant is to have the donor cells produce functioning leaves the hospital
blood cells for the client. In a bone marrow transplant, c. Wrap a bandage around his head
blood is not transfused into the client. Instead bone d. Help him select a variety of hats
marrow is given to the person in the hopes that it will (Answer: D) Rationale: Selecting hats to cover his head
begin producing normal, healthy blood cells. A bone will help Jack deal with the change in his body image.
marrow transplant is not done to increase the The other suggestions are all inappropriate.
effectiveness of medication therapy. A mask and gown
will still be worn until the clients begin to produce normal
white blood cells.

71. Susan, age 4, has leukemia. Her mother


understands the white count involvement in this disease
but doesn’t understand why her child has bruises and POSTTEST
anemia. The nurse explains that: 01. A mother asks the pediatric nurse about what she
a. All blood cells are made in the bone marrow and should begin to feed her 6-month-old infant. The correct
therefore all types will be affected response is:
b. The anemia is because her child hasn’t been a. Egg whites are the least allergenic food to be
eating well; the bruises are from the multiple introduce into the baby’s diet
needle sticks b. Rice cereal is the first solid introduced that is
c. They are related to inactivity least allergenic of the cereals
d. This is indicative that the end is near c. Formula is the only source of nutrition given for
(Answer: A) Rationale: In leukemia, bone marrow is the first year
replaced by blast cells, resulting in decreased white d. Fruits and vegetables are good sources of iron
cells, red cells, and platelets. The bruises are due to the (Answer: B) Rationale: Introduction of solid food is
child’s decreased platelet count. recommended at age 4 to 6 months, when the
gastrointestinal system has matured sufficiently to
72. A 14-year-old Louise has had an exacerbation of handle complex nutrients. The suck reflex and tongue-
acute lymphocytic leukemia. The primary effect of thrust reflex diminish at 4 months of age. Rice cereal is
leukemia on the bone marrow is: the first solid food because it is a rich source of iron and
a. Crowding out of normal bone marrow cells rarely induces allergic reactions. Fruits and vegetables,
b. Proliferation of cells producing blood good sources of vitamins and fiber, are introduced after
components cereal, one at a time to determine allergic reactions. Egg
c. A selective reduction in number of neutrophils whites are highly allergenic.
d. Leukopenia, thrombocytopenia, and anemia
(Answer: A) Rationale: Since leukemia cells are capable 02. A 1-year-old male child is scheduled for a routine
of an increased rate of production and a long cell life, exam at the pediatric clinic. The child’s birth weight was
they crowd out all of the normal bone marrow cells. Cells 8 lbs, 2 oz. The child now weights 18 lbs, 4 oz. The
producing blood components are then unable to nurse knows that this weight is:
reproduce. There is no selective reduction, although the a. Below the expected weight
neutrophils, too, may be crowded out in nonmylocytic b. Appropriate for the child’s age
leukemias. Leuko0enia is an absolute decrease in the c. Above the expected weight
number of white blood cells in the peripheral circulation; d. Individualized and thus unpredictable
in leukemia, the white blood cells count rises. (Answer: A) Rationale: The first year of life is one rapid
growth. The birth weight usually doubles by 6 months
73. A 14-year-old girl has acute lymphocytic leukemia and triples by the end of eth first year. The other choices
and is admitted. She is terminally ill. An appropriate are incorrect.
nursing action would be to:
03. The nurse provides anticipatory guidance to parents
of a 3-year-old child. Instructions should include:

St. Louis Review Center Inc. – BUTUAN CITY; Telephone No. (085) 342 – 2339 35
a. To restrain the child in the car seat facing rear in detachment. The child becomes interested in the
the back seat of the car environment, especially the caregivers. If the parents
b. The use of syrup of ipecac for accidental return, the child ignores them.
poisonings
c. Drug and alcohol education 07. A teenager refuses to wear the clothes his mother
d. The proper use of sports equipment bought for him. He states he wants to look like the other
(Answer: B) Rationale: Nurses are instrumental in kids at school and wear clothes like they wear. The
teaching parents how to make the toddler’s environment nurse explains this behavior is an example of teenage
safe by providing instructions about keeping syrup of rebellion related to internal conflicts of:
ipecac available, having the Poison Control Center a. Autonomy vs. shame and doubt
number close to the phone, using child-resistant b. Trust vs. mistrust
containers and cupboard safety closures, and keeping c. Identity vs. role confusion
medicines and other poisonous materials locked away. d. Initiative vs. inferiority
Infants are to be restrained in rear-facing car seat, (Answer: C) Rationale: Erikson’s theory of psychosocial
school-age children should be taught the proper use of development states that the child is faced with conflicts
sports equipment, and adolescents should be provided that need to be resolved. Erickson identifies stages of
education regarding drug and alcohol abuse. personality development. Identity vs. role confusion (12
to 19 years) is a period when adolescents search for
04. A school nurse prepares a lecture on puberty for 5 th answers regarding their future. During this time, the child
and 6th grade girls. She asks the group, “What is the first rejects the identity presented by his parents and
sign of puberty?” A student correctly replies: attempts to create his own identity. Identity is often
a. “The appearance of breast buds.” based on peers. Positive outcomes result in optimism
b. “An increase in energy and appetite.” and confidence. Negative outcomes result in sense of
c. “The occurrence of the first menarche.” purposelessness or deviance.
d. “Appearance of body odor.”
(Answer: A) Rationale: Puberty is a process that brings 08. In providing her 8-month-old child’s medical history,
about the development of secondary sex characteristics, the mother states the child has received on MMR
which begin with the appearance of breast buds at 9 to vaccine. The nurse taking the history should:
11 years followed by the growth of pubic hair. Menarche a. Ask the mother if the child has received the
follows approximately 1 year later. Body odor may result MMR booster
later because of an increase in secretions from the b. Plan to administer the MMR booster
apocrine glands. c. Explain that one MMR vaccine is all that is
required
05. The mother discusses with the nurse that her toddler d. Plan to administer another MMR vaccine after
asks every night for a bedtime story. The mother asks the child is 1 year old
why the child does this. The nurse would explain that (Answer: D) Rationale: This mother may have been
this behavior demonstrates: mistaken about the vaccine. Maternal antibodies
a. Ritualism interfere with the vaccine when it is given before 12
b. Object permanence months of age. Even if the child has had the vaccine, it
c. Dependency will need to be repeated. The first measles-mumps-
d. Conservation rubella (MMR) should be administered to the child
(Answer: A) Rationale: The toddler insists on sameness between the ages 12 to 15 months. The second is given
(such as a nightly bedtime story). Ritualism allows the at age 4 to 6 years or 11 to 12 years. Because of
toddler to have a sense of control and to feel more outbreaks of measles in preschoolers, school-age
secure and confident. The child may experience distress children, and college students in the 1980s, a second
if this routine is not followed. Object permanence is dose was added to there recommended childhood
when the infant develops an awareness that objects immunization schedule. The MMR is a vaccine that is
continue to exist when they are out of sight. Dependency not given to infants younger than 12 months old.
is the need for a caregiver (parent) to provide total care
for another (infant). The school-age child masters the 09. The mother of a 5-year-old expresses concern about
concept of conservation; learns that certain properties of her child who believes that “Grandma is still alive” 3
objects do not change simply because their form or months after the grandmother’s death. The nurse
appearance has changed. explains that:
a. Magical thinking often accounts for a
06. Whenever the parents of a 10-month-old leave their preschooler who believes that dead people will
hospitalized child for short periods, he begins to cry and come back
scream. The nurse explains that this behavior b. There is a need for psychological counseling for
demonstrates that the child: this child and family
a. Needs to remain with his parents at all times c. This is a form of regression exhibited by the
b. Is experiencing separation anxiety preschooler
c. Is experiencing discomfort d. The child is in denial regarding Grandma’s death
d. Is extremely spoiled (Answer: A) Rationale: The preschooler believes that
(Answer: B) Rationale: Infants and toddlers between the death is reversible. Their magical thinking and
ages of 6 months and 30 months experience separation egocentricity often results in their belief that the
anxiety. There are three stages of separation anxiety. deceased will come back to life. Preschoolers also often
The child who demonstrates crying and rejecting anyone will blame themselves for the death of another.
other than the parent is in protest, the first stage of
separation anxiety. This behavior does not exhibit 10. Hospitalization of a child results in disturbance of the
spoiling or any indication of discomfort. The second dynamics in family life. The most appropriate nursing
stage is despair. The child expresses hopelessness, diagnosis is:
appears quiet, and is withdrawn. The third stage is a. Diversional activity deficit related to separations

St. Louis Review Center Inc. – BUTUAN CITY; Telephone No. (085) 342 – 2339 36
from siblings and peers the use of common labels established by NANDA.
b. Sleep pattern disturbance related to unfamiliar Nursing diagnoses are based on data collected by the
surroundings nurse but are not related to disease etiology or
c. Altered family processes related to judgments of the overall health status of a client.
hospitalization
d. Ineffective individual coping related to 15. Screening for strabismus and amblyopia should be
procedures part of the physical assessment of which children?
(Answer: C) Rationale: Identification of nursing a. All children under 18
diagnoses that apply to the specific problem(s) of the b. Infants
child and family is an essential step of the nursing c. Preschool children
process. Family-centered care addresses the needs of d. Schoolage children
the family members, including the child’s siblings. The (Answer: C) Rationale: Strabismus is detected with the
primary goals are to maintain the relationship with the cover-uncover test that can first be reliably administered
child and siblings during the period of separation while to children over the age of 2. It is important to detect the
hospitalized and avoid boredom and distress for the problem early to prevent amblyopia. By school age,
hospitalized child. vision loss would have occurred.

11. When using the otoscope to examine the ears of a 2- 16. In infants, a positive Babinski reflex is:
year-old child, the nurse should: a. An indication of a neurological problem
a. Pull the pinna up and back b. Dorsiflexion of the toes
b. Pull the pinna down and back c. Fanning of the toes
c. Hold the pinna gently but firmly in its normal d. Withdrawing the foot from the stimulus
position (Answer: C) Rationale: A positive Babinski in infants is a
d. Hold the pinna against the skull fanning of the toes when a stimulus is applied to the foot
(Answer: B) Rationale: The ear canal in infants and along the lateral edge and across the ball. The response
young children is shorter, wider, and more horizontally disappears by about age 2.
positioned than in older children. To adequately examine
the tympanic membrane in young children the pinna 17. The review of systems part of the health history is
must be pulled back and down. best described as:
a. The description of the health problem in the
12. To assess the height of an 18-month-old child who is informant’s words
brought to the clinic for routine examination, the nurse b. The objective data recorded by the nurse
should: c. The evaluation of the past and present health of
a. Measure arm span to estimate adult height each body system
b. Use a tape measure d. A general statement about the overall health of
c. Use a horizontal measuring board the child
d. Have the child stand on an upright scale and (Answer: C) Rationale: The review of systems is a
use the measuring arm systematic review of each body system with respect to
(Answer: C) Rationale: Children younger than 2 or 3 past and present health problems. It is designed to
should be measured lying down, preferably on a provide a basis for focusing the physical assessment on
horizontal measuring board, to get an accurate problem areas. The statement of the health problem is
assessment of height. A tape measure would be used to the reason for seeking care, sometimes called the chief
measure head circumference. An arm-span measure is complaint. Objective data is gathered during the physical
not an appropriate estimation of adult height. assessment and from laboratory data.

13. At what age is it appropriate to change the sequence 18. The nurse would perform abdominal percussion to
of the examination of the child from that of chest and assess for:
thorax first to head-to-toe? a. Tenderness
a. Infant b. Inflammation
b. Toddler c. Density of tissues and organs
c. Preschool child d. Size and placement of liver
d. Schoolage child (Answer: C) Rationale: Percussion produces sounds of
(Answer: D) Rationale: The Schoolage years are the first varying loudness and pitch depending on the organs and
time a child is able to reliably cooperate with the tissue density. The nurse assesses the liver with
examiner and not squirm, talk, or otherwise interrupt the palpation and percussion, but not for placement.
exam. In younger children, it is essential to begin with Inflammation is assessed with inspection, and
the chest and thorax because the child needs to be quiet tenderness is assessed with palpation.
and at rest.
19. When assessing a 4-year-old child with a persistent
14. The best description of a nursing diagnosis is: cough, the nurse would assess respirations by observing
a. A process used to evaluate the etiology of a which muscle group?
disease a. Thoracic
b. A nursing judgment about the health of an b. Abdominal
individual c. Accessory
c. A problem-oriented description of an actual or d. Intercostal
potential health problem (Answer: B) Rationale: Infants and young children use
d. An efficient basis for communicating client data the diaphragm and abdominal muscles for respiration, so
among nurses the nurse would watch the rise and fall of the abdomen
(Answer: C) Rationale: A nursing diagnosis is a to count respirations. Use of accessory or intercostal
statement of an actual or potential problem that can be muscles may be observed in respiratory distress.
resolved or changed by nursing interventions. It involves

St. Louis Review Center Inc. – BUTUAN CITY; Telephone No. (085) 342 – 2339 37
20. When assessing the fontanels of a 6-week-old infant, options are incorrect.
how soon does the nurse expect the posterior fontanel to
close? 25. During a day-surgery hospitalization experience for
a. By 3 months tonsillectomy, a 3-year-old child will most likely be fearful
b. By 6 months of:
c. By 12 months a. Intrusive procedures
d. By 18 months b. Perceived abandonment
(Answer: A) Rationale: The posterior fontanel closes by c. Premature death
3 months of age. The anterior fontanel closes by 18 d. Unfamiliar caregivers
months. (Answer: A) Rationale: One of the greatest fears of
preschoolers is fear of mutilation. Other options are not
21. A client is to receive eye drops that are ordered to be developmentally appropriate responses for a
given “OS”. The nurse would administer the eye drops: preschooler.
a. To the left eye
b. To the right eye 26. The nurse is performing an assessment of a 14-
c. In both eyes month-old toddler admitted to the day-surgery unit for
d. In alternating eyes bilateral myringotomy and placement of tympanostomy
(Answer: A) Rationale: OS is the abbreviation for left tubes. How should the nurse obtain the child’s
eye. OD is the abbreviation for the right eye. OU is the temperature?
abbreviation for both eyes. a. The nurse should use a tympanic thermometer
with disposable speculum
22. The nurse is caring for a 6-year-old child who just b. The nurse should use an oral thermometer with
returned to the day-surgery recovery area from surgery disposable plastic sheath
following a tonsillectomy, adenoidectomy, and bilateral c. The nurse should use a rectal thermometer with
myringotomy with insertion of tympanostomy tubes. disposable plastic sheath
Which assessment data would indicate that the child is d. The nurse should use a temperature strip placed
experiencing active, uncontrolled bleeding at the on the child’s forehead
operative site? (Answer: A) Rationale: The tympanic method is
a. Tachycardia, hypertension, hemoptysis preferred. It is quick, accurate, and convenient. Oral
b. Bradycardia, hypotension, increased swallowing temperature can be obtained on a cooperative child
c. Tachycardia, hypotension, decreased aged 3 and older. A rectal temperature is obtained as a
swallowing last resort, when other methods are not possible.
d. Tachycardia, hypotension, increased swallowing
(Answer: D) Rationale: The nurse observes increases 27. Which of the following care measures is indicated in
swallowing. Tachycardia and hypotension are late signs teaching home care of a child with bilateral bacterial
of significant blood loss. The other options are incorrect. conjunctivitis?
a. Use of warm, moist, disposable compresses to
23. The nurse is planning postoperative care for a remove crusting
pediatric client following tonsillectomy. Nursing b. Use of oral antihistamine medication to relieve
considerations include which of the following? eye itching
a. A child’s behavior response to pain is affected c. Use of ophthalmic corticosteroids to decrease
by age and developmental level inflammatory response
b. Recovery from a painful procedure occurs at a d. Use of topical anesthetics applied to relieve
faster rate in children as compared to adults discomfort
c. Opioid analgesic use in children is dangerous (Answer: A) Rationale: Crusting of dried exudates is
because of increased risk of addiction and common with bacterial conjunctivitis. Other options are
respiratory depression not indicated in the management of bacterial
d. The immaturity of the nervous system in young conjunctivitis.
children provides them with an increased pain
threshold 28. The parent of an infant diagnosed with viral
(Answer: A) Rationale: Option 1 is the only true nasapharyngitis should be taught to notify the health
statement. Infants are less able to communicate their care provider:
feelings than an older child and usually demonstrate a. Of increased fussiness
restlessness and crying behaviors. Adolescents are able b. If the infant develops a cough
to describe their pain sensations. c. Of temperature above 98.6F
d. If the infant develops signs of ear infection
24. The nurse is beginning an otoscopic examination of (Answer: D) Rationale: Options 1 to 3 are expected
the ear of a 2-year-old child. The child cries, kicks, and symptoms of viral pharyngitis in infants. Symptoms of
pulls away from the nurse. How should the nurse ear infection should be reported to the health care
proceed? provider.
a. Explain to the child why the ear must be
examined 29. Decongestant nasal drops are prescribed for an
b. Postpone the examination until the next clinic infant with nasapharyngitis. Instructions for administering
visit in one year the drops include which one of the following:
c. State, “I though you were going to be grown up a. Do not use the drops or dropper for any other
for me today.” family member
d. Get assistance to restrain the child to proceed b. Save any remaining medication for the next time
with the exam the child is congested
(Answer: D) Rationale: Uncooperative pediatric clients c. Administer the drops frequently until the nasal
may need to be restrained long enough to accomplish congestion subsides
the assessment or procedure that is necessary. Other d. Insert the dropper tip as far into the infant’s nose

St. Louis Review Center Inc. – BUTUAN CITY; Telephone No. (085) 342 – 2339 38
as is possible breaths.”
(Answer: A) Rationale: Eliminating contact or sharing of c. “I always start with the meter reading about
items with the infected person can reduce the potential halfway up. That way I don’t waste any breath.”
spread of infection to other family members. The other d. “If I use my peak flow meter every day, I will not
options are incorrect. have an asthma attack.”
(Answer: A) Rationale: Peak expiratory flow readings
30. The nurse teaches the family of a toddler with over time indicate the child’s respiratory ability when she
streptococcal pharyngitis the importance of finishing the is well. Readings of 50 percent below “personal best”
full course of oral antibiotic therapy. The nurse explains indicate an asthma episode is imminent. It does not
that a potential complication of untreated streptococcal prevent an attack.
infection is:
a. Otitis media 35. A child with cystic fibrosis is hospitalized for a
b. Diabetes insipidus respiratory infection. Which documentation in the chart
c. Nephritic syndrome would indicate the need for counseling regarding
d. Acute rheumatic fever nutrition and gastrointestinal complications?
(Answer: D) Rationale: Rheumatic fever can follow an a. Frothy, foul-smelling stools
infection of certain strains of group A betahemolytic b. Weight unchanged from yesterday
streptococci. Other options are incorrect. c. Consumed 80 percent of breakfast
d. Eats three snacks every day
31. The mother of an infant diagnosed with bronchiolitis (Answer: A) Rationale: Frothy, foul-smelling stools
asks the nurse what causes this disease. The nurse’s reflects malabsorption and indicate that pancreatic
response would be based on the knowledge that the enzymes are not being consumed or dosages may need
majority of infections that cause bronchiolitis are a result adjustment. Maintenance of weight and consuming
of: meals and snacks are positive nutrition goals for children
a. Ribavirin with cystic fibrosis.
b. Mycoplasma pneumoniae
c. Respiratory syncytial virus (RSV) 36. An adolescent was diagnosed with cystic fibrosis as
d. Hemophilus influezae an infant. At this time, the adolescent will need additional
(Answer: C) Rationale: At least one-half of all cases of teaching related to:
bronchiolitis are attributed to RSV. The other responses a. Obtaining a sweat chloride test
are incorrect. b. The effect of pancreatic enzymes on the sex
hormones
32. A child is brought to the Emergency Department with c. Weight reduction diet
suspected epiglottitis. Which nursing intervention would d. Reproductive ability
be considered unsafe? (Answer: D) Rationale: The developmental task of
a. Allowing the child to remain in the position of adolescence is to set future goals, including marriage
choice and family. Men are usually sterile, and women may
b. Placing intubation equipment at the bedside have decreased fertility as thick cervical mucus
c. Encouraging parents to comfort the child interferes with mobility of sperm. The difference between
d. Examining the throat sterility and impotence should also be addressed.
(Answer: D) Rationale: Any manipulation of the tongue
or throat may stimulate the gag reflex and cause 37. The parents of a child with cystic fibrosis inform the
complete obstruction. Emergency intubation equipment nurse that they will be unable to perform postural
should be readily available before any examination of drainage at home because their bed does not recline like
the throat is attempted. the hospital bed. The nurse’s response is based on an
understanding that:
33. An 18-month-old child is seen in the Emergency a. Postural drainage is essential to mobilize
Department with respiratory distress and is admitted with secretions in the airways so they can be
a diagnosis of pneumonia. Following the initial workup, coughed out
the baby is still short of breath but is rubbing his eyes as b. Postural drainage is not necessary as long as
if he is sleepy. The mother wants to lay the baby down the child takes his pulmozyme to decrease the
for his nap. The infant refuses to lie down. The nurse viscosity of the mucus
would suggest: c. Postural drainage does not influence the
a. Rocking the baby until he is asleep and then lay pulmonary status of a child with cystic fibrosis
him down d. The parents can be referred to the Cystic
b. The mother hold him in her arms while he sleeps Fibrosis Foundation for a flexible bed
c. The mother allow the baby to sleep in an upright (Answer: A) Rationale: The removal of thick, pulmonary
position secretions is critical to the maintenance of adequate lung
d. A sleeping pill to help the baby rest function and prevention of infection. Daily chest
(Answer: C) Rationale: The child’s respiratory distress physiotherapy, including postural drainage, is required
makes lying down difficult. The child will breath more and must be consistently performed. Playground
easily in semi-to high-Fowler’s position. activities such as monkey bars, trapeze bar,
somersaults, and headstands can accomplish the
34. Which statement by an 8-year-old child with asthma purposes of postural drainage.
indicates that she understands the use of a peak
expiratory flow meter? 38. An 11-month-old child is being discharged home for
a. “My peak flow meter can tell me if an asthma the first time after being diagnosed with
episode might be coming, even though I might bronchopulmonary dysplasia (BPD). She will require
still be feeling okay.” home oxygen therapy. Which statement by the mother
b. “When I do my peak flow, it works best if I do indicates that discharge teaching is incomplete?
three breaths without pausing in between a. “We will not allow any smoking at our home.”

St. Louis Review Center Inc. – BUTUAN CITY; Telephone No. (085) 342 – 2339 39
b. “We have several fire extinguishers, and we c. Chorea, muscle weakness, and decreased
know how to use them.” erythrocyte sedimentation rate
c. “Her brother will blow out the birthday candles at d. Erythema, polyarthritis, and elevated
her party.” antistreptolysin-O (ASO) titer
d. “We will return to the hospital if she seems (Answer: Jones Criteria is a protocol to assist in
irritable and won’t play.” identifying rheumatic fever. It consists of major
(Answer: C) Rationale: There should be no open flames symptoms, minor symptoms, and supporting evidence.
when oxygen is in use; oxygen enhances combustion Erythema, polyarthritis, and elevated ASO titer are
and is a fire hazard. among the major and minor symptoms and supporting
evidence.
39. The nurse is teaching home tracheostomy care to
the parents of a toddler. What information is essential to 44. A toddler with Kawasaki’s disease is ordered to
include? receive aspirin therapy. Typical administration of aspirin
a. The importance of changing the tracheostomy for Kawasaki’s disease would include which of the
every day following principles?
b. How to recognize signs of infection and a. High doses of aspirin should be given while
obstruction fever is high
c. How to remove the tracheostomy so the child b. Length of aspirin therapy is related to child’s
can talk response
d. Teaching the child to keep large objects away c. Aspirin dose increases after fever is gone
from the tube d. Aspirin dosage is unrelated to platelet count
(Answer: B) Rationale: Accumulating mucopurulent (Answer: B) Rationale: Aspirin therapy is ordered 80 to
secretions may provide a medium for bacterial growth or 100mg/kg/day until fever drops. Then aspirin is
can obstruct the lumen of the tube. Suctioning is another continued at 10mg/kg/day until platelet count drops.
risk for introduction of bacteria. Early recognition of signs Aspirin is used as an antipyretic and anti-agglutination
of infection is important. drug.

40. A child with a respiratory infection is scheduled to 45. A newborn with possible hypoplastic left heart
have a sweat test. The mother asks the purpose of this disease is to be admitted to the nursing unit. Which drug
diagnostic test. The nurse’s response would be based should be available for use?
on the knowledge that the rest: a. Digitoxin (Crystodigin)
a. Determines if the child is dehydrated b. Prostaglandin E1 (Prostin VR)
b. Assess if the sweat glands are functioning c. Morphine
c. Identifies the infectious organism d. Testosterone (Andro)
d. Establishes a diagnosis of cystic fibrosis (Answer: B) Rationale: Prostaglandin E1 prevents
(Answer: D) Rationale: Children with cystic fibrosis have closure of ductus arteriosus and thereby allows for
elevated chloride concentrations of sweat because of the mixing of oxygenated and unoxygenated blood until
dysfunction of the exocrine glands. palliative surgery can be done.

41. A toddler has been diagnosed with an acyanotic 46. A 2-year-old child is being discharged home and will
cardiac defect. Which assessment data would most have palliative surgery for tetralogy of Fallot at a later
likely indicate congestive heart failure? date. The mother wants to know about how much
a. Heart murmur physical activity she can allow for the child. The nurse’s
b. Cardiac volume overload best answer is:
c. Anuria a. “Allow the child to regulate her activity.”
d. Excitability b. “Keep her on complete bedrest.”
(Answer: B) Rationale: Congestive heart failure may c. “Limit her activities to a few hours.”
occur when the amount of blood passing from left to right d. “Keep the child from crying.”
side of the heart overloads the pulmonary system. (Answer: A) Rationale: Although a child requiring surgery
for tetralogy of Fallot may have a need for additional
42. An infant who ahs a congenital heart defect comes services, such as supplemental oxygen at home, the
into the clinic with complaints of irritability, pallor, and child should be able to play and move about in the
increased cyanosis that began quickly over the last 30 environmental needs.
minutes. As the nurse assesses the infant, the parent
asks why the child’s color is bluish. The best response 47. After a pediatric client has a cardiac catheterization,
by the nurse is, “Skin color is: which intervention would have the highest priority in the
a. Related to the time of day.” immediate postoperative period?
b. Related to brain function.” a. Encourage intake of small amounts of fluid
c. Related to hemoglobin level and oxygen b. Teach the parents signs of congestive heart
saturation.” failure
d. Unrelated to your child’s condition.” c. Monitor the site for signs of infection
(Answer: C) Rationale: The hemoglobin molecule carries d. Apply direct pressure to entry site for 15 minutes
oxygen. The oxyhemoglobin gives the skin the pink (Answer: D) Rationale: Direct pressure on wound site
color. In the absence of oxyhemoglobin, the skin color helps to form clot and reduce bleeding. Hemorrhage can
darkens. be life threatening in the immediate postoperative period.

43. A client is admitted with a diagnosis of “rule out 48. A child is being seen in the ambulatory clinic for a
rheumatic fever.” Based on Jones Criteria, the nurse sore throat diagnosed as caused by group A
assesses for: betahemolytic streptococcus. The nurse provides care
a. Polyarthritis and dental caries with the understanding that the risk of developing
b. Fever, headache, and low red blood cell count rheumatic fever is greatest:

St. Louis Review Center Inc. – BUTUAN CITY; Telephone No. (085) 342 – 2339 40
a. Two weeks later mask is of little benefit. Overall, the child must be
b. Prior to administering an antibiotic protected form injury from the environment.
c. Once the child has begun antibiotic therapy
d. With the onset of the strep infection 54. The nurse conducts Denver II screenings at a
(Answer: A) Rationale: Rheumatic fever often follows 2 community center for infants and young children. The
weeks after a streptococcal infection regardless of nurse explains that the purpose of these screenings is
treatment. to:
a. Reverse degenerative processes that have
49. Which evaluation would indicate a toxic dose of occurred
digoxin? b. Recognize early infection in order to prevent
a. Tachycardia and dysrhythmia spread to individuals in close contact with the
b. Headache and diarrhea child
c. Bradycardia and nausea and vomiting c. Recognize a disorder early so strategies can be
d. Tinnitus and nuchal rigidity developed to promote optimum development
(Answer: C) Rationale: Signs of digoxin toxicity include d. Measure intelligence and readiness for school
bradycardia, arrhythmia, nausea, vomiting, anorexia, (Answer: C) Rationale: The Denver II is a developmental
dizziness, headache, weakness and fatigue. screening test. The primary reason for doing
developmental screenings to find children who might be
50. The nurse is developing a discharge-teaching plan at risk and refer them for further assessment so that
for the family of a child with Kawasaki’s disease. Which possible delays can be identified and appropriate early
of the following is the first priority? intervention initiated. The Denver II is not a measure of
a. Teaching parents to administer aspirin and intelligence. It has nothing to do with infection control nor
watch for side effects is it an intervention to correct degenerative processes.
b. Recommending the child avoid contact sports
c. Monitoring the child’s temperature and notifying 55. The nurse is providing client education for a family
the doctor if it is over 98.6F whose child has cerebral palsy and is receiving baclofen
d. Establishing home schooling for 6 months epidural therapy to control spasticity. Which of the
(Answer: A) Rationale: Aspirin is an anti-inflammatory following is most important for the nurse to include in the
and antipyretic. The child may experience bleeding and discussion?
G.I. upset as side effects. a. The drug acts to inhibit the neurotransmitter
GABA
51. A 3-month-old infant has been admitted with a b. The child should be able to run with normal gait
diagnosis of encephalitis. The first nursing priority would after insertion of the pump
be to assess: c. Parents must bring the child back to the clinic on
a. Pupillary reaction a regular basis to have more medicine added to
b. Level of consciousness the pump
c. Ability to maintain airway d. Parents can be taught to regulate the dosage on
d. Blood glucose level a sliding scale
(Answer: C) Rationale: While all other choices are (Answer: C) Rationale: This therapy involves an
important to monitor, the priority in assessing any implanted pump that must be accessed through the skin
critically ill child follows the AABC rule – airway, to refill the pump. Parents are not taught to refill the
breathing, and circulation. pump. Baclofen does inhibit the neurotransmitter GABA;
however, it is not the essential data to be shared with the
52. The nurse places the young child scheduled for a parents. Promising the parents that the child will be able
lumbar puncture in a side-lying position with head flexed to run with normal gait offers false hopes. The implanted
and knees drawn up to the chest. The mother asks why ump’s dosage cannot be changed without special
the child has to be positioned this way. The nurse equipment.
explains the rationale for the positioning is that:
a. Pain is decreased through this comfort measure 56. A 10-year-old client presents with weakness in legs
b. Injury to the spinal cord is prevented and history of the flu. The medical diagnosis is Guillain-
c. Access to the spinal fluid is facilitated Barre syndrome. It would be imperative that the
d. Restraint is needed to prevent unnecessary physician be informed if the nurse observes:
movement a. Weak muscle tone in feet
(Answer: C) Rationale: This position opens the b. Weak muscle tone in legs
intervertebral spaces and allows easier access to spinal c. Increasing hoarseness
canal. The position does not decrease pain or help to d. Tingling in the hands
restrain the child. All lumbar punctures are done below (Answer: C) Rationale: Gurllain-Barre is an ascending
L4 (the level of the spinal nerves) so injury to the spinal paralysis. While the child will have increasingly less
cord is always avoided. muscle tone in extremities, the hoarseness could
indicate involvement in the muscles of respiration.
53. An 18-month-old child is observed having a seizure. Serious concern is raised when the respiratory muscles
The nurse notes that the child’s jaws are clamped. The are affected. Sometimes mechanical ventilation is
priority nursing responsibility at this time would be: indicated. Tingling is a common sign of Guillain-Barre
a. Start oxygen via mask and not related to respiratory distress.
b. Insert padded tongue blade
c. Restrain child to prevent injury to soft tissue 57. The nurse is providing discharge instructional for a
d. Protect the child from harm from the child who has suffered a head injury within the last four
environment hours. The nurse will recognize the need for additional
(Answer: D) Rationale: Never forcibly restrain a child teaching when the mother states:
during a seizure or insert a padded tongue blade; both a. “I will call my doctor immediately if my child
are more likely to add trauma than prevent. Oxygen via starts vomiting.”

St. Louis Review Center Inc. – BUTUAN CITY; Telephone No. (085) 342 – 2339 41
b. “I won’t give my child anything stronger than Pyelonephritis with resulting kidney scarring and
Tylenol for headache.” damage. Early diagnosis and prompt antimicrobial
c. “My child should sleep for at least 8 hours therapy will prevent or minimize permanent renal
without arousing after we get home.” damage.
d. “I recognize that continued amnesia about the
injury is not uncommon.” 62. When reviewing a urinalysis report of a client with
(Answer: C) Rationale: Discharge instructions will acute glomerulonephritis, the nurse would expect to
include the necessity of waking the child to check for note:
neuro status throughout the night. Vomiting could be a a. Decreased creatinine clearance
sign of increasing intracranial pressure and should be b. Decreased specific gravity
reported. Narcotics are not given after a head injury. c. Proteinuria
Amnesia for the events surrounding the injury may be d. Decreased erythrocyte sedimentation rate (ESR)
permanent. It is not a sign of increasing intracranial (Answer: C) Rationale: Proteinuria (presence of protein
pressure. in urine) is a prime manifestation of acute
glomerulonephritis. The other options are inconsistent
58. The Glasgow Coma Scale is used to measure with this diagnosis.
neurological functioning. Which of the following criteria
would indicate the lowest level of functioning for an 63. While a child is receiving prednisone (Deltasone) for
infant or young child? treatment of nephritic syndrome, it is important for the
a. Confused nurse to assess the child for:
b. Irritable, cries a. Infection
c. Eyes open only to pain b. Urinary retention
d. No response to painful stimuli c. Easy bruising
(Answer: D) Rationale: No eye opening, no verbal d. Hypoglycemia
response, and no motor response are the lowest criteria (Answer: A) Rationale: Prednisone is synthetic
on the scale. Confusion is a criterion applicable only for corticosteroids that depresses the immune response and
the older child and adult but is comparable to “irritable increases susceptibility to infection. Steroids mask
and cries” for the infant (which is a 4 out of 5 on the infection; therefore, the child must be assessed for
verbal response subscale). “Eyes open only to pain” is subtle signs and symptoms of illness.
next to the lowest level on the eye-opening category.
64. Which of the following interventions will help obtain
59. Upon performing a physical assessment of a 7- accurate urinalysis data?
month-old child, the nurse notes an abnormal finding a. Force fluids to 1000mL prior to specimen
that could suggest cerebral palsy. The finding suggestive collection
of cerebral palsy is that the child has: b. Cleanse the specimen container with povidon-
a. No head lag when pulled to a sitting position iodine (Betadine) prior to collecting the
b. No Moro or startle reflex specimen
c. Positive tonic neck reflex c. Allow the urine to cool to room temperature
d. Absence of tongue extrusion before taking it to the lab
(Answer: C) Rationale: The Moro or startle, tongue d. Provide client/parent education for specimen
extrusion, and tonic neck reflex are all neonatal reflexes collection before the specimen is obtained
that should have disappeared by this child’s age. Lack of (Answer: D) Rationale: Specimens collected utilizing
head lag indicates good motor development. A proper technique will minimize contamination of the urine
developmental delay or the presence of a neonatal reflex sample ensuring accurate urinalysis results. It is
is some of the earliest clues to cerebral palsy. unnecessary to force fluids prior to specimen collection.
The specimen container is not cleansed, although the
60. A 4-year-old child is being evaluated for urinary meatus is. The specimen should be sent to the
hydrocephalus. An early indication of hydrocephalus in lab immediately after collection to prevent urine
this child would be: degradation.
a. Bulging fontanels
b. Rapid enlargement of the head 65. The parents of a child diagnosed with upper urinary
c. Shrill, high-pitched cry tract infection (UTI) ask the nurse why the child needs a
d. Early morning headache daily weight. In formulating a response, the nurse
(Answer: D) Rationale: All of the above are symptoms of includes that it is important because a daily weight will:
increased ICP or hydrocephalus. Head enlargement and a. Determine if the child’s caloric intake is
bulging fontanels would not be seen in the child after adequate
closure of the sutures (12 to 18 months). Shrill, high- b. Indicate the need for dietary restrictions of
pitched cry is a late-stage symptom of children. sodium and potassium
Headache and vomiting on arising would be an early c. Keep track of possible loss or gain of fluid
symptom in an older child. retained in body tissues
d. Track the amount of fluid ingested orally each
61. The most important nursing activity in managing a day
young child diagnosed with urinary tract infection (UTI) is (Answer: C) Rationale: With infectious or inflammatory
to: processes of the upper urinary tract, the kidneys’ ability
a. Provide adequate nutrition to prevent to filter and reabsorb salt and water is altered, resulting
dehydration in edema. Weights can be an easy and effective
b. Prevent enuresis measure to determine fluid loads.
c. Administer ordered antibiotics on schedule
d. Restrict fluids to provide kidney rest 66. A child has been diagnosed with acute renal failure
(Answer: C) Rationale: Urinary tract infections are secondary to an infectious organism. The nurse would
ascending in nature; an untreated UTI can lead to acute question the medical order for:

St. Louis Review Center Inc. – BUTUAN CITY; Telephone No. (085) 342 – 2339 42
a. Aqueous penicillin diabetes mellitus. As you are teaching him about his
b. Gentamicin (Garamycin) insulin infections, he asks you why he can’t take the
c. Antihypertensive diabetic pills that his aunt takes. What would be the best
d. Corticosteroids response?
(Answer: B) Rationale: Gentamicin is an aminoglycoside a. “You will be able to take the pills once you reach
antibiotic that is nephrotoxic. Nephrotoxic drugs should adult height.”
be avoided in a child with acute renal failure. The other b. “You have a different type of diabetes, and the
options do not represent drug groups that are particularly pill won’t work.”
nephrotoxic. c. “We have to test you to see if you can take the
diabetic pills.”
67. The newborn has been diagnosed with d. “You might be able to switch between taking the
cryptorchidism. The physician has ordered human pills and insulin.”
chorionic gonadotropin (HCG) to be administered to the (Answer: B) Rationale: Children with Type 1 diabetes
baby. The mother asks the nurse why the baby is must take insulin because they have a total absence of
receiving this drug. The nurse’s best explanation would secretion of insulin from their pancreas. Clients with
be the drug will: Type 2 diabetes mellitus may produce some insulin so
a. Maintain an adequate temperature around the they can take the oral hypoglycemics.
testes
b. Prevent infections in the undescended testes 72. When instructing a client on the best way to check
c. Prevent the development of cancer the control of diabetes, you would say:
d. Promote descent of the testes a. “Check your rune glucose three times a week.”
(Answer: D) Rationale: HCG is given to induce the b. “Check the glycosolated hemoglobin every 3
descent of testes if testes have not descended during months only.”
the first year of life. The other reasons listed are c. “Check the blood glucose at least twice a day
incorrect rationales. and the glycosolated hemoglobin every 3
months.”
68. The nurse admits children with the following d. “Don’t check anything as long as you feel well.”
diseases to the unit. Which disease places the child at (Answer: C) Rationale: Checking the blood glucose at
risk for the development of acute renal failure (ARF)? least twice a day prevents sustained levels of either high
a. Leukemia or low glucose readings. The glycosolated hemoglobin
b. Cryptorchidism measures long-term control and is a very important
c. Nephrotic syndrome value.
d. Phenylketonuria
(Answer: C) Rationale: Nephrotic syndrome is an 73. A mother attends the pediatric clinic with her 10-
inflammatory reaction in the kidneys. The other diseases year-old daughter, who has diabetes. After completing
pose minimal risk of developing acute renal failure. the diabetic teaching, the nurse evaluates the mother’s
knowledge. Which statement by the mother indicates a
69. A child has recurrent nephritic syndrome. The satisfactory understanding of diabetes?
mother reports to the nurse that she is overwhelmed with a. “I worry about my daughter maintaining control
the care of her child. After the nurse discusses options since children with diabetes have more
with the mother, which statement by the mother complications than adults do.”
indicates continued coping difficulties? b. “My daughter should drink vanilla milkshakes to
a. “I joined a support group like you suggested. I maintain a high caloric intake.”
hope it does some good.” c. “Complications from diabetes could include
b. “I’m going to ask my mother-in-law to come on a cataracts and kidney stones.”
regular basis to allow m e an afternoon out.” d. “My child won’t need a midafternoon snack since
c. “My husband has agreed to help me manage my she takes a gym class in the afternoon.”
son’s medication.” (Answer: A) Rationale: Long-term complications of Type
d. “We’re going to skip his dietary restrictions one 1 diabetes may include retinopathy, heart disease, renal
day a week to allow us both some relaxation.” failure, and peripheral vascular disease. These
(Answer: D) Rationale: The parents must understand the complications can affect children and adults. The longer
need for compliance with medical orders to promote the the child lives with diabetes, the greater the likelihood of
child’s health. Relaxation should be accomplished complications. Exercise increases the utilization of
without harming the child. glucose, thus an afternoon snack would be very
important. Milkshakes contain concentrated
70. A child returning to the unit after an intravenous carbohydrates that should be avoided.
pyelogram (IVP) has an order to drink extra fluids. When 74. Considering a child’s developmental level in diabetic
the mother asks the purpose of these fluids, the nurse care is essential. The nurse should include which
responds that increased fluid intake will: information in teaching the parents of a recently
a. Overhydrate the child diagnosed toddler with diabetes?
b. Increase serum creatinine levels a. Allow the toddler to assist with the daily insulin
c. Make up for fluid losses from NPO status before injections
tests b. Prepare meat, vegetables, and potatoes for
d. Flush any remaining dye from the urinary tract each dinner. The toddler cannot be allowed any
(Answer: D) Rationale: The additional fluids will increase choices in food selection
urinary output, causing greater urine volume and more c. Test the toddler’s blood glucose every time he
frequent voiding, thus flushing the dye from the urinary goes outside to play
system. The other options do not describe the correct d. Allow the toddler to assist with cleaning off his
rationale for this intervention. finger before blood glucose monitoring
(The toddler needs to feel some control. Cleaning off his
71. A 12-year-old client was just diagnosed with Type 1 fingers with alcohol, under supervision, will allow some

St. Louis Review Center Inc. – BUTUAN CITY; Telephone No. (085) 342 – 2339 43
control. Another possibility to promote the toddler’s 79. A child demonstrates a sudden onset of
sense of control would be to allow the toddler to choose thyrotoxicosis, exhibited by irritability and restlessness
food selections from options offered. with accompanying hypertension and tachycardia.
Besides antithyroid therapy, what other drug therapy
75. A 2-month-old infant arrives at the pediatric clinic. does the nurse prepare to administer?
Upon assessment, the baby exhibits the following a. Antacids
characteristics. Which characteristics would be related to b. Beta-andrenergic blocker
the diagnosis of congenital hypothyroidism? c. Muscle relaxants
a. Open fontanels d. Cardiac glycoside
b. Protruding tongue (Answer: B) Rationale: A beta-adrenergic blocking agent
c. Tachycardia provides relief from adrenergic hyper responsiveness. It
d. Hypertonia is usually needed for 2 to 3 weeks along with anti-thyroid
(Answer: B) Rationale: Most babies with congenital hormone therapy.
hypothyroidism exhibit bradycardia, protruding tongue,
and hypotonia. Open fontanels are normal for a 2- 80. Four newborns have had blood drawn for the Guthrie
month-old infant. test for phenylketonuria. The nurse would question the
results of the baby:
76. A 10-year-old girl comes to the office of the school a. Whose test is performed at 48 hours of age
nurse after recess. This is the child’s first day back in b. Who was breast-fed for the 24 hours before the
school after hospitalization, where she was diagnosed test
with diabetes. The child reports she took the dose of c. Fed glucose water followed by formula for 30
insulin as instructed and that it was the same as she hours
took while hospitalized. The nurse notices that she is d. Tested immediately after birth
nervous with hand tremors present. She is pale, sweaty, (Answer: D) Rationale: The screening is done only after
and complaining of sleepiness. The school nurse would an adequate amount of protein has been ingested.
suspect: Breast milk and formula meet the requirements. The
a. Exercise-induced hypoglycemia testing is usually done at 48 hours of age.
b. Hyperglycemia caused by increased intake at
lunch 81. Parents of an unborn infant have just learned that,
c. Ketoacidosis caused by an infection based on ultrasound, their infant has clubfoot. They ask
d. The child is avoiding returning to class the nurse how clubfoot is treated. Which of the following
(Answer: A) Rationale: Exercise makes the body more treatments should the nurse discuss with the parents?
sensitive to insulin, thus metabolizing the glucose faster. a. Weekly cast changes with manipulation
While hospitalized, the child was less active. Now that b. Probable surgery on the affected limb
the child has returned to normal activity, the insulin dose c. Abduction device to keep the hip in full
is too high or more carbohydrate is required in the diet. abduction
d. Use of a Denis Browne splint to achieve
77. After being diagnosed with Grave’s disease, a correction
teenager begins taking propylthiouracil (PTU) for (Answer: A) Rationale: The initial treatment for clubfoot
treatment of the disease. What symptom would indicate begins immediately or shortly after birth and consists of
that the dose might be too high? weekly cast changes and manipulation. Surgery is
a. Weight loss completed only if nonsurgical intervention of serial
b. Polyphagia casting is not effective. A Denis Browne splint may be
c. Lethargy used to maintain correction once it is achieved.
d. Difficulty with school work Abduction devices are used for hip conditions.
(Answer: C) Rationale: Lethargy may indicate an
overdose of the drug, causing the child to exhibit signs of 82. An infant is placed in a Pavlik harness for
hypothyroidism. The other signs indicate developmental dysplasia of the hi0p. Which of the
hyperthyroidism. following statements made by a parent indicates correct
knowledge of the care of this infant?
78. A 13-year-old male client is being evaluated for a. “The straps of the harness should be placed
delayed puberty. He has had an examination with a nest to the skin.”
pediatric endocrinologist who states that the child ahs a b. “The harness should be worn for 6 hours a day.”
constitutional delay. What type of follow-up counseling c. “It will take a long time for my child to walk and
would you offer this client? craw.”
a. “All of your hormone levels are normal, so no d. “I can move my child around on a large
medication is needed at this time. If you want to skateboard.”
talk about it, I would be happy to discuss it with (Answer: D) Rationale: A child in an abduction splint
you.” needs to be kept mobile, which can be done with the use
b. “I am worried about your stature. I think you of a wagon, large skateboard, or cart. Though diapers
should get another opinion.” should be placed over the straps of a Pavlik harness, a t-
c. “Your father’s stature doesn’t matter. We just shirt should be worn under the straps of the harness.
look at your height.” The harness should be worn for 23 hours a day. The
d. “If you want testosterone shots, I will arrange for child quickly “catches up” once the device is no longer
them to be given.” worn if developmental milestones are delayed because
(Answer: A) Rationale: An adolescent client with delayed of the abduction device.
puberty may need to talk about issues of low self-
esteem. If he has a constitutional delay, puberty will 83. Which of the following nursing diagnoses takes
usually follow with time. Hormone therapy is not given highest priority for the child hospitalized with
until after the age of 14. osteogenesis imperfecta?
a. Impaired skin integrity related to cast

St. Louis Review Center Inc. – BUTUAN CITY; Telephone No. (085) 342 – 2339 44
b. Pain related to fractures return of bowel function. The pain experienced by this
c. Risk for injury related to disease state client is severe and requires intravenous medication,
d. Body image disturbance related to short stature preferably with patient-controlled analgesia (PCA).
(Answer: C) Rationale: Because of their very fragile Logrolling must be done every 2 hours, once allowed, to
bones, children with OI experience countless fractures, prevent the accumulation of secretions in the lungs.
and the prevention of injury takes highest priority in this Urinary retention is common, and an indwelling catheter
child’s care. is used id present rather than repeated straight
catheterization.
84. Which of the following would not be consistent with
common assessment findings in a child diagnosed with 89. A 3-year-old child is suspected of having Duchenne’s
an acute onset of Legg-Calve-Perthes (LCP) disease? muscular dystrophy. Which of the following assessment
a. Swelling and redness of the involved joint(s) findings by the nurse would support this diagnosis?
b. Stiffness in the morning or after rest a. A history of delayed crawling
c. Insidious limp after activities b. Inability to ambulate independently
d. Referred pain to the knee c. Difficulty climbing stairs
(Answer: A) Rationale: Swelling and redness of involved d. Gower’s sign
joints is a symptom found in juvenile arthritis, not LCP (Answer: C) Rationale: The child with Duchenne’s
disease. All of the other symptoms listed are consistent muscular dystrophy has a history of meeting early
with this diagnosis. developmental milestone. Symptoms usually begin at
around 3 years of age and include difficulty-climbing
85. A 12-year-old male is admitted to the adolescent unit stairs, running, and pedaling. As the disease progresses,
with a diagnosis of slipped capitol femoral epiphysis. the child has a difficult time ambulating on even
Which of the following activities should not be allowed surfaces, and Gower’s sign is seen. The child loses the
prior to surgical correction? ability to ambulate independently by the age of 10 to 12.
a. Ambulation with crutches, avoid bearing weight 90. A child is suspected of having osteomyelitis. Which
on the affected leg of the following blood values supports this diagnosis?
b. Sitting in a wheelchair a. Decreased white blood cell (WBC) count
c. Moving on a stretcher b. Positive blood cultures
d. Maintaining bedrest c. Increased hematocrit (Hct)
(Answer: B) Rationale: Once the diagnosis is made, the d. Increased BUN
child should not bear weight on the affected hip, as (Answer: B) Rationale: Blood studies in a child with
weight bearing can increase the amount of slippage. osteomyelitis will reveal an increased WBC count, C-
Wheelchair use should be avoided, as this also may reactive protein, and sedimentation rate. The blood
increase the amount of slippage. culture is usually positive. This disease process does not
affect the HCT or BUN.
86. Which of the following symptoms is not typical in an
adolescent with idiopathic structural scoliosis? 91. A 4-year-old child was just diagnosed with impetigo.
a. Back pain What is the most important action the nurse should take
b. Skirts that hang unevenly to make sure it does not spread?
c. Unequal shoulder heights a. Apply bacitracin
d. Uneven waist angles b. Keep it covered
(Answer: A) Rationale: Back pain is not identified as a c. Isolate the child at home
symptom of idiopathic structural scoliosis. All the other d. Teach and use good handwashing
listed symptoms are. (Answer: D) Rationale: Handwashing is always the most
important action that a nurse can take to prevent the
87. Which nursing diagnosis should take highest priority spread of infection. Merely applying ointment or covering
when working with an adolescent with scoliosis? the site does not address the spread of infection, nor
a. Body image disturbance related to treatment of does isolation of a child at home. The nurse would teach
scoliosis the family the importance of good handwashing.
b. Diversional activity deficit related to treatment of
scoliosis 92. Which of the following would be appropriate home
c. Anxiety related to outcome of treatment for care instructions for a family that has a lice infestation?
scoliosis a. Immerse combs and brushes in boiling water for
d. Fear related to treatment and unknown 30 minutes to kill lice
outcomes b. Vacuum floor and furniture to remove hair that
(Answer: A) Rationale: Treatment for scoliosis extends might have live nits
over a long period of time, during the time when a great c. Take the child’s clothing and bed linens to a dry
deal of their psychological identity is formed. Treatment cleaner for sanitation
involves a modified lifestyle and being “different” from d. Use commercial anti-lice sprays on furniture and
their peers, so issues of self-image are paramount and mattresses
should take priority. (Answer: B) Rationale: Live nits can hatch up to 8 to 10
days later, so it is important to remove them from the
88. Postoperative care of an adolescent following a environment. Soaking combs in a Lysol or anti-lice
spinal fusion for scoliosis includes: shampoo moisture will kill lice or nits. Dry cleaning is not
a. Oral analgesia for pain necessary because home washing and drying on hot
b. Logrolling every 4 hours settings will b sufficient to kill lice and nits. Use of
c. Nasogastric intubation commercial sprays is not recommended.
d. Straight catheterization every 4 hours
(Answer: C) Rationale: There is some degree of paralytic 93. A 10-year-old child sustained partial thickness burns
ileus following a spinal fusion; therefore, nasogastric to his right arm and abdomen after tossing gasoline on a
intubation is required along with frequent assessment of fire. What would the nurse expect the appearance of the

St. Louis Review Center Inc. – BUTUAN CITY; Telephone No. (085) 342 – 2339 45
burn site to be? assessment?
a. Smooth and bright red a. Level of pain
b. Bright red with numerous blisters b. Airway patency
c. White and waxy c. Psychosocial needs
d. Dark brown and firm d. Signs of infection
(Answer: B) Rationale: The characteristics appearance (Answer: B) Rationale: Because he was in close
of second-degree burns is bright red skin with blisters of proximity to the fire and tried to put it out, he is at risk of
varying sizes. A first-degree burn typically only has pink having inhaled smoke and therefore having a
or red skin. A third-degree burn may be dark in color, compromised airway. Assessing pain, psychosocial
from deep red to black. needs, or for infection occurs only after establishing
airway patency.
94. Permethrin 5% (Elimite) is prescribed for a 10-year-
old child diagnosed with scabies. What instructions 99. Intravenous morphine is ordered for a 13-year-old
should the nurse provide for the mother? boy hospitalized with major burns to 30 percent of his
a. Apply the lotion liberally from head to toe body. What is the rationale for IV morphine?
b. Wrap the child in a clean sheet after treatment a. Longer half-life
c. Leave the lotion on for 4 to 6 hours b. Predictable absorption rate
d. Apply lotion only after the child has had a bath c. Prevents ileus
and dried thoroughly d. Fewest side effects
(Answer: D) Rationale: Permethrin is applied to cool dry (Answer: B) Rationale: The predictable rate of
skin after a bath, but only from the neck down. The child absorption makes IV morphine useful in treating severe
may dress after the lotion is applied. It should be washed pain. Side effects, including ileus, are considered
off after 8 to 12 hours. A second application of often secondary to the desired effect of predictability in
prescribed for 1 week later. managing pain. Knowing the half-life is not a rationale for
use of IV morphine.
95. When assessing a child with a possible diagnosis of
facial cellulitis, the nurse will want to question the parent 100. The nurse is providing a teaching session for
about the recent history of: parents about over-the-counter treatment for head lice.
a. Otitis media Which of the following will be mentioned as appropriate
b. Cat scratch for treating this problem?
c. Sunburn a. Neosporin
d. Sinusitis b. Mafenide (Sulfamylon)
(Answer: A) Rationale: A recent history of otitis media is c. Silver sulfadiazine (Silvadene)
often present in children with facial cellulitis. Sunburn d. Permethrin (Nix)
would present as more diffuse and widespread redness. (Answer: D) Rationale: Permethrin is the over-the-
An insect or animal bite can be a cause of cellulitis, but counter treatment of choice for head lice. Other choices
in the case of a cellulitis on the face the nurse would are topical agents, but they would not be used for lice.
question a recent history of an ear infection if a bite were Option 1 would be used for infection, while options 2 and
not obvious. 3 would be used to treat burns.
101. The nurse has explained allergy-proofing the home
96. In teaching a group of school children, a nurse would to the mother of a child with dust allergies. Which
explain that lice on a child could be most easily spread statement by the mother indicates a clear understanding
by: of appropriate allergy proofing?
a. Sitting close to someone who had lice a. “I’m going to replace the cotton curtains on the
b. Sharing hats at recess window with blinds.”
c. Riding in the same car b. “The only toys allowed in his bedroom are his
d. Riding on the same bus stuffed toys.”
(Answer: B) Rationale: Lice can only be passed by direct c. “I should store his out-of-season clothes in his
contact because lice do not fly. The usual mode of bedroom.”
transmission is sharing of hats, combs, brushes, or hair d. “The mattress and box springs both need to be
ornaments. Being close to someone in a classroom, bus, enclosed in a thick plastic cover.”
or car does not presuppose direct contact with hair or (Answer: D) Rationale: Cloth items hold in dust. Only
nits that have been shed on hair. essential items should be stored in the child’s bedroom
and those should be in drawers or closets. Stuffed
97. What would be an appropriate nursing goal for a 10- animals retain dust and should be removed from the
year-old girl with eczema of the elbows, hands, and bedroom. Cotton curtains would be preferred over blinds
face? because cotton curtains can be washed frequently. Both
a. Pain will be managed the mattress and the bed should be enclosed in special
b. Spread of infection will be prevented plastic covers to eliminate a source of dust.
c. Wee-hydrated skin will be maintained
d. Dietary restriction will be maintained 102. A child is in the clinic for a prick test. Because of
(Answer: C) Rationale: Keeping the skin well hydrated the risk of anaphylaxis, the nurse has available for
will prevent the need to scratch dry skin that can lead to emergency treatment:
excoriation and secondary infection. Eczema is not a. Epinephrine
infectious, nor is it managed by dietary restrictions. b. Corticosteroids
Pruritus, not pain, is associated with eczema. c. Narcan
d. Cromolyn sodium
98. A 5-year-old boy was brought to the Emergency (Answer: Prick tests determine allergens. Should the
Department after being burned trying to put out a fire child have an allergy, epinephrine might be needed to
that started in his closet where he was playing with counteract anaphylaxis.
matches. What should be the priority nursing

St. Louis Review Center Inc. – BUTUAN CITY; Telephone No. (085) 342 – 2339 46
103. A mother brings a 3-year-old child to the clinic for a precautions include:
well-child checkup. The child has not been to the clinic a. Storing all of this infant’s supplies separately
since 6 months of age. What is the priority of care for from the other children
this child? b. Wearing gloves when changing the child’s
a. Assess growth and development diapers
b. Begin dental care c. Always wearing gloves and isolation gowns
c. Update vaccinations when handling the infant
d. Complete hearing screening d. Minimizing contact with the infant when he is
(Answer: C) Rationale: Every time a child enters the febrile
healthcare system, the immunization status should be (Answer: B) Rationale: The HIV virus is spread by blood
checked. Some children have uncertain history of and body fluids. Clean gloves should be worn when
immunization because of parental noncompliance or changing the diapers as it exposes the worker to body
special circumstances such as being refugees. fluids. The other answers are incorrect.

104. The mother of a 1-year-old child says that 109. A mother overhears two nurses discussing a
breastfeeding her infant is sufficient to provide immunity. measles outbreak. The nurses are talking about the
She does not want to sigh the permit for immunizations. incubation period. The mother asks the nurses why it is
What is the nurse’s best response? important to know the incubation period for a childhood
a. Discuss activate and passive immunity disease. The nurse’s reply would be based on the
b. Tell her immunizations are legally mandatory knowledge that the incubation period:
c. Ask about the mother’s diet a. Describes a period when the child might be
d. Allow her the right to refuse contagious
(Answer: A) Rationale: Infants receive passive immunity, b. Determines the severity of the infection
which lasts 3 to 4 months through the placenta or c. Varies depending on the age of the child
breastmilk. Active immunity lasts long term and is d. Is a period of time when medications can
acquired by exposure to disease or immunizations. prevent the development of symptoms
Option 1 addresses the client’s need for information. (Answer: A) Rationale: The incubation period is the time
between exposure and outbreak of the disease. It is
105. The nurse is caring for several children on a often a period when the child can be contagious without
hospital unit where there has been a recent outbreak of others being aware of the possible exposure.
diarrhea. None of these children were admitted for
diarrhea, but the nurse is aware that they may be 110. An infant is admitted to the pediatric hospital
exposed. Of the children on the unit, the one most straight from the birth hospital with numerous congenital
susceptible would be the: defects and a diagnosis of rule out TORCH syndrome.
a. Toddler with SCID The father tells the pediatric nurse that he and his wife
b. Preschooler in traction for a fractured femur had planned a beautiful birth experience and can’t
c. School-age child with eczema believe what’s happened. An appropriate nursing
d. Teenager with frequent stools secondary to diagnosis for this family would be:
malabsorption syndrome a. Risk for caregivers role strain
(Answer: A) Rationale: The immunocompromised child b. Situational low self-esteem
would be the one at greatest risk for acquiring an c. Risk for altered parent/infant attachment
infectious organism. The other children would be at less d. Parental role conflict
risk for acquiring the gastrointestinal infection. (Answer: C) Rationale: With the birth of a less-than
expected infant, the parents may have difficulty
106. A child is admitted to the hospital with an allergic accepting the child. In addition, the prolonged
reaction. The physician orders a CBC with differential. hospitalization and separation from the parents inhibit
The nurse would expect to see an elevation in the level bonding, which could lead to altered attachment.
of:
a. RBCs 111. A school-age child is being admitted for surgical
b. Hemoglobin removal of a brain tumor. Expected nursing
c. Leukocytes assessments during the preoperative period would
d. Eosinophils include:
(Answer: D) Rationale: Eosinophils are the white blood a. Bulging fontanels
cell associated with allergic reactions. b. Vomiting
c. Drainage from the ear or nose
107. A child is being discharged from the nursery with a d. Elevated blood glucose levels
positive TORCH titer. Parents should be informed that: (Answer: B) Rationale: Vomiting is a symptom of
a. The child may shed the virus for a year increased intracranial pressure. Bulging fontanels would
b. TORCH is a genetic disorder not be present in a school-age child. Drainage from the
c. No follow-up is necessary ear or nose might indicate a basilar skull fracture, not a
d. Medication will not be needed for this condition brain tumor. Some brain tumors display the symptom of
(Answer: A) Rationale: TORCH is an acronym for a set diabetes insipidus, not diabetes mellitus, thus the
of microbes that includes toxoplasmosis, syphilis, symptom would be dilute urine rather than elevated
hepatitis, rubella, cytomegalovirus, and herpes simplex. blood glucose.
If an infant has one of the viruses, they could be shed for
up to 1 year. 112. A child is receiving chemotherapy to induce
remission in acute leukemia. When considering common
108. An infant with AIDS will be attending daycare. The side effects of chemotherapy, an appropriate nursing
daycare workers are concerned about spreading the diagnosis early in the course of therapy would include:
virus. The public health nurse is explaining to the a. Sleep pattern disturbance
workers the precautions they should take. These b. Altered mucous membranes

St. Louis Review Center Inc. – BUTUAN CITY; Telephone No. (085) 342 – 2339 47
c. Risk for infection 117. The nursing diagnosis for a child undergoing
d. Risk for impaired tissue perfusion peripheral chemotherapy for leukemia is Altered nutrition: less than
(Answer: B) Rationale: nausea and vomiting, anorexia, body requirements related to nausea and anorexia. An
mouth sores, constipation, and pain are early and appropriate goal for this client would be:
common side effects of chemotherapy. Bone marrow a. Administer antiemetics PRN
suppression reaches its peak 7 to 10 days after b. The child’s caloric intake will be within normal
induction. Sleep pattern disturbance may be related but range
is not directly caused by chemotherapy. c. The child does not complain of nausea
d. Intake and output are approximately equal
113. A client is to begin radiation therapy after the (Answer: B) Rationale: The client’s goal is stated in
removal of Wilms’ tumor. The parent statement that terms of behaviors of the child that demonstrate the
indicates a lack of understanding of related skin care problem is solved. Option 1 is a nursing action, not a
would be: goal. Absence of nausea does not guarantee adequate
a. “We will use loose-fitting clothes on our child.” intake. Equal intake and output does not indicate
b. “We will protect our child from sun exposure.” adequate nutrition.
c. “We will keep the area moist with Vaseline.”
d. “We will prevent our child from scratching the 118. A child is to receive chemotherapy intravenously
site.” with a vesicant drug. The nurse can ensure safe
(Answer: C) Rationale: Self-care during external administration of this drug by:
radiation therapy includes loose-fitting clothes, gentle a. Administering the drug using a positive pressure
washing with mild soap, avoiding sun exposure, and infusion pump
avoiding scratching and other irritation. Any lubricant b. Checking for blood return before, during and
must be water-soluble. after administration of the drug
c. Maintaining the infusion site below the level of
114. An adolescent on consolidation chemotherapy for the heart
acute lymphocytic leukemia (ALL) asks the nurse to d. Delivering the infusion as rapidly as possible
come quickly to evaluate “blood in my urine.” The nurse (Answer: B) Rationale: By checking for blood return
would do which of the following as the most important throughout the administration, the nurse can stop the
action? infusion at any time blood return does not occur. A
a. Explain this is normal for these drugs positive pressure infusion pump, maintaining the infusion
b. Measure intake and output site below the level of the heart, or rapid drug delivery
c. Force fluids to improve the hematuria does not guarantee the infusion will not extravasate.
d. Recognize that this is untoward and report the
event 119. A child with leukemia has developed pancytopenia.
(Answer: D) Rationale: This is an untoward effect of the Measures designed to reduce stomatitis in this child
commonly used cancer medication cyclophosphamide while receiving chemotherapy would include:
(Cytoxan) and should be reported. Fluids are usually a. Alcohol-based mouthwash to reduce oral
forced prior to administration and the bladder is emptied organisms
frequently to prevent hematuria. Measuring intake and b. Brushing the teeth twice a day with a firm-
output should be done routinely on all clients and is not bristled toothbrush
specific to managing this complication. c. Increasing intake of citrus juices, such as orange
juice, that contain vitamin C
115. A client is being admitted for mild neutropenia and d. Rinsing the mouth several times a day with plain
a severe oral monilial infection. The nurse should assign water
the child to which room? (Answer: D) Rationale: Studies have shown that simply
a. A semi-private room with a medical patient rinsing the mouth with water decreases the onset of
b. A semi-private room with a surgical patient stomatitis in clients receiving chemotherapy. Alcohol-
c. A private room without further precautions based mouthwash would be avoided as it is drying to the
d. A private room with protective isolation oral mucous membranes. A stiff toothbrush may cause
(Answer: C) Rationale: A private room assignment is the gums to bleed. Should oral lesions be present, acidic
indicated for children with chemotherapy-related foods ands liquids will increase discomfort.
neutropenia. Careful handwashing is also an essential
element to reduce the risk of infection. 120. During rounds, the interdisciplinary team is
discussing a child with leukemia who has just been
116. You are assigned to the postoperative care of a diagnosed as terminally ill. The nurses describe the
client with a below-the-knee amputation for osteogenic mother’s behavior as angry, claiming the nurses are not
sarcoma. Nursing care of the child would include: providing care for her child. The team leader will focus
a. Maintaining bedrest until able to use permanent on the probable cause of the mother’s anger, which is:
prosthesis a. Poor care on the part of the nurses
b. Keeping stump elevated continuously until b. Lack of attention for the mother’s needs
prosthesis applied c. Overwhelming guilt for having caused the
c. Applying a dressing to the stump that allows leukemia
continuous visualization of the distal stump d. A stage of bereavement over the anticipated
d. Encouraging early visits from friends loss of the child
(Answer: D) Rationale: Nursing care must be supportive (Answer: D) Rationale: The stages of grief and
of body image adjustment. The child would be bereavement include denial, anger, bargaining,
encouraged to sit in a chair and ambulate on crutches depression, and acceptance. The anger expressed may
while waiting for the permanent prosthesis. The stump often be displaced and directed towards persons who
dressing is a continuous ace bandage, which supports have a role in the loss. Nursing and other healthcare
the stump shape in preparation for the prosthesis. personnel must be aware of this in order to help the
family cope with the impending loss.

St. Louis Review Center Inc. – BUTUAN CITY; Telephone No. (085) 342 – 2339 48
increased desire to drink fluids and a higher specific
121. The best rationale to give parents who are gravity caused by the concentration of urine. Although
questioning the use of elbow restraints with their child the heart rate would be elevated, the fontanels are
who has had cleft palate repair is: closed on a 4-year-old. The degree of dehydration is
a. “This device is frequently used postoperatively based on the percent of weight loss so a weight gain
to protect the IV site in small children.” would not be likely. Diminished urine output is an
b. “The restraints will help us maintain proper body expected normal finding in dehydration, however the
alignment.” urine specific gravity would also be affected.
c. “Elbow restraints are used postoperatively to
keep their hands away from the surgical site.” 126. While performing a newborn assessment, the nurse
d. “The restraints help us remember that the child notices the infant is having difficulty breathing. Nasal
is NPO after surgery.” flaring, cyanosis, ands retractions are observed and
(Answer: C) Rationale: Elbow restraints are used to keep there are no breath sounds on the left side. The apical
hands away from the mouth after cleft palate surgery. pulse is auscultated on the right side of the chest. The
This precaution will be maintained at home until the nurse would notify the physician immediately because
palate is healed, usually 4 to 6 weeks. he or she suspects:
a. Diaphragmatic hernia
122. The nurse is caring for an infant vomiting secondary b. Pyloric stenosis
to pyloric stenosis. The mother questions why the c. Cleft palate
vomitus of this child appears different from that of her d. Omphalocele
other children when they have the flu. The nurse would (Answer: A) Rationale: Clinical findings will vary in
explain that the emesis of an infant with pyloric stenosis infants born with congenital diaphragmatic hernias but
does not contain bile because: the first indications are of respiratory distress. Further
a. The GI system is still immature in newborns and assessment will reveal bowel sounds auscultated over
infants the chest, cardiac sounds auscultated over the chest,
b. The obstruction is above the bile duct cardiac sounds on the right of the chest, and a sunken
c. The emesis is from passive regurgitation abdomen with a barrel-shaped chest.
d. The bile duct is obstructed 127. The nurse has taught dietary restrictions to the 7-
(Answer: B) Rationale: In pyloric stenosis, bile is unable year-old child with celiac disease. After teaching, the
to enter the stomach from the duodenum because the child is allowed to choose a correct menu. The nurse
pylorus muscle is hypertrophied, which causes the would know that teaching was effective when the child
obstruction. chooses:
a. Beef and barely soup, rice cakes, and celery
123. The nurse is teaching the parent of a child with b. Ham and cheese sandwich with lettuce and
celiac disease about the dietary restrictions. The nurse tomato on rye toast
would explain that the most appropriate diet for their c. Beef patty on a hamburger bun and French fries
child is: d. Baked chicken, green beans, and a slice of
a. Gluten-free cornbread
b. Salt-free (Answer: D) Rationale: Celiac disease is characterized
c. Fat-free by intolerance for gluten. Gluten is found in wheat,
d. High-calorie, low-fat barley, rye, and oats. This includes bread, cake,
(Answer: A) Rationale: Most children who remain on a doughnuts, cookies, and crackers, as well as processed
gluten-free diet are healthy and free of symptoms and foods that contain gluten as filler.
complications.
128. An infant returns from initial surgery for
124. A high school experiences an outbreak of hepatitis Hirschsprung’s disease. Because of the type of surgery
B. In teaching the high school students about hepatitis B, the child had, the nurse would exclude from the routine
the nurse would explain: postoperative plan-of-care instructions to:
a. Hepatitis B cannot exist in carrier state a. Maintain the child NPO until bowel sounds
b. Hepatitis B is primarily transmitted through the return
fecal-oral route b. Monitor rectal temperature every 4 hours
c. Immunity to all types of hepatitis will occur after c. Reunite the parents with the child as soon as
this current attack possible
d. Hepatitis B can be prevented by receiving the d. Assess the surgical site every 2 hours
HBV vaccine (Answer: B) Rationale: The corrective surgery for
(Answer: D) Rationale: HBV vaccine provides active Hirschsprung’s disease requires pulling the end of the
immunity and current recommendations include normal bowel through the muscular sleeve of the rectum.
immunizations for all newborns, as well as several high- With this type of procedure, rectal temperatures and any
risk groups. Hepatitis B is spread by blood and body invasive procedure would be avoided to allow proper
fluids, including through sexual contact. healing to occur.

125. A 4-year-old child is admitted to the unit with 129. A 3-month-old infant has gastroesophageal reflux
moderate dehydration. Which of the following symptoms (GER) but is thriving without other complications. The
led the physicians to the diagnosis of moderate mother wants to know if there is anything she can do
dehydration in this child? differently to decrease the reflux. Which of the following
a. Elevated heart rate and sunken fontanel interventions should the nurse suggest to minimize
b. Increased thirst and urine specific gravity of reflux?
1.038 a. Discontinue breast-feeding immediately
c. Weight gain and decreased skin turgor b. Decrease frequency of feedings as much as
d. Oliguria and urine specific gravity of 1.010 possible
(Answer: B) Rationale: The nurse would expect an c. Place the baby in prone position with the head

St. Louis Review Center Inc. – BUTUAN CITY; Telephone No. (085) 342 – 2339 49
elevated b. Delay the introduction of table food in the diet
d. Place the infant in a car seat after feeding c. Restrict the amount of milk or formula in the
(Answer: C) Rationale: Infants with GER should be given baby’s diet to 1 quart per day
small, frequent feedings. After a feeding the infant d. Provide dietary iron sources such as peanuts
should be placed in a prone position with the head of the ands unsweetened chocolates
bed elevated. After a feeding the infant should be placed (Answer: C) Rationale: Many infants with nutritional
in a prone position with the head of the bed elevated. A anemia rely primarily on the milk/formula for dietary
harness can be used to help maintain this position. intake and refuse solid foods. When the milk/formula is
Infant seats should be avoided because of the increased limited, the child will be more willing to take solid foods.
intra abdominal pressure this position creates. Cow’s milk is a poor source if iron. Peanuts and
unsweetened chocolates are sources of iron but are not
130. A 10-year-old boy has been admitted with a appropriate for a child this age.
diagnosis of “rule out appendicitis.” While the nurse was
conducting a routine assessment, the boy stated, “It 134. A child is being admitted to the unit with
doesn’t hurt anymore.” The nurse suspects that: thalassemia major. In preparing client assignments, the
a. The boy is afraid of going to surgery charge nurse would wan tot assign a nurse to this child
b. The boy is having difficulty expressing his pain who can:
adequately a. Teach dietary sources of iron
c. The appendix has ruptured b. Administer blood transfusions
d. This is a method the boy uses to receive c. Work with a dying child
attention d. Monitor the child for bleeding tendencies
(Answer: C) Rationale: Signs and symptoms of a (Answer: B) Rationale: Blood transfusions are utilized in
ruptured appendix include fever, sudden relief from order to maintain normal hemoglobin levels. This child
abdominal pain, guarding, abdominal distention, rapid has an excess of iron secondary to repeated
shallow breathing, pallor, chills, and irritability. transfusions and, thus, iron supplements will not be
necessary. The other therapies are inappropriate for the
131. A 2-year-old with hemophilia is being discharged, child with thalassemia major.
and the nurse is completing discharge teaching with his
parents. Which of the following statements by the 135. The nurse is caring for a child who is being treated
parents indicates they require further teaching regarding for extensive bleeding in the Emergency Department.
hemophilia? The source and extent of bleeding are being determined
a. “It is good to know that his sister will not get as the nurse is trying to control the bleeding. Which of
hemophilia also.” the following actions takes priority?
b. “If our son has a temperature, we will not give a. Obtain the client’s history
aspirin or ibuprofen, only acetaminophen.” b. Talk with the family regarding the risk of HIV and
c. “We will get a Medic-Alert bracelet for our son as hepatitis C with blood transfusions
soon as we get home.” c. Replace blood volume
d. “We will be sure to watch our son very closely to d. Provide psychosocial support to the family
make sure he does not have another episode of (Answer: C) Rationale: Appropriate oxygenation is not
bleeding.” possible when there is significant loss of blood volume.
(Answer: D) Rationale: It is not possible for parents of a Replacing the blood volume is critical to saving the
hemophiliac to prevent a bleeding episode, no matter child’s life, and it is imperative that replacement occurs
how careful they are. The nurse should reinforce this prior to any of the listed nursing actions.
information along with methods for decreasing the
chance of an injury that will lead to a bleeding episode. 136. The nurse is working with the family of a toddler
The other statements all indicate an appropriate who is being treated for iron-deficiency anemia. In
understanding of hemophilia. teaching dietary considerations, the nurse will instruct
the family to add sources of iron and:
132. The parents of a client with sickle cell anemia are a. Vitamin D and thiamine
asking for information about future pregnancies. Neither b. Calcium and riboflavin
parent has sickle cell anemia. The nurse would provide c. Carbohydrates and vitamins
them with the information that nay future pregnancies d. Folic acid and proteins
will have a: (Answer: D) Rationale: Folic acid potentiates the
a. 1 in 4 chance of producing a child with sickle cell removal of iron form ferritin, which makes it further
trait available for heme production. The synthesis of albumin,
b. 1 in 4 chance of producing a child with sickle cell blood proteins, fibrinogen, and hemoglobin is dependent
anemia upon the presence of proteins. None of the others are
c. 1 in 2 chance of producing a child with neither involved in building RBCs.
sickle cell disease or trait
d. 1 in 2 chance of producing a child with sickle cell 137. The elementary school nurse is assessing and
anemia giving initial care to a hemophiliac who has a significant
(Answer: B) Rationale: Sickle cell anemia is an pain in his knee. The nurse suspects hemarthrosis. As
autosomal recessive condition. Therefore, if both parents the nurse waits for his family to pick up the child, the
have the trait, each pregnancy carries a 25 percent risk nurse would:
that the child will have the disease. a. Maintain joint mobility with passive range of
motion exercises
133. The nurse is working with the family of an 8-month- b. Elevate the leg above his heart
old infant with severe nutritional anemia. In providing c. Administer children’s aspirin or ibuprofen for
dietary recommendations, the nurse should instruct the pain
family to: d. Apply warm soaks to reduce the swelling
a. Switch the baby to cow’s milk (Answer: B) Rationale: Elevate the leg above the level of

St. Louis Review Center Inc. – BUTUAN CITY; Telephone No. (085) 342 – 2339 50
the heart to reduce bleeding. Aspirin or aspirin-like
products such as ibuprofen interfere with the clotting
mechanisms. During active bleeds, the joint should be
immobilized. Warm soaks would promote bleeding; ice
packs should be used instead.

138. The nurse has admitted a 2-year-old in vaso-


occlusive crisis. AS the nurse starts the initial
assessment, the child insists upon lying in bed, on her
side with her knees flexed to the abdomen. The nurse
would want to further assess the child for the presence
of:
a. Stomach pain
b. Nausea
c. Constipation
d. Fear secondary to the impact of hospitalization
(Answer: A) Rationale: Such positioning indicates the
likelihood of abdominal pain. Nausea or constipation
does not generally cause a child to self-position as
described. Fear related to the hospitalization would be
common in a child this age. However, if this were the
case, it is more likely the child would seek refuge in the
arms of one of her parents.

139. The 10-year-old client in the Emergency


Department has CBC results that include hemoglobin of
eight g/dL and hematocrit of 24 percent. The nursing
activity with the highest priority is:
a. Assessing and promoting skin integrity
b. Promoting hydration
c. Promoting nutrition
d. Conserving energy
(Answer: D) Rationale: Such lab results indicate severe
anemia. Fatigue results when the oxygen-carrying
capacity of RBCS is impaired and cellular hypoxia is
present. Fatigue can be diminished and oxygen
depletion limited when the client’s energy is conserved.
There will be an increased oxygen requirement and
increased fatigue with increased mobility. Increasing
general hydration without transfusing RBCs will not
positively affect the anemic state. Skin integrity is not a
high priority at this point. Although improving nutrition is
appropriate, the response would not be immediate. The
priority activity would be conserving energy and reducing
cardiac stress.

140. The nurse is caring for a child diagnosed with


thalassemia major who is receiving her first chelation
therapy. The parents ask the purpose of chelation
therapy. The best response by the nurse is that chelation
therapy is done to:
a. Decrease the risk of hypoxia
b. Decrease the risk of bleeding
c. Eliminate excess iron
d. Prevent further sickling of RBCs
(Answer: C) Rationale: Chelation therapy works to rid
the body of excess iron storage that results from the
frequent hemoglobin. Chelation will have no effect upon
hypoxia or bleeding. Sickling of RBCs does not occur
with thalassemia.

St. Louis Review Center Inc. – BUTUAN CITY; Telephone No. (085) 342 – 2339 51

Potrebbero piacerti anche